Neuro Flashcards

1
Q

Which of the following features describes an intra-axial mass? [B3 Q14]
A. Cortex displaced towards bone
B. Mass contiguous with dura
C. Buckling of grey and white matter
D. Widened subarachnoid cistern
E. Dural feeding arteries

A

A
B-E are extra-axial features

How well did you know this?
1
Not at all
2
3
4
5
Perfectly
2
Q

A mass is seen peripherally in the middle cranial fossa on MR of the brain. Which of the following imaging features favours an intra-axial rather than an extra-axial location? [B4 Q17]
a. buckling of the grey–white matter interface
b. expansion of the cortex of the brain
c. expansion of the subarachnoid space
d. medial displacement of pial blood vessels by the mass
e. the mass has a dural base

A

b. expansion of the cortex of the brain
Once the presence of a mass has been established, the radiologist must determine whether the mass is intra-axial (arising within the brain parenchyma) or extra-axial (arising outside the brain substance) to formulate an appropriate differential diagnosis. An extra-axial mass characteristically causes buckling of the grey–white matter interface, expansion of the subarachnoid space at its borders, and medial displacement of the vessels in the subarachnoid space. A dural base is also a feature of an extra-axial mass. Intra-axial masses characteristically cause expansion of the cortex of the brain but no expansion of the subarachnoid space, and pial vessels may be seen peripheral to the mass.

How well did you know this?
1
Not at all
2
3
4
5
Perfectly
3
Q

Which of the following is the most common radiation-induced CNS tumour? [B4 Q60] {will repeat again in respective tumour section}
a. ependymoma
b. oligodendroglioma
c. lymphoma
d. glioblastoma multiforme
e. meningioma

A

e. meningioma
Meningioma is the most common radiation-induced CNS tumour and has been particularly associated with low-dose radiation treatment for tinea capitis. For the diagnosis of radiation-induced meningioma to be made, the meningioma must arise in the radiation field, appear after a latency period of years and should not have been the primary tumour irradiated. Radiation-induced meningiomas are more frequently multiple and have higher recurrence rates than non-radiation-induced tumours.

How well did you know this?
1
Not at all
2
3
4
5
Perfectly
4
Q

A 40-year-old man is being investigated by the neurologists for new onset epilepsy. An electroencephalogram (EEG) indicates an epileptogenic focus in the left temporal lobe. An MRI is carried out. The hippocampal structures are unremarkable. A 2-cm lesion is noted in the subcortical region of the left temporal lobe. This lesion demonstrates mild enhancement. Areas of low signal on T1WI and T2WI are felt to represent foci of calcification. There is also an area of high signal on T1WI and T2WI seen inferiorly within the lesion, which probably represents an area of haemorrhage. There is a rim of surrounding oedema noted on T2WI and fluid-attenuated inversion recovery (FLAIR). What is the most likely diagnosis? [B1 Q5]
A. Haemangioblastoma.
B. Desmoplastic infantile ganglioglioma (DIG).
C. Dysembryoplastic neuroepithelial tumour (DNET).
D. Pleomorphic xanthoastrocytoma (PXA).
E. Oligodendroglioma.

A

E. Oligodendroglioma.
Prior to evaluating the imaging characteristics, the patient’s demographics should be considered in this case, as in all cases of intracranial masses, as it will help limit the differential diagnosis significantly. The patient is an adult, thus making DIG and DNET unlikely. Secondly, consider the temporal lobe location. Haemangioblastomas are usually infratentorial in location, especially in the absence of a history of von Hippel–Lindau syndrome. All the other lesions are classical temporal lobe tumours. Finally, considering the imaging characteristics, both PXA and oligodendrogliomas may be entirely solid, but cysts are commonly seen in PXAs. Calcification is seen in 60–80% of oligodendrogliomas but is rarely seen in PXAs. Similarly, haemorrhage, while not typical of oligodendrogliomas, is rare in PXAs.

How well did you know this?
1
Not at all
2
3
4
5
Perfectly
5
Q

An 8-year-old female patient presents to your paediatric neurology service with a history of increasing ataxia, repeated headaches, and vomiting, increasing in severity over the last 5 months. Clinical examination reveals marked cerebellar signs of past pointing and dysdiadochokinesis. An MRI is requested, which shows a solid mass in the posterior fossa measuring 2 cm in size. This mass arises in the left cerebellar hemisphere and displaces the fourth ventricle. It is of low intensity on T1WI and high signal on T2WI. There is only a small rim of surrounding oedema. The lesion demonstrates relatively homogeneous moderate enhancement. There is no evidence of subarachnoid seeding. What is the most likely diagnosis? [B1 Q14]
A. Pilocytic astrocytoma.
B. Ependymoma.
C. Medulloblastoma.
D. Metastasis.
E. Lhermitte–Duclos syndrome

A

A. Pilocytic astrocytoma.
This question deals with the classical neurological differential diagnosis of a posterior fossa mass in a child. While there are many causes, pilocytic astrocytoma and medulloblastoma account for over 60% of all childhood posterior fossa masses. Pilocytic astrocytomas have a classical appearance of being cystic lesions with an avidly enhancing mural nodule. However, 30% of pilocytic astrocytomas are solid tumours. In differentiating them from medulloblastomas, pilocytic astrocytomas often arise more peripherally and displace the fourth ventricle, whereas medulloblastomas usually arise centrally from the vermis. Subarachnoid seeding is seen in up to 50% of cases of medulloblastoma. Ependymomas are also included in the differential. As these arise from the ependyma lining the ventricle, they tend to be centred on the fourth ventricle in children. Metastases are the most common cause of a posterior fossa mass in adults but are less common in children.

How well did you know this?
1
Not at all
2
3
4
5
Perfectly
6
Q

A patient is having an MRI scan carried out to investigate a possible right frontal astrocytoma, incidentally detected on CT following a head injury. The MRI features are typical of an astrocytoma, with no evidence of necrosis or callosal involvement to indicate glioblastoma multiforme (GBM). MRS has been carried out to help assess the grade of this tumour. What MRS features would indicate a high-grade lesion? [B1 Q17]
A. Elevated choline, reduced N-acetyl aspartate (NAA), choline/creatine (Cho/Cr) ratio of 1.
B. Elevated choline, reduced NAA, Cho/Cr ratio of 2.
C. Normal choline, elevated NAA.
D. Reduced choline, reduced NAA. Cho/Cr ratio of 1.2.
E. All normal, these are unaffected by tumour grade.

A

B. Elevated choline, reduced NAA, Cho/Cr ratio of 2.
NAA is thought to be a marker of neuronal integrity, choline indicates cell turnover, and creatine indicates cell metabolism. Lactate is not detectable in normal brain spectra but is elevated in inflammation, infarction, and some neoplasms. Most brain conditions, whether neoplastic, vascular, or demyelinating, are associated with a reduction in NAA. A notable exception is Canavan’s disease, which causes a rise in NAA. Choline is elevated in many disorders but is markedly increased in high-grade neoplasms. It has been reported that the ratio of choline to creatine can be used to help grade tumours, with a ratio over 1.5 indicating high grade in most cases. A reduced choline and NAA in an area of tumour can indicate necrosis.

How well did you know this?
1
Not at all
2
3
4
5
Perfectly
7
Q

A 45-year-old woman presents with a several-month history of neck pain and gradually progressive weakness and paraesthesia in the upper limbs. An MRI scan of the cervical spine is performed, and this shows a well-defined central intramedullary mass in the mid-cervical spinal cord. The mass is generally slightly hyperintense on T2WI, but also has a few low signal peripheral areas. It enhances homogeneously with gadolinium. What is the most likely diagnosis? [B1 Q27] {will repeat again in Spinal Cord Tumour Section}
A. Astrocytoma.
B. Metastasis.
C. Haemangioblastoma.
D. Ganglioglioma.
E. Ependymoma.

A

E. Ependymoma.
Ependymoma is the most common intramedullary neoplasm in adults. It tends to be centrally located within the cord, unlike astrocytoma, which can be eccentric. Astrocytoma can have a longer segment of cord involvement than ependymoma and may have a more infiltrative margin. The peripheral low signal areas seen on T2WI in ependymoma are related to haemosiderin deposition from prior haemorrhage. Haemangioblastoma is more often seen in the dorsal cord than the cervical cord and is typically a small well-defined lesion. It may have an associated cord cyst or syrinx. Flow voids may be seen within the lesion, from dilated vascular channels. Ganglioglioma is a very rare, slow-growing tumour of low malignant potential. The imaging appearance is non-specific, but there are some findings that may suggest the diagnosis. Compared with other spinal cord tumors, gangliogliomas are more likely to involve long segments of the cord (greater than four levels, up to the whole cord), to be associated with bone erosion or scalloping, to have tumoral cysts, and to have areas of mixed high signal on precontrast T1WI. Intramedullary metastasis represents less than 5% of intramedullary lesions. They usually occur in the setting of advanced malignant disease, typically from a lung or breast primary. The spinal cord oedema can seem out of keeping with the small size of the metastatic lesion.

How well did you know this?
1
Not at all
2
3
4
5
Perfectly
8
Q

When on call you are asked to perform a CT head scan for a 17-year-old male who presents with seizures. He is unable to provide a history. A look on the computer system shows that he has had previous regular abdominal ultrasounds and an echocardiogram as a child. Brain CT shows a hypodense, well-demarcated, rounded mass in the region of the foramen of Monro. It is partially calcified, and it demonstrates uniform enhancement. What is the most likely diagnosis? [B2 Q4] {will repeat again in periventricular tumour section}
a. Colloid cyst
b. Giant cell astrocytoma
c. Metastasis
d. Lymphoma
e. Haemangioblastoma

A

Giant cell astrocytoma
The main differential for a mass at the foramen of Monro is between a colloid cyst and a subependymal giant cell astrocytoma. The latter is associated with tuberous sclerosis (TS). Renal involvement is also relatively common in TS and patients regularly have surveillance renal ultrasounds. The echocardiogram was performed to assess for cardiomyopathy/rhabdomyoma. Other central nervous system findings in patients with TS include: 1. Subependymal hamartomas – these are nodular and irregular and can be located anywhere along the ventricular walls but predominantly occur around the foramen of Monro or along the lateral ventricles. In infants, with unmyelinated white matter the lesions are usually hyperintense on T1 and hypointense on T2. The reverse is seen in adults. 2. Subcortical and cortical hamartomas (tubers) – these appear as broad cortical gyri with abnormalities in the adjacent white matter. They frequently calcify but enhancement is extremely rare. 3. Heterotopic grey matter islands in white matter. These may calcify and show contrast enhancement.

How well did you know this?
1
Not at all
2
3
4
5
Perfectly
9
Q

A 40-year-old man undergoes investigation for seizures. Head CT with and without contrast shows a large, round, sharply marginated, hypodense mass involving the cortex and subcortical white matter of the left frontal lobe. The mass contains large nodular clumps of calcification. There is surrounding oedema and ill-defined enhancement. MRI demonstrates a heterogeneous mass which is predominantly isointense to grey matter on T1 and hyperintense on T2. There is moderate enhancement. What is the most likely diagnosis? [B2 Q20]
a. Astrocytoma
b. Ganglioglioma
c. Ependymoma
d. Glioblastoma
e. Oligodendroglioma

A

Oligodendroglioma
This is an uncommon glioma which usually presents as a large mass at the time of diagnosis. Mean age is 30–50 years and they are more common in men than women. The majority are in the frontal lobe (60%), although they can occur anywhere within the central nervous system, including the cerebellum, brainstem, spinal cord, ventricles, and optic nerve. Large nodular clumps of calcifications are present in up to approximately 90% of tumours. Cystic degeneration and haemorrhage are uncommon. Prognosis depends on the grade of the tumour. High-grade tumours show 20% ten-year survival whereas low-grade tumours show 46% ten-year survival. Although astrocytomas can calcify, the calcifications are rarely large and nodular. Glioblastomas rarely calcify. Gangliogliomas are more common in the temporal lobes and deep cerebral tissues and most of them (80%) occur below the age of 30 years. Ependymomas often demonstrate fluid levels due to internal haemorrhage.

How well did you know this?
1
Not at all
2
3
4
5
Perfectly
10
Q

A 36-year-old female presents following a tonic-clonic seizure. Over the preceding months she had suffered with progressive, severe headaches. Contrast-enhanced CT brain shows lateral displacement of the internal capsules by enlarged thalami but no abnormal enhancement. T2-weighted MRI demonstrates a diffuse, contiguous area of hyperintensity involving the thalami, caudate and lentiform nuclei, the splenium of the corpus callosum and the periventricular white matter. There is only minimal mass effect. T1-weighted gadolinium imaging shows no enhancement. What is the most likely diagnosis? [B2 Q50]
a. Multiple sclerosis
b. Gliomatosis cerebri
c. Viral encephalitis
d. Adrenoleukodystrophy
e. Vasculitis

A

Gliomatosis cerebri (GC)
GC is a diffusely infiltrative glioma that may be present with or without a dominant mass. It must, however, involve two or more lobes and usually involves contiguous areas. It affects all age groups and can be of varying histological grade. Presentation may be enigmatic as the normal cerebral architecture is usually preserved. Alternatively, patients present with seizures, headache and personality disorders. Prognosis is poor. MRI findings include diffuse T2 (and proton density) hyperintensity throughout the white matter that usually extends to involve the deep grey nuclei with enlargement of cerebral structures. It is often bilateral and symmetric with minor mass effect and absence of necrosis. The differential diagnosis of symmetric white matter lesions includes microvascular change, encephalitis, demyelinating disease, vasculitis, and leukoencephalopathy. GC is the most likely diagnosis in this scenario as there is involvement of the corpus callosum and the pattern is infiltrative with enlargement of the thalami (cerebral structures).

How well did you know this?
1
Not at all
2
3
4
5
Perfectly
11
Q

Which brain tumour has the greatest incidence across all age groups? [B3 Q9]
A. Glioma
B. Meningioma
C. Metastases
D. Pituitary adenoma
E. Haemangioblastoma

A

Glioma

Gliomas consist of astrocytoma, oligodendrogliomas, paragangliomas, gangliogliomas and medulloblastomas.

How well did you know this?
1
Not at all
2
3
4
5
Perfectly
12
Q

Which is the most common location of oligodendroglioma? [B3 Q23]
A. Frontal lobe
B. Temporal lobe
C. Parietal lobe
D. Occipital lobe
E. Cerebellum

A

Frontal
Most commonly involve cortical and subcortical white matter, occasionally through corpus callosum as butterfly glioma.

How well did you know this?
1
Not at all
2
3
4
5
Perfectly
13
Q

Which of the following conditions will typically demonstrate unrestricted diffusion on MR DWI and ADC map? [B4 Q6]
a. epidermoid cyst
b. acute infarction
c. cerebral abscess
d. glioblastoma multiforme
e. viral encephalitis

A

Glioblastoma Multiforme
Diffusion-weighted MRI provides image contrast which is different from that provided by conventional MR techniques. The sequence enables the measurement of net macroscopic water movement, which is anisotropic (varies in different directions) particularly in white matter. Restricted diffusion is seen as high signal on DWI (which is a T2W image with signal degraded by diffusion) and low signal on the ADC map. Restricted diffusion occurs in tissue that does not allow free movement of water molecules, such as areas of infection due to the high viscosity and cellularity of pus. Similarly, epidermoid cysts are very cellular and so also show restricted diffusion, a feature that helps distinguish them from arachnoid cysts, which are fluid structures. In stroke, restriction in water diffusion occurs within minutes after the onset of ischaemia. The basis of this change is not completely clear but is thought to be related to the cytotoxic oedema seen in ischaemic cells due to the impairment of the Na+/K+ ATPase pumps (which are very energy dependent), leading to loss of ionic gradients and a net translocation of water from the extracellular to the intracellular compartment, where water mobility is relatively more restricted.

How well did you know this?
1
Not at all
2
3
4
5
Perfectly
14
Q

A 46-year-old female presents with back pain and increasing weakness of the lower limbs. An MR scan shows a lesion in the cord at the level of T11. Which of the following features would suggest an ependymoma rather than demyelination? [B4 Q16]
a. multiple lesions
b. expansion of the cord
c. high signal on T2W images
d. enhancement with gadolinium
e. peripheral low signal on all sequences

A

Expansion of the cord
Ependymomas are the commonest tumour of the spinal cord in adults, accounting for 40–60% of cord tumours. They present with a long history of pain, and sensory or motor disturbance. Less commonly, bladder and bowel dysfunction may occur. Expansion of the cord is more often seen with ependymomas than with demyelination. Both lesions may enhance and have high signal on T2W images, but multiplicity is more often seen with demyelination. Peripheral low signal, usually indicating haemosiderin, is not a feature of either of these lesions.

How well did you know this?
1
Not at all
2
3
4
5
Perfectly
15
Q

A 5-year-old boy undergoes CT of the brain for investigation of headaches, vomiting and ataxia. This demonstrates a well-defined, multilobulated, iso-dense mass within the fourth ventricle containing areas of punctate calcification. The mass is seen to extend out of the foramina of Luschka into the cerebellopontine angles. There is associated hydrocephalus. What is the most likely diagnosis? [B4 Q31]
a. metastasis
b. haemangioblastoma
c. juvenile pilocytic astrocytoma
d. medulloblastoma
e. ependymoma

A

Ependymoma
Ependymomas most commonly arise in the floor of the fourth ventricle and are usually isodense. They have a greater incidence of calcification than other posterior fossa paediatric tumours; it is typically punctate and seen in 40–50% of cases. A characteristic feature of ependymomas is their propensity to extend through and widen the foramina of Luschka and Magendie. Juvenile pilocytic astrocytomas are the commonest paediatric infratentorial neoplasms and typically occur in the cerebellar hemispheres. They appear cystic with an enhancing mural nodule. Medulloblastomas tend to be homogeneous hyperdense lesions located in the vermis, and the presence of calcification is uncommon. Metastases are the commonest infratentorial tumour to occur in adults but are uncommon in children. Haemangioblastomas usually occur in young adults and are classically cystic masses with a solid mural nodule.

How well did you know this?
1
Not at all
2
3
4
5
Perfectly
16
Q

A 65-year-old, previously well man with a short history of headaches and behavioural change undergoes CT of the brain. This demonstrates an irregular, ill-defined mass in the left frontal lobe extending across the corpus callosum to involve the right frontal lobe. The mass is of low attenuation and contains cystic areas, demonstrates ring enhancement following intravenous contrast, and has considerable surrounding oedema. What is the most likely diagnosis? [B4 Q39]
a. progressive multifocal leukoencephalopathy
b. glioblastoma multiforme
c. lymphoma
d. abscess
e. metastasis

A

Glioblastoma Multiforme
Glioblastoma multiforme is the most malignant form of astrocytoma. It occurs in older patients, and most commonly affects the deep white matter of the frontal lobes. Classic appearances are of an irregular, ill-defined hypodense mass with necrosis, haemorrhage and extensive surrounding white matter oedema. Ninety per cent of cases show enhancement, which may be diffuse, heterogeneous or ring-like. Tumour spread is directly along white matter tracts, and commonly occurs across the corpus callosum to involve both frontal lobes (butterfly glioma). Lymphomas also have a propensity to involve the corpus callosum but usually are slightly hyperdense due to a high nuclearto-cytoplasmic ratio. Metastases may also involve the corpus callosum.

How well did you know this?
1
Not at all
2
3
4
5
Perfectly
17
Q

A 28-year-old woman presents with a history of headaches and refractory temporal lobe epilepsy. CT of the brain demonstrates a mixed solid–cystic, intraparenchymal mass located peripherally in the right temporal lobe, which contains calcification and demonstrates faint enhancement following intravenous contrast. There is minimal surrounding oedema. What is the most likely diagnosis? [B4 Q47]
a. arachnoid cyst
b. ganglioglioma
c. epidermoid
d. meningioma
e. dysembryoplastic neuroepithelial tumour

A

Ganglioglioma
Gangliogliomas are low-grade tumours with a good prognosis, generally occurring in patients under the age of 30. Typical presentation is with focal seizures, and ganglioglioma is the most common tumour seen in patients with chronic temporal lobe epilepsy. They are usually well circumscribed, hypo- or iso-dense lesions in the temporal lobes. Calcification (30%) and cyst formation (50%) are common features. There is usually minimal mass effect and surrounding oedema. Meningiomas commonly calcify and have minimal surrounding oedema, but are extra-axial, and usually demonstrate intense uniform enhancement following intravenous contrast. Dysembryoplastic neuroepithelial tumours are commonly associated with partial complex seizures, but usually occur before the age of 20, and characteristically appear as a soap-bubble, multicystic lesion, which may remodel the calvarium. Epidermoid and arachnoid cysts are of CSF density, do not enhance with contrast and are extra-axial lesions.

How well did you know this?
1
Not at all
2
3
4
5
Perfectly
18
Q

A 44-year-old man presents with a long history of headaches and more recent onset of seizures. CT of the brain demonstrates an oval, well-defined, heterogeneous, hypodense mass containing large nodular clumps of calcification located peripherally in the right frontal lobe. The mass extends to the cortical margin, and there is erosion of the inner table of the skull. There is minimal surrounding vasogenic oedema. What is the most likely diagnosis? [B4 Q50]
a. meningioma
b. oligodendroglioma
c. astrocytoma
d. glioblastoma multiforme
e. ganglioglioma

A

Oligodendroglioma
Oligodendrogliomas are slow-growing tumours, usually presenting in adults aged 30–50 years. They occur most commonly in the frontal lobe, often extending to the cortex and eroding the inner table of the skull. Calcification is seen in 70% of cases, typically appearing as large nodular clumps, with a relative absence of surrounding oedema. Astrocytomas also appear as hypodense calcified lesions but lack calvarial erosion. Glioblastoma multiforme typically has considerable surrounding oedema and rarely calcifies. Gangliogliomas show calcification in a third of cases but tend to occur in younger patients and have a predilection for the temporal lobes.

How well did you know this?
1
Not at all
2
3
4
5
Perfectly
19
Q

A 32-year-old man with a 3-month history of headaches presents to the Accident & Emergency Department with tonic-clonic seizures. MRI shows a 5 cm intra-axial lesion in the left frontal lobe. The lesion appears hypointense on T1 and hyperintense on T2 to brain parenchyma. No significant surrounding oedema is seen and there is no enhancement with gadolinium. The most likely diagnosis is? [B5 Q24]
(a) Oligodendroglioma
(b) Astrocytoma
(c) Arachnoid cyst
(d) Metastases
(e) Lymphoma

A

Astrocytoma
These MRI appearances are typical of a grade II astrocytoma. Grade III are more infiltrative and show more surrounding oedema. Oligodendrogliomas show calcifications. Arachnoid cysts show CSF density on all sequences. Metastatic lesions and lymphoma enhance with gadolinium.

How well did you know this?
1
Not at all
2
3
4
5
Perfectly
20
Q

A 42-year-old man presents with increasing headache and blurred vision. CT of the head shows a large lesion in the periphery of the left parietal lobe with extensive calcification. The lesion shows heterogeneous contrast enhancement. There is a mass effect with midline shift. What is the most likely diagnosis? [B5 Q32]
(a) Ganglioglioma
(b) Calcified arteriovenous malformation
(c) Oligodendroglioma
(d) Pilocytic astrocytoma
(e) Meningioma

A

Oligodendroglioma
These tumours are seen in young adults and are usually located in the peripheral cerebrum. They typically begin in the hemispheric white matter and grow towards the cortex, are well circumscribed but non-encapsulated, and calcification is a common feature.

How well did you know this?
1
Not at all
2
3
4
5
Perfectly
21
Q

A 42-year-old man presents in the Accident & Emergency Department with epileptic seizure. Head CT shows asymmetrical white matter oedema in the left parietal region with a mass effect. Post-contrast study shows a large, irregular, and peripheral enhancing lesion with a central area of low attenuation. What is the most likely diagnosis? [B5 Q41]
(a) Lymphoma
(b) Metastasis
(c) Glioblastoma multiforme
(d) Toxoplasmosis
(e) Cerebral abscess

A

Glioblastoma multiforme
These tumours are typically inhomogeneous on CT and MRI, showing irregular areas of peripheral enhancement. Tumour necrosis is a hallmark of glioblastoma multiforme.

How well did you know this?
1
Not at all
2
3
4
5
Perfectly
22
Q

A 35-year-old man attends the accident and emergency department complaining of episodic lower back pain radiating down the legs. History and clinical examination also suggest pelvic sphincter dysfunction. MRI shows a spinal cord mass located at the conus medullaris. The mass is isointense on T1 and hyperintense on T2. It demonstrates contrast enhancement. The most likely diagnosis is: [B2 Q25]
a. Astrocytoma
b. Intradural lipoma
c. Haemangioblastoma
d. Myxopapillary ependymoma
e. Ganglioglioma

A

Myxopapillary ependymoma
This variant of ependymoma is the most common neoplasm of the conus medullaris, originating from ependymal glia of the filum terminale. It typically presents around age 35 and is more common in men. T1-weighted imaging shows an isointense or occasionally hyperintense mass, hyperintense on T2, and almost always shows enhancement post-contrast. Intradural lipomas are hyperintense on T1 and do not enhance. Haemangioblastomas can also show high signal on T1 and are highly vascular. Gangliogliomas are more common at the cervical and thoracic levels.

How well did you know this?
1
Not at all
2
3
4
5
Perfectly
23
Q

A 9-year-old male presents to the paediatric A&E department with a history of increasing drowsiness over the last 24 hours. Neurological assessment reveals that there is absence of upward gaze. A CT brain is requested which reveals hydrocephalus, with marked dilatation of the lateral and third ventricles. The fourth ventricle is unremarkable and there is obliteration of the ambient cistern due to mass effect from a hyperdense mass noted posterior to the third ventricle. This mass has some central areas of calcification. There is no evidence of haemorrhage. An MRI is carried out following insertion of a shunt to decompress the ventricles. On sagittal sequences a lesion is located between the splenium of the corpus callosum and the tectal plate, which exerts mass effect. This lesion is of intermediate signal intensity on both T1WI and T2WI and displays avid contrast enhancement. Enhancing meningeal lesions are also noted in the spinal cord, indicating seeding. What is the most likely diagnosis? [B1 Q1]
A. Pituitary teratoma.
B. Meningioma.
C. Pineoblastoma.
D. Germinoma.
E. Pineal cyst.

A

Germinoma.
The lesion is described in the pineal region, so the differential of pineal tumours should be considered. These are differentiated between germ cell tumours and pineal cell tumours. Germinomas account for 40% of all pineal region masses and are much more frequent in males than females. They are also the most common germ cell tumour, with teratomas and choriocarcinoma having different imaging characteristics. The main differential in this case is between pineoblastoma and germinoma, as both occur in patients of this age group and both are hyperdense on CT. Imaging features described to help differentiate are the avid enhancement, which is more characteristic of germinomas, but can occur in either. Central calcification is seen commonly in germinomas, but is uncommon in pineoblastomas, and when it occurs is often peripheral, giving the impression of an ‘exploded’ pineal gland. Subarachnoid seeding is seen in both tumours and if present CSF sampling can yield a tissue diagnosis. Pineocytomas occur in an older age group and infrequently cause subarachnoid seeding.

How well did you know this?
1
Not at all
2
3
4
5
Perfectly
24
Q

A 20-year-old male presents with the inability to gaze upwards. CT brain shows moderate hydrocephalus and a rounded mass adjacent to the tectal plate. The mass demonstrates marked homogeneous enhancement and is not calcified. MRI confirms a well-circumscribed, relatively homogeneous mass that is isointense to grey matter on T2-weighted imaging. The mass is hyperintense on contrast-enhanced T1-weighted imaging. What is the most likely diagnosis? [B2 Q1]
a. Germinoma
b. Teratoma
c. Pineoblastoma
d. Pineocytoma
e. Benign pineal cyst

A

Germinoma
Germinomas are germ-cell tumours arising from primordial germ cells. They frequently occur in the midline, mostly in the pineal region but also in the suprasellar region. In men, 80% of pineal masses are germ-cell tumours, in contrast to 50% in women. They tend to occur in children or young adults (10–25 years old). Symptoms depend on the location, but the case describes Parinaud syndrome – paralysis of upward gaze due to compression of the mesencephalic tectum. Germinomas may also cause hydrocephalus by compression of the aqueduct of Sylvius, thus patients may present with signs and symptoms of raised intracranial pressure. Germinomas are a known cause of precocious puberty in children under the age of ten years. They are malignant tumours and may show CSF seeding, making cytological diagnosis possible with lumbar puncture. They are, however, very radiosensitive and show excellent survival rates. Pineal teratomas tend to be heterogeneous masses containing fat and calcifications. Pineoblastoma is a highly malignant tumour which is more common in children and usually has poor tumour margins.

How well did you know this?
1
Not at all
2
3
4
5
Perfectly
25
Q

A 30-year-old man presents with an orbital frontal headache and visual disturbance. CT head shows a large mass arising from the region of the spheno-occiput and extending into the pontine cistern and towards the hypothalamus. The mass contains amorphous calcification and is seen to cause bone destruction. There is no reactive bone sclerosis. On MRI the mass exhibits mixed heterogeneous signal and a soap bubble appearance. The solid components show marked contrast enhancement. What is the most likely diagnosis? [B2 Q36]
a. Meningioma
b. Metastasis
c. Chordoma
d. Plasmacytoma
e. Sphenoid sinus cyst

A

Chordoma
Chordomas originate from malignant transformation of notochordal cells. They are typically located in the sacrum (50%), clivus (35%) and vertebrae (15%). They may rarely be found in the mandible, maxilla and scapula. Spheno-occipital chordomas typically affect males and females in equal incidence and the average age ranges from 20 to 40 years. Most of them demonstrate bone destruction and amorphous calcification. The solid components show variable but often marked contrast enhancement and MRI may show a ‘soap-bubble’ appearance. Bone sclerosis is rare. The description given is that of a malignant process. Meningiomas typically affect older females and only rarely (<1%) arise from the clivus. Plasmacytomas tend to occur in an older age group, are more common in the thoracic/lumbar spine and are often osteolytic and grossly expansile lesions. Metastasis is a possibility, although from the description given – particularly the calcification and amount of tumour extension – chordoma remains the most likely diagnosis.

How well did you know this?
1
Not at all
2
3
4
5
Perfectly
26
Q

A 35-year-old male presents with ataxia and nystagmus. Blood tests reveal polycythaemia. CT head demonstrates a mass predominantly of CSF density in the posterior fossa. Subsequent MRI shows a largely cystic mass with an enhancing mural nodule. There is surrounding oedema but no calcification. The most likely diagnosis is: [B2 Q5]
a. Metastasis
b. Pilocytic astrocytoma
c. Haemangioblastoma
d. Choroid cyst
e. Ependymoma

A

Haemangioblastoma
Haemangioblastoma is the most common primary intra-axial, infratentorial tumour in adults. They are benign autosomal dominant tumours of vascular origin. Approximately 20% occur with von Hippel–Lindau disease. Other associations include phaeochromocytomas, syringomyelia and spinal cord haemangioblastomas. About 20% of tumours cause polycythaemia. Typical CT and MRI appearances are of a largely cystic mass with an enhancing mural nodule. Oedema may be absent or extensive, but calcification is rare. Prognosis is 85% post-surgical five-year survival rate. Infratentorial pilocytic astrocytomas may have very similar appearances to haemangioblastomas but some differences exist that can help differentiate the two. Pilocytic astrocytomas predominantly occur in children and young adults, are generally larger (>5 cm) than haemangioblastomas, may contain calcifications and are not associated with polycythaemia.

How well did you know this?
1
Not at all
2
3
4
5
Perfectly
27
Q

A 22-year-old man with von Hippel–Lindau syndrome presents with headaches, vomiting and ataxia. CT of the brain demonstrates an abnormality in the posterior fossa. What are the most likely findings? [B4 Q27]
a. cystic lesion with an enhancing mural nodule
b. multiple ring-enhancing lesions with surrounding oedema
c. gyriform cortical calcifications
d. multiple calcified subependymal nodules
e. hypodense, ill-defined mass with central necrosis and marked surrounding oedema

A

Cystic lesion with an enhancing mural nodule
Haemangioblastomas are the most recognized manifestation of von Hippel–Lindau syndrome. They usually occur in the cerebellum and may be multiple in up to 15% of cases. Patients also commonly develop renal cell carcinoma, and difficulty may occasionally arise in distinguishing metastases from multiple haemangioblastomas. Typical features are of a cystic mass in the hemisphere or vermis, with an enhancing mural nodule, though entirely solid lesions may also occur. Calcification is not a feature. Gyriform cortical calcifications are a feature of Sturge–Weber syndrome, and usually occur in the temporoparieto-occipital region. Calcified subependymal nodules (hamartomas) are a feature of tuberous sclerosis. A hypodense, ill-defined mass with central necrosis and marked surrounding oedema is a classic appearance of a glioblastoma multiforme.

How well did you know this?
1
Not at all
2
3
4
5
Perfectly
28
Q

A 40-year-old man presents with gradually worsening symptoms of ataxia, nausea and vomiting. CT of the head shows a 2 cm cystic lesion in the cerebellum with an enhancing mural nodule. What is the most likely diagnosis? [B5 Q10]
(a) Cerebellar haemangioblastoma
(b) Metastasis
(c) Cystic astrocytoma
(d) Arachnoid cyst
(e) Medulloblastoma

A

Cerebellar haemangioblastoma
These CT features are typical of this lesion.
Metastases usually have ring-like enhancement or homogenous enhancement rather than having a mural nodule. Cystic astrocytomas are usually > 5 cm, show calcifications, thick walled and have no enhancing mural nodule. An arachnoid cyst is a possibility if no enhancing nodule seen. Medulloblastoma is uncommon in adults and is usually a solid tumour with homogenous enhancement.

How well did you know this?
1
Not at all
2
3
4
5
Perfectly
29
Q

A 35-year-old man presents with headache and ataxia. CT of the brain shows a 6 cm cystic lesion in the right cerebellar hemisphere with a small enhancing nodule at the margin of the cyst. The most likely diagnosis is? [B5 Q25]
(a) Arachnoid cyst
(b) Necrotic metastasis
(c) Haemangioblastoma
(d) Juvenile pilocytic astrocytoma
(e) Cysticercosis

A

Haemangioblastoma
This is the most common posterior fossa tumour in adults after metastases. They are usually seen in the cerebellum and there is an association with von Hippel–Lindau disease. CT appearances are typically with a large hypodense cyst and an enhancing mural nodule. The cyst wall does not usually enhance. On MRI, flow voids may be seen representing draining vessels adjacent to the nodule.
Juvenile pilocytic astrocytoma is seen in young age and is not associated with feeding vessels. Metastases are usually multiple in older people.

How well did you know this?
1
Not at all
2
3
4
5
Perfectly
30
Q

A 40-year-old female is investigated for worsening headaches. CT shows a well-defined hyperdense globular lesion within the trigone of the left lateral ventricle. There is intense contrast enhancement. The most likely diagnosis is: [B2 Q2] {will repeat again in Meningioma Section}
a. Choroid cyst
b. Ependymoma
c. Colloid cyst
d. Meningioma
e. Neurocytoma

A

Meningioma
It is rare for meningiomas to occur intraventricularly (2–5% of all meningiomas) but they are the most common trigonal intraventricular mass in adulthood. They tend to occur in 40-year-old females.

How well did you know this?
1
Not at all
2
3
4
5
Perfectly
31
Q

When on call you are asked to perform a CT head scan for a 17-year-old male who presents with seizures. He is unable to provide a history. A look on the computer system shows that he has had previous regular abdominal ultrasounds and an echocardiogram as a child. Brain CT shows a hypodense, well-demarcated, rounded mass in the region of the foramen of Monro. It is partially calcified, and it demonstrates uniform enhancement. What is the most likely diagnosis? [B2 Q4] {will repeat again in periventricular tumour section}
a. Colloid cyst
b. Giant cell astrocytoma
c. Metastasis
d. Lymphoma
e. Haemangioblastoma

A

Giant cell astrocytoma
The main differential for a mass at the foramen of Monro is between a colloid cyst and a subependymal giant cell astrocytoma. The latter is associated with tuberous sclerosis (TS). Renal involvement is also relatively common in TS and patients regularly have surveillance renal ultrasounds. The echocardiogram was performed to assess for cardiomyopathy/rhabdomyoma.

How well did you know this?
1
Not at all
2
3
4
5
Perfectly
32
Q

A 30-year-old female complains of increasing headaches, episodic vomiting, and drowsiness. Fundoscopy reveals papilledema. Non-contrast CT of the head demonstrates hydrocephalus and a globular lesion within the lateral ventricle. There are several small internal foci of calcification. MR shows the mass to be attached to the septum pallucidum. It is isointense to grey matter on T1 and T2. It densely enhances after intravenous gadolinium. What is the most likely diagnosis? [B2 Q23]
a. Ependymoma
b. Subependymoma
c. Central neurocytoma
d. Heterotopic grey matter
e. Meningioma

A

Central neurocytoma
Central neurocytoma is an intraventricular WHO grade II neuroepithelial tumour with neuronal differentiation. This rare neoplasm tends to occur between the ages of 20–40 years. Patients typically present with symptoms and signs of hydrocephalus.
Imaging typically demonstrates a globular lesion attached to the septum pellucidum. Calcification is considered characteristic; however, it may be absent in approximately half the cases. On MRI, the lesions are usually isointense to grey matter and show dense contrast enhancement. It is extremely uncommon to see peri-tumoural oedema.

How well did you know this?
1
Not at all
2
3
4
5
Perfectly
33
Q

Which of the following is an extra-axial posterior fossa tumour in adults? [B3 Q10]
A. Choroid plexus papilloma
B. Metastasis
C. Haemangioblastoma
D. Lymphoma
E. Glioma

A

Choroid plexus papilloma
Other extra-axial posterior fossa masses include acoustic neuroma, meningioma, chordoma and epidermoid.

How well did you know this?
1
Not at all
2
3
4
5
Perfectly
34
Q

Which is the most prevalent intraventricular tumour? [B3 Q11]
A. Astrocytoma
B. Meningioma
C. Choroid plexus papilloma
D. Dermoid
E. Subependymoma

A

A
Ependymomas followed by astrocytoma are the most common.

How well did you know this?
1
Not at all
2
3
4
5
Perfectly
35
Q

Which of the following best describes imaging changes in a colloid cyst? [B3 Q18]
A. Typically hypodense on non-contrast CT
B. Appears high SI on T1
C. Appears low SI on T2
D. Commonly widens septum pellucidi
E. Most commonly causes symmetrical enlargement of lateral ventricles

A

B
Protein content/paramagnetic effect of Magnesium Mg2+/ Calcium Ca2+/ Iron Fe, in a cyst cause increased T1 and T2 SI. Colloid cysts appear iso/hyperdense on NCCT. They can occasionally widen septum pellucidum and cause asymmetrical enlargement of the lateral ventricles.

How well did you know this?
1
Not at all
2
3
4
5
Perfectly
36
Q

A 27-year-old, previously well man gives a history of positional headaches. He undergoes unenhanced CT of the brain, which demonstrates a well-defined, hyperdense, spherical lesion in the anterosuperior portion of the third ventricle, with asymmetrical enlargement of the lateral ventricles. What is the most likely diagnosis? [B4 Q58]
a. meningioma
b. colloid cyst
c. ependymoma
d. subependymal giant cell astrocytoma
e. choroid plexus papilloma

A

Colloid Cyst
Colloid cysts arise from the inferior aspect of the septum pellucidum and protrude into the anterior aspect of the third ventricle, where they may cause positional headaches and hydrocephalus due to transient obstruction at the foramen of Monro. They usually contain mucinous fluid, desquamated cells and proteinaceous debris, making them hyperdense on CT. On MR scan, the high protein content, as well as the paramagnetic effect of magnesium, copper and iron in the cyst, results in high signal intensity on T1W and T2W sequences in 60% of cases. Meningiomas and ependymomas may also appear hyperdense, but location within the third ventricle is uncommon for both. Subependymal giant cell astrocytoma is a benign tumour occurring in the region of the foramen of Monro, which may cause obstruction, but it is usually hypodense and is nearly always seen in association with tuberous sclerosis. Choroid plexus papilloma predominantly occurs in children under 5 years of age, and location in the third ventricle is unusual.

How well did you know this?
1
Not at all
2
3
4
5
Perfectly
37
Q

A 21-year-old man presents with acute headache. He undergoes CT of the brain, which demonstrates a well-circumscribed, lobulated, partially cystic, calcified mass in the frontal horn of the left lateral ventricle, attached to the septum pellucidum. There is acute blood seen layering in the left lateral ventricle and mild hydrocephalus. What is the most likely diagnosis? [B4 Q99]
a. choroid plexus papilloma
b. colloid cyst
c. intraventricular oligodendroglioma
d. central neurocytoma
e. subependymoma

A

Central neurocytoma
Central neurocytomas (also known as intraventricular neurocytomas) are benign tumours of the lateral and third ventricles usually presenting in adults aged 20–40 years. They frequently calcify (69%) and contain cystic spaces. Attachment to the septum pellucidum is a characteristic feature. Lesions appear isointense to grey matter on all MR sequences and show mild-to-moderate contrast enhancement. Central neurocytomas were previously frequently mistaken for intraventricular oligodendrogliomas, which have very similar imaging features but are actually quite rare. In addition, neurocytomas undergo haemorrhage into the tumour or ventricle more frequently, helping to distinguish the two. Colloid cysts arise within the third ventricle, and rarely calcify. Subependymomas may arise in the lateral ventricles with an attachment to the septum pellucidum, and cyst formation and calcification may be seen in large tumours. However, most occur in patients over 40 years of age. Choroid plexus papillomas generally occur in children under 5 years of age.

How well did you know this?
1
Not at all
2
3
4
5
Perfectly
38
Q

A 37-year-old man presented with a history of intermittent headaches. Unenhanced CT scan of the head demonstrates a 1 cm, dense, round lesion in the region of the interventricular foramen. Mild hydrocephalus was also seen. On MRI, the lesion returns high signal on T1 and T2 sequences. The most likely diagnosis of this lesion is? [B5 Q8]
(a) Meningioma
(b) Ependymoma of the 3rd ventricle
(c) Colloid cyst
(d) Dermoid cyst
(e) Arachnoid cyst

A

Colloid cyst
Colloid cysts are seen in the region of the interventricular foramen and cause positional and intermittent obstruction, leading to hydrocephalus and headaches. The lesion is dense on CT and high signal on T1 and T2 sequences as they commonly contain large protein molecules and the paramagnetic effect of iron and copper in the cyst.
Dermoid contains fat whereas arachnoid cysts show CSF features on imaging.
Meningioma is usually low signal on T1 and high signal on T2.

How well did you know this?
1
Not at all
2
3
4
5
Perfectly
39
Q

A 30-year-old male patient is referred from ENT for an MRI with a history of tinnitus and slight hearing loss on the left side. A lesion is noted in the left cerebellopontine angle. This extends along the nerve and expands the internal auditory canal. A separate nerve is noted to enter the anterior superior portion of the internal auditory canal. The lesion is isointense to the pons on all pulse sequences. The lesion makes an acute angle with the petrous bone. There is no evidence of a dural tail following enhancement. What is the most likely cause? [B1 Q15]
A. Meningioma.
B. Facial nerve schwannoma.
C. Vestibular nerve schwannoma.
D. Epidermoid.
E. Arachnoid cyst.

A

C. Vestibular nerve schwannoma
This accounts for 75% of CPA tumors. The acute angle with the petrous bone and expansion of the internal auditory canal are characteristic features. The absence of a dural tail further supports this diagnosis. Facial nerve schwannomas are rare in this location and typically cannot be separated from the mass on imaging.

How well did you know this?
1
Not at all
2
3
4
5
Perfectly
40
Q

A 42-year-old woman has a CT scan of the brain performed for the investigation of headache. A lesion with density like cerebrospinal fluid is noted at the right cerebellopontine angle. A subsequent MRI scan shows that the lesion is well-defined, of decreased signal on T1WI and increased signal on T2WI. You think the lesion is most likely an epidermoid cyst but wish to exclude an arachnoid cyst. Which of the following further MRI sequences will be most helpful in achieving this aim? [B1 Q25]
A. Proton density.
B. STIR.
C. T1 with fat suppression.
D. FLAIR.
E. T1 post gadolinium.

A

D. FLAIR
On FLAIR sequences, epidermoid cysts show mixed iso- to hyperintense signal with poor demarcation, while arachnoid cysts have their signal suppressed, resembling CSF. Diffusion-weighted imaging can also help distinguish them, as epidermoid cysts show very high signal due to restricted diffusion.

How well did you know this?
1
Not at all
2
3
4
5
Perfectly
41
Q

A 54-year-old man presents with hearing loss in the left ear, which is of the sensori-neural type. He undergoes an MRI scan of the internal auditory meati and subsequently a full MRI scan of the brain. A large extra-axial mass lesion is identified at the left cerebellopontine angle. Which of the following features on MR imaging will be most helpful in indicating that this is probably a large vestibular schwannoma, rather than a meningioma? [B1 Q74]
A. Enhancing dura adjacent to the mass.
B. Erosion of the adjacent porus acousticus.
C. Tumour within the adjacent internal auditory meatus.
D. Intense enhancement within the mass.
E. Hyperostosis in the adjacent petrous temporal ridge.

A

B. Erosion of the adjacent porus acousticus
Vestibular schwannomas typically erode the posterior edge of the porus acousticus as they grow, whereas meningiomas usually do not cause such erosion. This distinction is crucial in identifying the nature of the lesion.

How well did you know this?
1
Not at all
2
3
4
5
Perfectly
42
Q

A 13-year-old boy is investigated for chronic headache and visual disturbance. CT shows a well-defined mass in the left middle cranial fossa. It is isodense to CSF. There are no calcifications, no surrounding edema, and no contrast enhancement. There is erosion of the underlying calvarium. You suspect this is an arachnoid cyst, but your consultant suggests the possibility of an epidermoid cyst. What MR imaging sequence would best differentiate the two? [B2 Q26]
a. Diffusion-weighted MR imaging
b. Gadolinium-enhanced T1-weighted imaging
c. Proton density imaging
d. MR spectroscopy
e. Perfusion-weighted MR imaging

A

A. Diffusion-weighted MR imaging
Epidermoid cysts appear bright on diffusion-weighted imaging due to restricted diffusion, while arachnoid cysts appear dark. This sequence is particularly useful for distinguishing between the two cyst types when they are isodense to CSF on CT and standard MRI.

How well did you know this?
1
Not at all
2
3
4
5
Perfectly
43
Q

Which of the following favours an arachnoid rather than an epidermoid cyst? [B3 Q15]
A. Hyperdense to CSF on CT
B. Smooth margin
C. Encased vessels
D. Deviates from CSF on proton density
E. Restricted diffusion

A

B. Smooth margin
Smooth margins are characteristic of arachnoid cysts, while epidermoid cysts tend to have irregular margins and can encase vessels.

How well did you know this?
1
Not at all
2
3
4
5
Perfectly
44
Q

Which is the most common location for epidermoid in the Central Nervous System (CNS)? [B3 Q19]
A. Cerebellar pontine angle (CPA)
B. Suprasellar region
C. Perimesencephalic cisterns
D. Ventricles
E. Skull vault

A

A. Cerebellar pontine angle (CPA)
Epidermoid cysts are most commonly found in the CPA, accounting for 40% of cases and 5% of CPA tumors.

How well did you know this?
1
Not at all
2
3
4
5
Perfectly
45
Q

A 2cm mass is seen on MR at the left CPA with uniform enhancement and high SI on T2 and dural tail. What is the diagnosis? [B3 Q33]
A. Meningioma
B. Vestibular schwannoma
C. Epidermoid
D. Metastatic deposit
E. Glomus tumour

A

A. Meningioma
The presence of a dural tail and broad-based attachment to the petrous bone are characteristic of meningiomas. Uniform enhancement and less bright T2 signal further support this diagnosis

How well did you know this?
1
Not at all
2
3
4
5
Perfectly
46
Q

35-year-old woman presents with progressive deafness and tinnitus in the left ear. She undergoes MRI, which demonstrates a 2 cm mass at the left cerebellopontine angle. Which of the following features would favour a diagnosis of meningioma rather than vestibular schwannoma? [B4 Q18]
a. acute angle with the petrous bone
b. hyperintensity on T2W images
c. expansion of the internal auditory canal
d. presence of a dural tail
e. internal cystic degeneration and haemorrhage

A

Presence of a dural tail
The most common causes of a cerebellopontine angle mass are vestibular schwannoma (also called acoustic neuroma) (75%), meningioma (10%) and epidermoid cyst (5%). Features suggestive of a meningioma include a dural tail (thickening of enhancing adjacent dura resembling a tail extending from the mass), adjacent hyperostosis and an obtuse angle with the petrous bone (vestibular schwannomas make an acute angle). Distinguishing features of schwannomas include extension into the internal auditory canal, causing expansion of the canal and flaring of the porus acousticus (bony opening of the internal auditory canal). Meningiomas may show a small tongue of extension into the canal but usually no expansion. Schwannomas undergo cystic degeneration and haemorrhage more commonly than meningiomas (particularly larger lesions), and may show very high signal on T2W images, which is unusual for a meningioma.

How well did you know this?
1
Not at all
2
3
4
5
Perfectly
47
Q

A 23-year-old man presents with acute headache. Unenhanced CT of the brain demonstrates a heterogeneous mass at the inferior cerebellar vermis. It is predominantly of fat attenuation with areas of calcification and does not enhance following administration of intravenous contrast. Multiple droplets of fat attenuation are noted throughout the subarachnoid space. What is the most likely diagnosis? [B4 Q72]
a. lipoma
b. arachnoid cyst
c. dermoid
d. epidermoid
e. teratoma

A

Dermoid
Epidermoids and dermoids are congenital lesions resulting from inclusion of ectodermal elements during closure of the neural tube. Both have a squamous epithelial lining and produce keratin, but dermoids contain both ectodermal and mesodermal elements (hair follicles, sweat and sebaceous glands), while epidermoids contain only ectodermal elements. Epidermoids are lobulated masses, usually located off the midline, which compress adjacent structures such as cranial nerves, and tend to have appearances on CT and MR scan following that of CSF. Dermoids are usually midline in location and cause symptoms by obstruction of CSF pathways or by rupture and leakage of fat contents, causing chemical meningitis. They have appearances on CT and MR scan following that of fat and are often heterogeneous with areas of calcification and other soft-tissue components. This heterogeneity helps to distinguish a dermoid from a lipoma. Arachnoid cysts are of CSF density, but may be distinguished from epidermoid on DWI, where they do not show evidence of water restriction. Teratomas are composed of ectodermal, mesodermal and endodermal elements, and also appear heterogeneous with areas of fat, calcification and cystic components, but these lesions occur most commonly in the pineal and suprasellar regions.

How well did you know this?
1
Not at all
2
3
4
5
Perfectly
48
Q

A 40-year-old teacher presents with a history of hearing loss in the left ear. Gadolinium-enhanced MRI shows a non-enhancing lesion in the left cerebellopontine angle (CPA). The lesion is isointense to CSF on T1 and T2 sequences. On FLAIR imaging, the lesion shows incomplete attenuation of fluid signal and on diffusion-weighted imaging it returns a bright signal. The most likely diagnosis is? [B5 Q11]
(a) Arachnoid cyst in the left CPA
(b) Schwannoma in the left CPA
(c) Epidermoid cyst in the left CPA
(d) Lipoma in the left CPA
(e) Cystic meningioma in the left CPA

A

(c) Epidermoid cyst in the left CPA
This resembles CSF on non-enhanced CT. On MRI, the lesion is isointense, or slightly hyperintense to CSF. Incomplete attenuation on FLAIR and high signal on diffusion (suggesting restricted diffusion) concludes the diagnosis. Non enhancement is the rule.

How well did you know this?
1
Not at all
2
3
4
5
Perfectly
49
Q

A 50-year-old woman complains of tinnitus, headaches, and hearing loss. MRI shows a heterogenous, well-defined mass in the left cerebellopontine angle producing a local mass effect. The lesion returns low signal on T1, heterogenous high signal on T2 and heterogeneously enhances with gadolinium. The most likely diagnosis is? [B5 Q20]
(a) Schwannoma
(b) Meningioma
(c) Epidermoid
(d) Arachnoid cyst
(e) Metastasis

A

(a) Schwannoma
Vestibular schwannomas are the most common cerebellopontine angle tumours with typical imaging features as given above. These are typically benign, slow growing tumours from Schwann cells which envelop and myelinate cranial, spinal and peripheral nerves. In the skull, they most commonly arise in the cerebellopontine angle, from the vestibular portion of the 8th cranial nerve. The main differential diagnosis is meningioma, which can also grow from the 8th cranial nerve. A schwannoma is hyperintense on T2 while a meningioma is isohypointense. Also, a meningioma forms an obtuse angle with the petrous bone while a schwannoma forms an acute angle. Finally, a meningioma may have a dural tail, which is absent in a schwannoma.

How well did you know this?
1
Not at all
2
3
4
5
Perfectly
50
Q

A 18-year-old man was admitted after a road traffic accident. CT of the head shows an incidental lesion in the right cerebello-pontine angle. MRI shows a 4 cm homogenous lesion in the right cerebellopontine angle which is high signal on T2, intermediate on T1 and without restricted diffusion. No gadolinium enhancement seen. What is the most likely diagnosis? [B5 Q31]
(a) Arachnoid cyst
(b) Acoustic neuroma
(c) Epidermoid cyst
(d) Lipoma
(e) Necrotic metastasis

A

(a) Arachnoid cyst
An arachnoid cyst returns signal characteristics of cerebrospinal fluid. High on T2, intermediate on T1 and no restriction on diffusion-weighted images.

How well did you know this?
1
Not at all
2
3
4
5
Perfectly
51
Q

A 40-year-old man presents with unilateral sensorineural hearing loss. MRI shows a well-defined mass in the left cerebellopontine angle. The lesion returns high signal on T1, T2 and FLAIR sequences. On fat saturated T1 with contrast, the lesion returns low signal and no contrast enhancement is seen. What is the most likely diagnosis? [B5 Q37]
(a) Acoustic schwannoma with haemorrhage
(b) Lipoma
(c) Epidermoid cyst
(d) Giant aneurysm
(e) Arachnoid cyst

A

Cerebellopontine angle lipoma
The lesion has characteristics of fat on all sequences (high signal on T1 and T2 with loss of signal on fat-suppression imaging).

How well did you know this?
1
Not at all
2
3
4
5
Perfectly
52
Q

A 12-year-old male with a history of gelastic seizures is referred for MRI of the brain. Which of the following statements regarding hamartomas of the tuber cinereum is true? [B1 Q41]
A. No change in size, shape, or signal intensity on follow-up MRI.
B. Demonstrate homogenous contrast enhancement.
C. Calcification is a common finding.
D. Hyperintense on T1WI and T2WI, and hypointense on fat suppressed sequences.
E. Located in the sella turcica.

A

No change in size, shape, or signal intensity on follow-up MRI.
Hypothalamic hamartomas are developmental malformations located in the tuber cinereum of the hypothalamus. The typical patient is male, in the first or second decade of life, presenting with precocious puberty or gelastic seizures. On MRI, they appear as well-defined pedunculated or sessile lesions that are iso/mildly hypointense on T1WI and iso/hyperintense on T2WI, with no contrast enhancement or calcification. Lack of interval change strongly supports the diagnosis.

How well did you know this?
1
Not at all
2
3
4
5
Perfectly
53
Q

A 45-year-old female undergoes an MRI of the pituitary that demonstrates a kidney-shaped lesion located centrally in the pituitary fossa in the axial plane. It is hyperintense on T1WI and hypointense on T2WI. There is no enhancement following gadolinium administration and no fluid–fluid level. What is the diagnosis? [B1 Q53]
A. Rathke cleft cyst.
B. Craniopharyngioma.
C. Cholesterol granuloma.
D. Haemorrhagic adenoma.
E. Lipoma.

A

Rathke cleft cyst.
These are benign cystic lesions of the pituitary fossa derived from the Rathke pouch. They are usually asymptomatic. Rathke cleft cysts are in the midline between the anterior and posterior pituitary lobes and have a characteristic kidney shape on axial images. They are homogenously hyperintense on T1WI, due to high protein concentration, and hypointense on T2WI, due to low intracystic water content. They do not enhance following contrast administration. Absence of a fluid–fluid level is helpful in differentiating Rathke cleft cyst from hemorrhagic adenoma. Acute haemorrhage has heterogenous signal on T2WI and may demonstrate thin peripheral enhancement on T1WI. Craniopharyngiomas have variable solid, cystic, and calcified components. They demonstrate heterogenous enhancement following contrast. They may be intrasellar or suprasellar. A pseudo-fluid–fluid level may occasionally be seen in craniopharyngioma. Lipoma and cholesterol granuloma are hyperintense on both T1WI and T2WI.

How well did you know this?
1
Not at all
2
3
4
5
Perfectly
54
Q

A 32-year-old female is referred to neurology complaining of visual disturbance and headache. She is 4 months postpartum. On examination a bitemporal hemianopia is noted. Hormonal testing reveals hypoadrenalism and hypothyroidism. A dedicated MRI of her pituitary gland is requested. Which of the following features is suggestive of autoimmune hypophysitis over pituitary adenoma? [B1 Q75]
A. Asymmetric pituitary enlargement.
B. Heterogenous gadolinium enhancement.
C. Loss of the posterior pituitary bright spot.
D. Sphenoid sinus mucosal thickening.
E. Age >30.

A

Loss of the posterior pituitary bright spot.
Autoimmune hypophysitis (AH) and non-secreting pituitary adenomas can only be differentiated with certainty on histology. As a result, approximately 40% of patients with AH are misdiagnosed as having pituitary macroadenoma and undergo unnecessary surgery. Hormone production is compromised in both conditions, although a history of infertility is common with adenomas, whereas patients with AH typically achieve spontaneous pregnancy. To develop a scoring system to differentiate the two conditions, a recent study found that features significantly associated with AH over adenoma were age <30, relation to pregnancy, homogenous gadolinium enhancement, loss of the posterior pituitary bright spot, and enlarged stalk. Features consistent with adenoma were asymmetrically enlarged pituitary, size >6 cm³, and associated sinus mucosal thickening. The normal posterior pituitary gland is bright on T1WI due to the rich content of vasopressin neurosecretory granules. This is frequently lost in AH due to direct autoimmune involvement of the neurohypophysis, whereas it is conserved in most adenomas, even when displaced by large tumour size.

How well did you know this?
1
Not at all
2
3
4
5
Perfectly
55
Q

A 40-year-old female presents with bitemporal hemianopia. CT brain shows a large, slightly hyperdense suprasellar lesion. The mass contains several lucent foci and there is bone erosion of the sella floor. There is enhancement post-contrast. T1-weighted MR imaging shows a predominantly isointense mass causing sella expansion and compression of the optic chiasm. The mass contains foci of low and high signal intensity. What is the most likely diagnosis? [B2 Q3]
a. Craniopharyngioma
b. Meningioma
c. Rathke’s cleft cyst
d. Giant internal carotid aneurysm
e. Pituitary adenoma

A

Pituitary adenoma
Pituitary adenomas are divided into microadenomas (<1 cm) and macroadenomas (>1 cm). Macroadenomas may present with endocrine dysfunction but are generally less active than microadenomas. Thus, macroadenomas often present with symptoms of mass effect on the optic chiasm, or if there is lateral extension into the cavernous sinuses, patients may present with other local cranial nerve palsies (III, IV, VI). The differential diagnosis of a suprasellar mass includes (‘SATCHMO’): Suprasellar extension of pituitary adenoma/sarcoid; Aneurysm/arachnoid cyst; TB/teratoma (other germ-cell tumours); Craniopharyngioma; Hypothalamic glioma or hamartoma; Meningioma/metastases (especially breast); and Optic/chiasmatic glioma. In this case, the sellar is widened and the floor is eroded suggesting the mass arises from the pituitary itself. Low-density/low-intensity regions on CT/T1 MRI correspond to necrotic areas and high-signal foci on T1 MRI (found relatively frequently) represent areas of recent haemorrhage.

How well did you know this?
1
Not at all
2
3
4
5
Perfectly
56
Q

Which of the following features favour Rathke’s cleft cyst rather than craniopharyngioma? [B3 Q1]
A. Absence of calcification
B. Cystic element on MR
C. Involvement of suprasellar and sellar regions
D. Enhancement of the wall
E. High signal intensity on T1

A

Absence of calcification
Rathkes cleft cysts do not calcify. They affect women to men in a 2:1 ratio and adults from 40-60 years of age. They cause variable MR appearances depending on protein content of cyst. They can rarely show enhancement.

How well did you know this?
1
Not at all
2
3
4
5
Perfectly
57
Q

What are the typical appearances of a pituitary microadenoma on early, contrast enhanced T1W MR images? [B4 Q13]
a. focus of hypo-intensity within normal enhancing gland
b. focus of enhancement within normal, non-enhancing gland
c. lesion and normal gland enhance similarly
d. hyper-enhancing focus within normal, mildly enhancing gland
e. not usually visualized on this sequence

A

Focus of hypo-intensity within normal enhancing gland
Pituitary microadenomas are typically hypointense compared with the normal gland on unenhanced T1W images, and the diagnosis can usually be made without contrast. Following contrast, the microadenoma does not initially enhance, and maximal contrast between enhancing normal gland and pituitary tumour is seen on dynamic images obtained within the first minute. Contrast enhancement may therefore be useful in identifying lesions that are not obviously hypointense on the unenhanced images. However, contrast enhancement of the tumour relative to the normal gland may be seen on delayed (>20 min) images.

How well did you know this?
1
Not at all
2
3
4
5
Perfectly
58
Q

A 46-year-old man presents with headaches and visual disturbance and is found to have bitemporal hemianopia on visual field testing. CT of the brain demonstrates a multilobulated, heterogeneous, suprasellar mass containing cystic areas and rim-like calcification. Enhancement of the solid component is observed following intravenous contrast. What is the most likely diagnosis? [B4 Q57]
a. epidermoid
b. craniopharyngioma
c. pituitary macroadenoma
d. meningioma
e. Rathke’s cleft cyst

A

Craniopharyngioma
Craniopharyngiomas are the most common suprasellar mass, predominantly occurring in the first and second decades, but with a second peak in the fifth decade. Presenting symptoms include headache secondary to hydrocephalus, bitemporal hemianopia (compression of the optic chiasm) and diabetes insipidus (compression of the pituitary gland). Typical imaging features are calcifiication, cyst formation and enhancement that may be solid or nodular. Meningiomas may arise in the suprasellar region and commonly demonstrate calcifiication but are generally not cystic. Epidermoids may occasionally demonstrate rim calcifiication but rarely enhance following intravenous contrast. Pituitary macroadenomas may undergo haemorrhage, resulting in heterogeneity that can cause confusion with craniopharyngioma, but this typically occurs in adolescence. Calcifiication in macroadenomas is infrequent. Rathke’s cleft cysts are thin-walled, benign cysts arising in the anterior sellar or suprasellar region. They show no contrast enhancement and rarely calcify.

How well did you know this?
1
Not at all
2
3
4
5
Perfectly
59
Q

A 1-year-old child presents with precocious puberty and MRI shows a suprasellar mass attached to the mamillary bodies with a thin stalk. The most likely cause is? [B5 Q4]
(a) Hypothalamic hamartoma
(b) Craniopharyngioma
(c) Hypothalamic glioma
(d) Kallman syndrome
(e) Pituitary adenoma

A

Hypothalamic hamartoma
Also called hamartoma of the tuber cinereum, this is seen in children less than 2 years of age. Precocious puberty is due to luteinising releasing hormone secretion. Craniopharyngioma presents with growth failure and visual field defects. Kallmann syndrome presents with hypogonadism in later age. Pituitary adenomas are seen in girls (9–13 years of age) and are usually prolactin or adrenocorticotropic hormone secreting lesions.

How well did you know this?
1
Not at all
2
3
4
5
Perfectly
60
Q

A 21-year-old boy with neurofibromatosis type 1 complains of visual difficulties. MRI shows abnormal enlargement of the optic chiasm and intense and homogenous enhancement with gadolinium. The abnormality extends into the left optic tract. What is the most likely diagnosis? [B5 Q33] {will repeat again in neuro-cutaneous syndrome section}
(a) Craniopharyngioma
(b) Lymphoma
(c) Neurosarcoidosis
(d) Chiasmal glioma
(e) Tuberculosis

A

Chiasmal glioma
Chiasmal glioma is associated with NF1 in 15-25% cases. Craniopharyngioma tends to displace the chiasm rather than enlarge it. It is often cystic and may show calcification. Lymphoma is more likely to involve peripheral nerves. Tuberculosis may involve the chiasm but in the setting of basal meningitis, leptomeningeal enhancement and multiple cranial neuropathies.

How well did you know this?
1
Not at all
2
3
4
5
Perfectly
61
Q

A 9-year-old male presents to the paediatric A&E department with a history of increasing drowsiness over the last 24 hours. Neurological assessment reveals that there is absence of upward gaze. A CT brain is requested which reveals hydrocephalus, with marked dilatation of the lateral and third ventricles. The fourth ventricle is unremarkable and there is obliteration of the ambient cistern due to mass effect from a hyperdense mass noted posterior to the third ventricle. This mass has some central areas of calcification. There is no evidence of haemorrhage. An MRI is carried out following insertion of a shunt to decompress the ventricles. On sagittal sequences, a lesion is located between the splenium of the corpus callosum and the tectal plate, which exerts mass effect. This lesion is of intermediate signal intensity on both T1WI and T2WI and displays avid contrast enhancement. Enhancing meningeal lesions are also noted in the spinal cord, indicating seeding. What is the most likely diagnosis? [B1 Q1]
A. Pituitary teratoma.
B. Meningioma.
C. Pineoblastoma.
D. Germinoma.
E. Pineal cyst.

A

Germinoma.
The lesion is described in the pineal region, so the differential of pineal tumours should be considered. These are differentiated between germ cell tumours and pineal cell tumours. Germinomas account for 40% of all pineal region masses and are much more frequent in males than females. They are also the most common germ cell tumour, with teratomas and choriocarcinoma having different imaging characteristics. The main differential in this case is between pineoblastoma and germinoma, as both occur in patients of this age group and both are hyperdense on CT. Imaging features described to help differentiate are the avid enhancement, which is more characteristic of germinomas, but can occur in either. Central calcification is seen commonly in germinomas, but is uncommon in pineoblastomas, and when it occurs is often peripheral, giving the impression of an ‘exploded’ pineal gland. Subarachnoid seeding is seen in both tumours and if present CSF sampling can yield a tissue diagnosis. Pineocytomas occur in an older age group and infrequently cause subarachnoid seeding.

How well did you know this?
1
Not at all
2
3
4
5
Perfectly
62
Q

A 20-year-old male presents with the inability to gaze upwards. CT brain shows moderate hydrocephalus and a rounded mass adjacent to the tectal plate. The mass demonstrates marked homogeneous enhancement and is not calcified. MRI confirms a well-circumscribed, relatively homogeneous mass that is isointense to grey matter on T2-weighted imaging. The mass is hyperintense on contrast-enhanced T1-weighted imaging. What is the most likely diagnosis? [B2 Q1]
a. Germinoma
b. Teratoma
c. Pineoblastoma
d. Pineocytoma
e. Benign pineal cyst

A

Germinoma.
Germinomas are germ-cell tumours arising from primordial germ cells. They frequently occur in the midline, mostly in the pineal region but also in the suprasellar region. In men, 80% of pineal masses are germ-cell tumours, in contrast to 50% in women. They tend to occur in children or young adults (10–25 years old). Symptoms depend on the location, but the case describes Parinaud syndrome – paralysis of upward gaze due to compression of the mesencephalic tectum. Germinomas may also cause hydrocephalus by compression of the aqueduct of Sylvius, thus patients may present with signs and symptoms of raised intracranial pressure. Germinomas are a known cause of precocious puberty in children under the age of ten years. They are malignant tumours and may show CSF seeding, making cytological diagnosis possible with lumbar puncture. They are, however, very radiosensitive and show excellent survival rates. Pineal teratomas tend to be heterogeneous masses containing fat and calcifications. Pineoblastoma is a highly malignant tumour which is more common in children and usually has poor tumour margins.

How well did you know this?
1
Not at all
2
3
4
5
Perfectly
63
Q

A 9-year-old boy presents with precocious puberty and headache. CT of the brain shows an enhancing mass in the pineal region with calcifications. There is moderate hydrocephalus with a dilated lateral and 3rd ventricle. The most likely diagnosis is? [B5 Q21]
(a) Pineal germinoma
(b) Glioma
(c) Medulloblastoma
(d) Meningioma
(e) Metastases

A

Pineal germinoma (also called pinealoma).
This is the most common pineal tumour and is associated with precocious puberty in children less than 10 years old. The finding of pineal calcification before 10 years with a pineal mass enhancing with contrast is usually diagnostic. Hydrocephalus is secondary to compression of the cerebral aqueduct. Medulloblastoma is a tumour usually in the posterior fossa and presenting in childhood. Suprasellar meningioma does not arise from the pituitary fossa.

How well did you know this?
1
Not at all
2
3
4
5
Perfectly
64
Q

A 49-year-old woman with AIDS presents with increasing headache. T1-weighted MR imaging demonstrates a hypointense lesion in the periventricular white matter of the left parietal lobe. The lesion is hypointense on FLAIR sequencing and is seen to cross the splenium of the corpus callosum. There is peripheral enhancement post-contrast injection. The most likely diagnosis is: [B2 Q44]
a. Toxoplasmosis
b. Progressive multifocal leukoencephalopathy
c. Primary CNS lymphoma
d. Cryptococcosis
e. Tuberculosis

A

Primary CNS lymphoma (PCNSL).
PCNSL is the second most common cause of a CNS mass in patients with AIDS (behind toxoplasmosis). Typical features include periventricular location with subependymal spread and crossing of the corpus callosum. Non-contrast CT may show a hyperdense lesion due to dense cellularity (for this reason it may also be hypointense on T2/FLAIR). There is often a paucity of oedema and frequent ring enhancement due to central necrosis (note this is in contrast with the solid homogeneous enhancement seen with lymphoma in the immunocompetent patient). Cryptococcosis is the most common cause of fungal infection in AIDS patients. CT is frequently normal, and MRI shows low T1 and high T2 signal intensities without enhancement in the lenticulostriate region. Progressive multifocal leukoencephalopathy typically shows bilateral white matter lesions in the periventricular region, centrum semi-ovale or subcortical white matter, which are hypointense on T1 and hyperintense on T2/FLAIR. There is typically no oedema nor mass effect and no contrast enhancement.

How well did you know this?
1
Not at all
2
3
4
5
Perfectly
65
Q

A 35-year-old man presents with persistent headaches. CT of the head shows a 3 cm homogenous and hyperdense mass with homogenous contrast enhancement. The lesion resolved with radiotherapy. What is the most likely diagnosis? [B5 Q42]
(a) Glioma
(b) Metastases
(c) Lymphoma
(d) Sarcoidosis
(e) Oligodendroglioma

A

Lymphoma
This is seen not only in immunocompromised but also immunocompetent patients. On CT, the lesion is usually hyperdense showing homogenous enhancement. Resolution with steroids and/or radiotherapy is a characteristic finding of cerebral lymphoma.

How well did you know this?
1
Not at all
2
3
4
5
Perfectly
66
Q

A 50-year-old male patient is referred from A&E for a CT brain. He has a history of headache for 2 months and increasing clumsiness. The CT shows a lesion abutting the sphenoid in the anterior cranial fossa. It measures 4 cm in size and demonstrates heterogeneous enhancement. There is no evidence of calcification. The lesion is displacing and effacing the third ventricle, causing mild hydrocephalus in the lateral ventricles. An MRI is performed. The lesion is isointense on T1WI and T2WI and demonstrates an enhancing dural tail and broad dural base. Again, the enhancement pattern is heterogeneous. Several flow voids are noted in the lesion. There is little peri-tumoural oedema noted. An MR angiogram is performed, which indicates dual supply to the lesion from the internal and external carotid artery. What lesion typically demonstrates these imaging findings? [B1 Q10]
A. WHO 1 meningioma.
B. WHO 3 meningioma.
C. Haemangiopericytoma.
D. Melanocytoma.
E. Schwannoma

A

Haemangiopericytoma.
The imaging findings clearly describe an extra-axial tumour. These are classed as either lesions of meningeal origin or tumours of neurogenic origin. The broad dural base and dural tail indicates that this is a tumour of meningeal origin. Meningioma is the most common of these tumours, but the imaging characteristics are not typical. World Health Organization (WHO) 1 and 2 meningiomas demonstrate uniform enhancement, although haemorrhage into meningiomas is recognized. WHO 3 meningiomas (malignant meningiomas) are indicated on imaging when there is invasion of the adjacent parenchyma, which is not described. Otherwise, the diagnosis of malignant meningioma is made by an aggressive pattern of growth on serial imaging and biopsy. The dual arterial supply is also atypical. While meningiomas can have dual supply, they more typically derive their arterial supply from the external carotid (via the meningeal artery). Melanocytomas are suggested on imaging by increased signal on T1WI and are more commonly infratentorial. Without this, the diagnosis is again reached most following biopsy. All the features described are typical of haemangiopericytoma.

How well did you know this?
1
Not at all
2
3
4
5
Perfectly
67
Q

A 40-year-old female is investigated for worsening headaches. CT shows a well-defined hyperdense globular lesion within the trigone of the left lateral ventricle. There is intense contrast enhancement. The most likely diagnosis is: [B2 Q2]
a. Choroid cyst
b. Ependymoma
c. Colloid cyst
d. Meningioma
e. Neurocytoma

A

Meningioma
It is rare for meningiomas to occur intraventricularly (2–5% of all meningiomas) but they are the most common trigonal intraventricular mass in adulthood. They tend to occur in 40-year-old females.

How well did you know this?
1
Not at all
2
3
4
5
Perfectly
68
Q

Which of the following favours dural meningeal carcinomatosis rather than leptomeningeal cacinomatosis? [B3 Q21]
A. Short discontinuous thin sections of enhancement
B. Positive cytology
C. Thin area of subarachnoid enhancement following convulsions of gyri
D. Discrete leptomeningeal nodules
E. Invasion of underlying brain with mass effect and oedema

A

E
Dural meningeal carcinomatosis is rarely associated with positive cytology and involves localised or diffuse curvilinear enhancement underneath inner table in expected position of dura.

How well did you know this?
1
Not at all
2
3
4
5
Perfectly
69
Q

Which of the following is the most common radiation-induced CNS tumour? [B4 Q60]
a. ependymoma
b. oligodendroglioma
c. lymphoma
d. glioblastoma multiforme
e. meningioma

A

Meningioma
Meningioma is the most common radiation-induced CNS tumour and has been particularly associated with low-dose radiation treatment for tinea capitis. For the diagnosis of radiation-induced meningioma to be made, the meningioma must arise in the radiation field, appear after a latency period of years and should not have been the primary tumour irradiated. Radiation-induced meningiomas are more frequently multiple and have higher recurrence rates than non-radiation-induced tumours.

How well did you know this?
1
Not at all
2
3
4
5
Perfectly
70
Q

A 50-year-old man presents with headaches. A CT of the head reveals a 3 cm extra-axial lesion in the posterior fossa adjacent to the tentorium. The lesion is iso-dense with brain on non-contrast CT and has small areas of calcifications within. The lesion enhances homogenously with contrast. There is no surrounding oedema in the brain parenchyma. The most likely diagnosis of the abnormality is? [B5 Q3]
(a) Medulloblastoma
(b) Meningioma
(c) Lymphoma
(d) Glioblastoma multiforme
(e) Osteosarcoma metastasis

A

Meningioma
These are all typical features of a meningioma. Medulloblastoma is a childhood infratentorial tumour which is usually hyperdense and enhances with contrast. Lymphoma can be isodense with homogenous enhancement but usually is associated with significant peritumoral oedema and no calcifications. Glioblastoma multiforme is seen as an irregular lesion with mass effect, oedema and a heterogenous enhancement pattern. Sarcoma metastases are rare and would induce peritumoral oedema.

How well did you know this?
1
Not at all
2
3
4
5
Perfectly
71
Q

A 52-year-old man presents with headaches. Head CT shows a 4 cm extra-axial, homogenous, hyperdense lesion which enhances avidly with contrast. There is hyperostosis in the adjacent part of frontal bone. What is the most likely diagnosis? [B5 Q44]
(a) Meningioma
(b) Lymphoma
(c) Metastasis
(d) Glioma
(e) Oligodendroglioma

A

Meningioma
These tumours arise from the arachnoid ‘cap’ cells of the arachnoid villi.

How well did you know this?
1
Not at all
2
3
4
5
Perfectly
72
Q

A 45-year-old woman presents with a several-month history of neck pain and gradually progressive weakness and paraesthesia in the upper limbs. An MRI scan of the cervical spine is performed, and this shows a well-defined central intramedullary mass in the mid-cervical spinal cord. The mass is generally slightly hyperintense on T2WI, but also has a few low signal peripheral areas. It enhances homogeneously with gadolinium. What is the most likely diagnosis? [B1 Q27]
A. Astrocytoma.
B. Metastasis.
C. Haemangioblastoma.
D. Ganglioglioma.
E. Ependymoma

A

Ependymoma.
Ependymoma is the most common intramedullary neoplasm in adults. It tends to be centrally located within the cord, unlike astrocytoma, which can be eccentric. Astrocytoma can have a longer segment of cord involvement than ependymoma and may have a more infiltrative margin. The peripheral low signal areas seen on T2WI in ependymoma are related to haemosiderin deposition from prior haemorrhage.

How well did you know this?
1
Not at all
2
3
4
5
Perfectly
73
Q

A 22-year-old woman presents with upper and lower limb neurological symptoms and signs. She is subsequently discovered on MRI to have a mass in the cervical spinal cord. Which of the following features on MRI is going to point more towards a diagnosis of spinal cord astrocytoma, rather than ependymoma? [B1 Q48]
A. Predominant T2WI high signal.
B. Homogeneous enhancement post gadolinium.
C. Short segment of cord involvement.
D. Eccentrically placed lesion in the cord.
E. Sharply marginated lesion.

A

D
Eccentrically placed lesion in the cord. Table 6.2 illustrates the diagnostic features of astrocytoma and ependymoma.

How well did you know this?
1
Not at all
2
3
4
5
Perfectly
74
Q

A 52-year-old woman presents with gradually increasing gait disturbance and lower limb sensory symptoms. An MRI of her spine is performed, and this shows an anteriorly placed intradural, but extramedullary spinal mass. It is fairly markedly low signal on T1WI and T2WI and shows only minimal patchy enhancement post administration of intravenous gadolinium. What is the most likely diagnosis? [B1 Q49]
A. Neurofibroma.
B. Schwannoma.
C. Lymphoma.
D. Metastasis.
E. Meningioma.

A

Meningioma.
Spinal meningiomas are typically iso- to hypointense on T1WI and slightly hyperintense on T2WI. There is usually strong and homogeneous enhancement with gadolinium. However, some meningiomas may contain calcification and are typically the only intradural extramedullary tumours to do so. Some meningiomas can be heavily calcified and such a meningioma is being described in the question. These will remain dark on all MRI sequences and demonstrate only little contrast uptake (in the non-calcified areas). Schwannomas, neurofibromas, and metastases would not typically be hypointense on T2WI. Meningeal lymphomas are very rare and usually manifest as diffuse thickening of nerve roots and/or multiple enhancing nodules.

How well did you know this?
1
Not at all
2
3
4
5
Perfectly
75
Q

A 35-year-old man attends the accident and emergency department complaining of episodic lower back pain radiating down the legs. History and clinical examination also suggest pelvic sphincter dysfunction. MRI shows a spinal cord mass located at the conus medullaris. The mass is isointense on T1 and hyperintense on T2. It demonstrates contrast enhancement. The most likely diagnosis is: [B2 Q25]
a. Astrocytoma
b. Intradural lipoma
c. Haemangioblastoma
d. Myxopapillary ependymoma
e. Ganglioglioma

A

Myxopapillary ependymoma
This is a variant of ependymoma and is the most common neoplasm of the conus medullaris. It originates from ependymal glia of the filum terminale. Average age at presentation is 35 years and it is more common in men. T1-weighted imaging shows an isointense or occasionally hyperintense (due to the mucin content) mass. It is hyperintense on T2 and almost always shows enhancement postcontrast. Intradural lipomas are hyperintense on T1-weighted imaging, but they should not enhance. They also tend to occur in younger individuals and usually have an associated, clinically apparent lumbosacral mass. Haemangioblastoma can also demonstrate high signal on T1 but is also highly vascular, can show signal voids and approximately half of them will have an intra-tumoural cystic component. Gangliogliomas are much more common at the cervical and thoracic levels.

How well did you know this?
1
Not at all
2
3
4
5
Perfectly
76
Q

A follow-up of a young man under the care of the neurooncologists reveals a drop in metastases. Which statement is most true? [B3 Q28]
A. Drop metastases tend to be in the upper spine
B. Metastases within the spinal canal are usually ventral
C. Glioblastoma is the commonest cause of drop metastases
D. Are associated with positive CSF cytology in approximately 10% of cases
E. Usually demonstrates homogenous enhancement with contrast

A

E
Drop metastases are usually dorsal in location in the spinal canal. Medulloblastomas are the most common cause of drop metastases. A higher percentage of CSF cytology is positive.

How well did you know this?
1
Not at all
2
3
4
5
Perfectly
77
Q

Which is most likely to represent an intramedullary mass lesion? [B3 Q43]
A. Ependymoma
B. Meningioma
C. Neurofibroma
D. Arachnoid cyst
E. Abscess

A

Ependymoma
Other intramedullary masses include astrocytoma are dermoids (lipoma/teratomas), acutely expanding infarcts and haematoma.

How well did you know this?
1
Not at all
2
3
4
5
Perfectly
78
Q

A 50-year-old female presents with increasing weakness of the lower limbs and sensory disturbance. MRI shows an extramedullary, intradural ovoid mass in the mid-thoracic region. The lesion has signal isointense to the cord on T1W and T2W images and enhances avidly with intravenous gadolinium. What is the most likely diagnosis? [B4 Q98]
a. meningioma
b. nerve sheath tumour
c. ependymoma
d. dermoid
e. arachnoid cyst

A

Meningioma
Meningiomas in the spine are seen mainly in females (80%) and in those over 40 years of age. They have similar signal characteristics to the cord on MRI and enhance avidly. They can cause symptoms related to cord compression. Bone erosion is seen in ,10%. Nerve sheath tumours produce fusiform masses arising from the nerve roots, often extending through the intervertebral foramen, causing them to be dumbbell shaped. They are isointense to muscle on T1W and hyperintense to fat on T2W images. Ependymomas are usually located in the filum terminale and show low/intermediate signal on T1W images, with foci of high signal on T2W images, and they often enhance. Dermoids usually occur in the conus or cauda equina and are associated with spinal dysraphism in one-third of cases. They are variable in signal on T1W but are high signal on T2W images.

How well did you know this?
1
Not at all
2
3
4
5
Perfectly
79
Q

A 48-year-old female presents with tinnitus. CT shows a soft-tissue mass in the region of the hypotympanum. There is irregular bone demineralisation in the region of the carotid canal and jugular foramen, making their margins irregular and partially indistinct. On proton density MR imaging, the mass has mixed hyper- and hypo-intensity signal. The tumour shows strong enhancement after gadolinium administration. What is the most likely diagnosis? [B2 59]
a. Glomus tympanicum tumour
b. Glomus jugulare tumour
c. Carotid body tumour
d. Glomus vagale tumour
e. Cholesteatoma

A

Glomus jugulare tumour
All the tumours listed in the differential (apart from cholesteatoma) are paragangliomas. They grow slowly and rarely metastasise. Glomus jugulare tumours originate from the adventitia of the jugular vein. CT demonstrates a soft-tissue mass in the region of the jugular bulb/hypotympanum/middle ear space. Local bone destruction is common, particularly of the jugular plate or the lateral portion of the carotico-jugular spine. A unique ‘salt-and-pepper’ pattern of hyper- and hypo-intensity on T1- and T2-weighted images is also seen. This represents multiple small tumour vessels. They are highly vascular lesions, usually deriving a blood supply from branches of the external carotid artery.

How well did you know this?
1
Not at all
2
3
4
5
Perfectly
80
Q

A 45-year-old man presents with a 4-month history of worsening lower back pain radiating into the right lower extremity with weakness. An MRI scan of lumbar spine shows a 3-cm well-defined ovoid lesion eccentrically placed at the conus medullaris, the location of which is felt to be intradural and extramedullary. It is hypointense on T1WI and hyperintense on T2WI. There are flow voids and hemorrhage within the lesion. It enhances avidly post injection of contrast. Which of the following lesions fits best with the imaging findings? [B1 Q26]
A. Meningioma.
B. Schwannoma.
C. Neurofibroma.
D. Paraganglioma.
E. Metastasis

A

Paraganglioma.
Paragangliomas are rare intradural extramedullary tumours that are usually benign and have imaging characteristics as those described. Schwannomas, neurofibromas, and meningiomas are much more common, but do not typically contain vascular flow voids or areas of hemorrhage. Meningiomas may have a dural tail or foci of calcification. Schwannomas and neurofibromas can be difficult to distinguish on imaging. They are typically isointense on T1WI and markedly hyperintense on T2WI. Enhancement may be intense and homogenous or peripheral. Neurofibromas tend to encase, rather than displace, nerve roots. Leptomeningeal metastases present with three different imaging patterns: 1) Diffuse, thin enhancing coating of the surface of the spinal cord and nerve roots. 2) Multiple small enhancing nodules on the surface of the cord and nerve roots. 3) A single mass in the lowest part of the thecal sac.

How well did you know this?
1
Not at all
2
3
4
5
Perfectly
81
Q

A 60-year-old woman presents with a painless, slowly growing mass in the lateral aspect of the neck. The patient is referred for imaging with a clinical diagnosis of carotid body paraganglioma. Which of the following is a distinctive feature of carotid body paraganglioma on imaging? [B1 Q47]
A. Soft-tissue mass in the carotid space.
B. Intense enhancement after IV contrast administration.
C. High signal on T2WI.
D. Splaying of the internal and external carotid arteries.
E. Low signal on T2WI.

A

D. Splaying of the internal and external carotid arteries.
Carotid body tumour or paraganglioma is the most common paraganglioma of the head and neck. It arises from the paraganglionic cells located on the medial aspect of the carotid bifurcation. On MRI, they are of low to intermediate signal intensity on T1WI and hyperintense on T2WI. They are hypervascular and demonstrate intense enhancement after contrast administration. Splaying of the internal and external carotid arteries and multiple flow voids producing a ‘salt and pepper’ appearance are distinctive features on imaging.

How well did you know this?
1
Not at all
2
3
4
5
Perfectly
82
Q

A 56-year-old male presents with a mass in the anterior triangle of the neck and is referred for ultrasound scan and biopsy. Ultrasound scan shows a vascular mass splaying the internal and external carotid arteries at their origin. How should cell sampling of the mass be undertaken? [B4 Q64]
a. performs FNA rather than core biopsy
b. uses multiple passes
c. uses a single pass
d. ensures that needle midway between internal and external carotid arteries
e. both FNA and core biopsy contraindicated

A

Both FNA and core biopsy contraindicated
The imaging appearances are highly suggestive of a carotid body tumour (paraganglioma), and biopsy should not be performed, as hemorrhage would be certain due to the high vascularity of the lesion (in addition to the proximity of the carotid system). They present as painless, pulsatile masses in the neck of adults, below the angle of the jaw, and are laterally mobile but vertically fixed. There is splaying of the carotid bifurcation but preservation of calibre of the two arteries. Contrast imaging shows avid enhancement.

How well did you know this?
1
Not at all
2
3
4
5
Perfectly
83
Q

A 40-year-old hypertensive woman presents with a neck mass. A left carotid angiogram demonstrates an intensely enhancing mass splaying the carotid bifurcation. What is the most likely diagnosis? [B5 Q30]
(a) Metastasis
(b) Carotid body paraganglioma
(c) Lymphoma
(d) Branchial cyst
(e) Carotid dissection

A

Carotid body paraganglioma
Carotid body tumours are the most common extracranial head and neck paragangliomas. These typically splay the internal and external carotid arteries because they arise from the tissue located at the carotid artery bifurcation. They demonstrate an intense and persistent vascular blush on imaging. The combination of intense blush with flow voids on MRI has been described as a ‘salt and pepper’ appearance. These tumours may be familial and multicentric and are malignant in 10% cases.

How well did you know this?
1
Not at all
2
3
4
5
Perfectly
84
Q

A 45-year-old female patient is referred to you with a history of mild confusion and new onset seizures. This patient has a history of breast cancer and is currently undergoing chemotherapy following a wide local excision with axillary radiotherapy. The patient has a low-grade pyrexia and a low neutrophil count. An MRI reveals a solitary lesion in the right frontal lobe, which is low/intermediate signal on T1WI and high signal on T2WI. The lesion demonstrates ring enhancement. Diffusion-weighted imaging (DWI) reveals slightly increased signal on B1000 imaging, with increased signal on B0 imaging, and this area is bright on the apparent diffusion coefficient (ADC) map. What is the most likely diagnosis? [B1 Q3]
A. Abscess.
B. Metastasis.
C. Glioblastoma multiforme.
D. Infarct.
E. Radiotherapy change.

A

Metastasis.
The answer options give some of the classical radiological differentials for a ring-enhancing lesion in the brain: MAGIC DR (metastasis, abscess, glioblastoma multiforme, infarct, contusion, demyelinating conditions, and post-radiotherapy change). The clinical history should steer the reader toward the first two. DWI is reasonably useful at differentiating between these two (exceptions being in some fungal infections and toxoplasmosis), with necrotic metastasis having unrestricted diffusion and abscess having restricted diffusion. The appearances in this case are slightly confused by the DWI b1000 imaging showing increased signal. However, the presence of increased signal on the ADC map correctly identifies this as being T2 shine through, showing the importance of checking both sequences.

How well did you know this?
1
Not at all
2
3
4
5
Perfectly
85
Q

A 52-year-old man presents following collapse. He was previously fit and well, describing only a relatively recent history of dull back pain. Initial CT scan of the head reveals a 1.5 cm hyperdense mass at the corticomedullary junction of the right cerebral hemisphere. The mass shows surrounding oedema which exceeds the volume of the lesion. There is strong lesional enhancement following contrast injection. What is the most likely diagnosis and subsequent management? [B2 Q21]
a. Glioblastoma multiforme with subsequent MRI of the brain
b. Prostatic cancer metastasis with digital rectal examination and measuring of the prostatic specific antigen
c. Acute haemorrhagic contusion with referral to the neurosurgeons for active monitoring
d. Renal cell metastasis with subsequent CT staging
e. Brain abscess with subsequent intravenous antibiotics

A

Renal cell metastasis with subsequent CT staging
Brain metastases account for approximately a third of all intracranial tumours and are the most common intracranial neoplasm. They characteristically occur at the corticomedullary junction of the brain and have surrounding oedema that typically exceeds the tumour volume. Multiple lesions are present in approximately two-thirds of cases and should be searched for with administration of intravenous contrast. Most are hypodense on CT unless haemorrhagic or hypercellular, hence the lesion in this case is haemorrhagic. This lends itself to a differential of primary neoplasms which includes melanoma, renal cell carcinoma, thyroid carcinoma, bronchogenic carcinoma, and breast carcinoma. The history of back pain also suggests bone metastases. Glioblastoma multiforme usually appears as an irregular, heterogeneous, low-density mass. Abscesses typically demonstrate ring enhancement post-contrast and may show loculation and specules of gas. The patient’s history describes collapse rather than headache or confusion following a fall, which moves the differential away from traumatic contusion. Although prostate cancer typically metastasises to the vertebrae, it is an uncommon primary site for brain metastases, especially as the lesion described is haemorrhagic.

How well did you know this?
1
Not at all
2
3
4
5
Perfectly
86
Q

A 65-year-old woman is investigated for enophthalmos and headache. She is cachetic, anaemic and you suspect a metastatic process. CT head demonstrates an infiltrative retrobulbar mass. What is the most likely site of primary disease? [B2 Q33] {will repeat again in orbit session}
a. Breast
b. Lung
c. Renal
d. Melanoma
e. Ovarian

A

Breast
Most retrobulbar metastases are extraconal (outside the muscle cone). Neuroblastoma and Ewing’s sarcoma are the most common in children and produce smooth extraconal masses related to the posterior lateral wall of the orbit. In adults, an infiltrative retrobulbar mass and enophthalmos is characteristic of scirrhous carcinoma of the breast (invasive ductal carcinoma). Enophthalmia is also considered to be one of the earliest signs of metastatic breast cancer.

How well did you know this?
1
Not at all
2
3
4
5
Perfectly
87
Q

Which of the following favours dural meningeal carcinomatosis rather than leptomeningeal cacinomatosis? [B3 Q21]
A. Short discontinuous thin sections of enhancement
B. Positive cytology
C. Thin area of subarachnoid enhancement following convulsions of gyri
D. Discrete leptomeningeal nodules
E. Invasion of underlying brain with mass effect and oedema

A

Short discontinuous thin sections of enhancement
Dural meningeal carcinomatosis is rarely associated with positive cytology and involves localised or diffuse curvilinear enhancement underneath inner table in expected position of dura.

How well did you know this?
1
Not at all
2
3
4
5
Perfectly
88
Q

Which is the cause of a cystic rather than a haemorrhagic cause of brain metastases? [B3 Q22]
A. Adenocarcinoma of the lung
B. Malignant melanoma
C. Choriocarcinoma
D. Renal cell carcinoma
E. Thyroid carcinoma

A

Adenocarcinoma of the lung
Squamous cell lung cancer and adenocarcinoma of the lung cause cystic metastasis to the brain. Answers B-E are causes of haemorrhagic metastases.

How well did you know this?
1
Not at all
2
3
4
5
Perfectly
89
Q

A 64-year-old woman presents with progressive headache and confusion. On CT, she is found to have multiple, well-defined, rounded, low-attenuation masses of varying sizes in both hemispheres at the grey–white matter junction. The masses demonstrate intense enhancement following intravenous contrast, and there is considerable surrounding oedema. Which of the following is the most appropriate next imaging investigation? [B4 Q19]
a. mammography
b. thyroid ultrasound scan
c. barium enema
d. renal ultrasound scan
e. chest radiograph

A

Chest Radiograph
Brain metastases are the most common intracranial tumours. Six primary tumours account for 95% of all brain metastases. Primary bronchial carcinoma is the most common (47% of cases), though squamous cell carcinoma rarely metastasizes to the brain. Other common primary tumours are breast carcinoma (17%), gastrointestinal malignancy (15%), renal cell carcinoma, melanoma and choriocarcinoma. Metastases characteristically occur at the grey–white matter junction, are multiple in 66% of cases, and typically appear as hypodense masses that demonstrate solid or ring enhancement.

How well did you know this?
1
Not at all
2
3
4
5
Perfectly
90
Q

A 42-year-old Caucasian woman presents with multiple fits. CT of the head shows multiple, small enhancing lesions in the cortical and subcortical areas. On MRI, these lesions return low signal on T2 and hyperintense on post-gadolinium T1. What is the most likely diagnosis? [B5 Q43]
(a) Tuberous sclerosis
(b) Calcifications
(c) Melanoma metastases
(d) Haemorrhagic metastases
(e) Lymphoma

A

Melanoma metastases
The T2 shortening effect is attributed to the paramagnetic effects of iron and copper bound to melanin.

How well did you know this?
1
Not at all
2
3
4
5
Perfectly
91
Q

A 30-year-old male patient attends A&E 30 minutes after a head injury. He has consumed alcohol. You are contacted by the A&E doctor, who requests a CT brain. Currently, which of the following is a correct indication for immediate scanning? [B1 Q28]
A. Two episodes of vomiting.
B. GCS 13.
C. Loss of consciousness.
D. Amnesia for 20 minutes before accident.
E. Visual hallucinations.

A

Two episodes of vomiting.

Almost anything will buy you a CT brain these days, but surprisingly, the GCS must be less than 13 in the A&E department within 2 hours of the injury under NICE guidelines to warrant one. After 2 hours have passed, a GCS of anything less than normal is an indication for scanning. Other factors which require a CT within 1 hour are suspected open or depressed skull fracture, sign of fracture at skull base, post-traumatic seizure, focal neurological deficit and amnesia, or loss of consciousness and coagulopathy. Additional factors which necessitate a CT brain within 8 hours are amnesia of events for greater than 30 minutes before impact, and if there is any amnesia or loss of consciousness and the patient is older than 65/dangerous mechanism of injury. Note that, perversely, even if the scan can be delayed but be performed within 8 hours, the guidelines state it should be requested immediately. Prepare to be awoken from your sleep!

How well did you know this?
1
Not at all
2
3
4
5
Perfectly
92
Q

A 27-year-old female patient undergoes urgent neuroimaging following loss of consciousness because of an RTA. CT is unremarkable. MRI reveals multiple small areas of increased signal on T2WI in the white matter near the grey–white matter junction within the frontal and temporal lobes. In the same locations, DWI reveals areas of increased signal on the B1000 image and reduced signal on the ADC map. What is the most likely diagnosis? [B1 Q29]
A. Subarachnoid haemorrhage.
B. Extradural haematoma.
C. Subdural haematoma.
D. Hypoxic brain injury.
E. Diffuse axonal injury

A

Diffuse axonal injury (DAI).

CT is initially often normal (up to 80% of cases) in DAI. If positive, it may reveal small low attenuation foci (oedema) or high attenuation foci of petechial haemorrhage. The gray/white matter interface of the frontotemporal lobes, corpus callosum (especially the splenium), and brainstem are the most involved sites in DAI. MRI is much more sensitive and is the investigation of choice. The signal on MRI depends on the age of the lesion and whether haemorrhage is present, but classically hyperintense foci on T2WI sequences are seen acutely. In the more chronic phase, the lesions may only be detected as hypointense foci at characteristic locations on GE sequences: this appearance may remain for years. DWI reveals hyperintense foci of restricted diffusion on B1000 images, with corresponding low signal on the ADC map. The findings on DWI are easily distinguishable from extradural haematoma/subarachnoid haemorrhage/subdural haematoma/generalized oedema, which are discussed in other questions in this chapter.

How well did you know this?
1
Not at all
2
3
4
5
Perfectly
93
Q

A 44-year-old woman presents with severe facial injuries following an RTA. CT of the facial bones demonstrates multiple maxillofacial fractures in keeping with a Le Fort configuration. Which of the following statements regarding Le Fort fractures is false? [B1 Q51]
A. Any combination of Le Fort I, II, and III fractures can occur.
B. Disruption of pterygoid plates from the posterior maxilla is an essential finding in Le Fort fractures.
C. Le Fort fractures, by definition, refer to fractures involving the maxilla bilaterally.
D. Craniofacial separation is noted in Le Fort III pattern.
E. Le Fort fracture associated with a palate fracture will result in widening of the maxillary arch.

A

Le Fort fractures by definition refer to fractures involving the maxilla bilaterally.

Separation of all or a portion of the maxilla from the skull base is described as a Le Fort fracture. This can be unilateral when it is associated with sagittal or parasagittal fractures of the palate. Le Fort fractures by definition involve the posterior maxillary buttress at the junction of the posterior maxillary sinus and the pterygoid plates of the sphenoid. This may be either through the pterygoid plates or through the posterior walls of the maxillary sinus. Once a pterygomaxillary disruption has been identified, the remaining facial buttresses are inspected to identify the type of Le Fort fracture. In Le Fort I fracture, the maxillary arch will move in relation to the rest of the face and skull. In Le Fort II fracture, the entire maxilla will move in relation to the skull base. In Le Fort III, there is complete craniofacial separation. Any combination of Le Fort I, II, and III can occur. Posterior extension of Le Fort fracture into the hard palate results in widening of the maxillary arch and dental malocclusion.

How well did you know this?
1
Not at all
2
3
4
5
Perfectly
94
Q

A 40-year-old man with no previous medical history or medication attends the accident and emergency department. He was the driver of a car that was involved in a car-on-car vehicle collision at approximately 40mph. He was wearing a seat belt and his airbag deployed appropriately. According to NICE guidelines for head injury, which one of the following criteria alone does not warrant an acute head CT scan? [B2 Q34]
a. GCS <15 when he was assessed in the emergency department two hours after the accident
b. Haemotympanum
c. Amnesia of events <30 minutes after impact
d. Seizure following the accident
e. More than one episode of vomiting following the accident

A

Amnesia of events <30 minutes after impact

NICE defines head injury as ‘any trauma to the head, other than superficial injuries to the face’. All of the other criteria listed are requisites for an acute head CT scan. Haemotympanum implies a basal skull fracture which should be investigated by CT. Amnesia of events >30 minutes before impact would require an acute head CT. Any amnesia or loss of consciousness since the injury requires a CT scan if the patient is equal to or older than 65 years, has a coagulopathy (including warfarin treatment) or if there is a history of dangerous mechanism of injury, which is listed as:
* Pedestrian or cyclist struck by a motor vehicle.
* Occupant ejected from a motor vehicle.
* Fall from over one metre or five stairs.

How well did you know this?
1
Not at all
2
3
4
5
Perfectly
95
Q

A 24-year-old man is an unrestrained passenger in a car involved in a high-speed collision. He is found unconscious at the scene. CT of the brain is normal. MR scan of the brain demonstrates multiple small foci of high signal on T2W images in the white matter of the parasagittal regions of the frontal lobes and the periventricular regions of the temporal lobes. What is the most likely diagnosis? [B4 Q80]

a. acute subdural haematoma
b. diffuse axonal injury
c. cortical contusions
d. intracerebral haematoma
e. subcortical grey matter injury

A

b. diffuse axonal injury

Diffuse axonal injury is characterized by widespread axonal disruption occurring in response to acceleration or deceleration forces – direct impact is not necessary. Typically, patients are immediately unconscious after the injury. CT is commonly negative, though 20% of lesions contain sufficient haemorrhage to be visible. On MR scan, typical findings are of multiple small foci of decreased signal intensity on T1W images and increased signal intensity on T2W images. Characteristic locations are the frontal and temporal white matter near the grey–white matter junction. More severe injuries may involve the lobar white matter and corpus callosum, with the brain stem involved in the most severe cases. Cortical contusions usually involve the superficial grey matter, and patients are less likely to present with immediate loss of consciousness. They characteristically occur near bony protuberances and are more commonly haemorrhagic. Subcortical grey matter injury is an uncommon type of injury seen after severe head trauma, with petechial haemorrhages in the basal ganglia and thalamus.

How well did you know this?
1
Not at all
2
3
4
5
Perfectly
96
Q

A 32-year-old man presents with recent onset of migraine and TIAs. He also reports some cognitive decline. Cerebral angiogram is normal. An MRI of brain reveals discrete hyperintensities in the anterior temporal poles and external capsules. What is the most likely diagnosis? [B1 Q8]

A. Cerebral autosomal dominant arteriopathy with subcortical infarcts and leukoencephalopathy (CADASIL).
B. Mitochondrial myopathy, encephalopathy, lactic acidosis, and stroke (MELAS).
C. Myoclonic epilepsy with ragged red fibres (MERRF).
D. Sporadic subcortical arteriosclerotic encephalopathy (sSAE).
E. Protein S deficiency

A

A. Cerebral autosomal dominant arteriopathy with subcortical infarcts and leukoencephalopathy (CADASIL).

This is a hereditary small vessel disease, which causes stroke in young adults. The genetic mutation is found on chromosome 19. Presentation can include migraine, cognitive decline, psychiatric disturbance, TIAs, and stroke, the latter usually with substantial/complete recovery after individual strokes, particularly early in the disease process. Imaging reveals subcortical lacunar infarcts and leukoencephalopathy in young adults. The frontal lobe has the highest lesion load, followed by the temporal lobe and insula. Anterior temporal pole and external capsule lesions have higher sensitivity and specificity for CADASIL. The cerebral cortex is usually spared. MRI is the investigation of choice: CT will reveal only areas of hypodensity, and angiography is normal. sSAE is associated with hypertension and results in multiple lacunar infarcts in the lenticular nuclei, pons, internal capsule, and caudate nuclei. MELAS and MERRF are mitochondrial disorders. MELAS results in bilateral multiple cortical and subcortical hyperintense lesions on FLAIR images; MERRF has a propensity for the basal ganglia and caudate nuclei, and watershed ischaemia/infarcts are common. Hypercoagulable states such as protein S deficiency result in cortical and lacunar infarcts of various sizes, but the cerebral angiogram is abnormal.

How well did you know this?
1
Not at all
2
3
4
5
Perfectly
97
Q

A 74-year-old woman is admitted with acute left-sided hemiplegia of 2 hours onset. On assessment by the stroke team, she is deemed suitable for thrombolysis and referred for CT. As part of your institution’s work-up, CT perfusion (CTP) is performed following the unenhanced study. Which of the following CTP findings with regards to cerebral blood flow (CBF), cerebral blood volume (CBV) and mean transit time (MTT) are consistent with infarction? [B1 Q59]

A. Decreased CBF/decreased CBV/increased MTT.
B. Decreased CBF/increased CBV/increased MTT.
C. Decreased CBF/decreased CBV/decreased MTT.
D. Increased CBF/increased CBV/decreased MTT.
E. Increased CBF/decreased CBV/decreased MTT.

A

A. Decreased CBF/decreased CBV/increased MTT.

In identifying the ischaemic penumbra, CT perfusion offers promise in improved patient selection for thrombolysis beyond a rigid time window. In the infarct core (tissue which is not salvageable) both CBF and CBV are decreased with a corresponding increase in MTT. Penumbral tissue (which is potentially recoverable by thrombolysis) exhibits a CBF/CBV mismatch with an increased CBV, but decreased CBF (and increased MTT). An increase in CBF and CBV with decreased MTT is a feature noticed in tumours secondary to angiogenesis and microvascular permeability.

How well did you know this?
1
Not at all
2
3
4
5
Perfectly
98
Q

A 74-year-old man with a history of diabetes presents with a history of slurred speech and right-sided weakness, which spontaneously resolved over 1–2 hours. Initial unenhanced CT is normal. Further investigation by MRI is requested. What sequence is most sensitive for diagnosing acute ischaemia? [B1 Q66]

A. T2WI.
B. FLAIR.
C. DWI.
D. GE imaging.
E. T1WI post gadolinium

A

C. DWI.

The sensitivity and specificity of DWI in detecting acute ischaemia are 88–100% and 86–100%, respectively, and DWI is established as better than other conventional MRI techniques. In addition, acute from chronic ischaemia can be differentiated. DWI is a measure of the Brownian motion of water molecules within a tissue. Two sets of images are acquired: the DWI and the quantitative ADC map. Chronic lesions may appear bright on the DWI (T2 shine through), but only acute lesions with restricted diffusion will be dark on the ADC map. It should be noted, however, that high signal intensity on DWI with low ADC has been reported in a variety of conditions such as abscess, lymphoma, and gliomas. GE is useful for identifying microbleeds that may influence antithrombotic treatment.

How well did you know this?
1
Not at all
2
3
4
5
Perfectly
99
Q

You are called by a paediatrician to perform a cranial ultrasound on a term neonate who requires intensive therapy following delivery. Ultrasound demonstrates a well-defined area of increased parenchymal echogenicity over the periphery of the right parietal lobe. What is the most likely diagnosis? [B2 Q14]

a. Germinal matrix haemorrhage
b. Venous infarction
c. Middle cerebral artery infarction
d. Periventricular leukomalacia
e. Subarachnoid haemorrhage

A

c. Middle cerebral artery infarction

Vascular occlusive disease is rare in the neonatal period. When present, it is more common in the term infant and usually results from thrombosis rather than embolism. The MCA is most involved. Aetiology includes traumatic delivery, vasospasm due to meningitis and emboli secondary to congenital heart disease. Ultrasound demonstrates echogenic parenchyma in the distribution of the arterial territory.

How well did you know this?
1
Not at all
2
3
4
5
Perfectly
100
Q

Which of the following best describes the appearance of an acute ischaemic infarct of the brain? [B2 Q22]

a. Hypointense on DW MRI and low apparent diffusion coefficient (ADC) values.
b. Hyperintense on DW MRI and low ADC values
c. Hypointense on DW MRI and high ADC values
d. Hyperintense on DW MRI and high ADC values
e. Isointense on DW MRI and low ADC values

A

b. Hyperintense on DW MRI and low ADC values

Diffusion-weighted imaging is dependent on the motion of water molecules and provides information on tissue integrity. It is thought that interruption of cerebral blood flow results in rapid breakdown of energy metabolism and ion exchange pumps. This causes a shift of water from the extracellular compartment into the intracellular compartment, giving cytotoxic odema. This produces the hyperintensity on DW MR images. ADC values tend to be low within hours of stroke and continue to decline for the next few days. They remain reduced through the first four days and then show pseudonormalisation between four and ten days. After ten days the ADC tends to rise. Hyperintensity on diffusion weighted (DW) MRI and low ADC values are not pathognomonic of acute infar

How well did you know this?
1
Not at all
2
3
4
5
Perfectly
101
Q

Considering Moya-Moya disease in adults: [B3 Q41]
A. Infarct seen in cortical/subcortical areas
B. Multiple small flow voids are characteristic
C. Affects anterior circulation
D. Presentation with ischemia is more common in adults than in children
E. The supra-clinoid MCA is spared

A

B
Multiple flow voids are due to hypertrophied lenticulostriate arteries. Children more often present with ischemia and infarct in cortical/subcortical areas. Adults more often present with haemorrhage than children, but when infarcts do occur, they are most often in the deep white matter. The disease can involve the posterior circulation. The supra-clinoid MCA is the first to be involved.

How well did you know this?
1
Not at all
2
3
4
5
Perfectly
102
Q

Carotid Doppler examination is performed on a patient following a transient ischaemic attack. All other factors are in favour of surgical treatment for carotid stenosis. At what peak internal carotid artery velocity would ultrasound scan also support this management? [B4 Q68]
a. 50 cm/s
b. 75 cm/s
c. 100 cm/s
d. 200 cm/s
e. 250 cm/s

A

250 cm/s
Carotid ultrasound scan can be used to assess the common, external, and internal carotid arteries and the carotid bulb, including the vessel walls and the presence of plaques and stenoses. Doppler scan can display velocity profiles and allow waveform analysis and peak velocity measurement. Flow velocity increases proportionally with the degree of stenosis, except when the affected vessel is almost completely or totally occluded. Here flow velocity drops off. Stenosis above 70% is considered for surgery. This corresponds to a flow rate of greater than 250 cm/s.

How well did you know this?
1
Not at all
2
3
4
5
Perfectly
103
Q

A 30-year-old male with recurrent Transient Ischemic Attacks (TIAs) and a history of migraine with aura undergoes CT brain. Subcortical infarcts are identified raising suspicion of cerebral autosomal dominant arteriopathy with subcortical infarcts (CADASIL). Which is the most characteristically involved location for subcortical infarcts? [B3 Q3]
A. Anterior temporal pole
B. Frontal lobe
C. Centrum semi-ovale
D. Deep grey matter structure
E. Pons

A

Anterior temporal pole
A young patient with migraines, auras, TIAs or subcortical strokes should raise suspicion of CADASIL. Subcortical infarcts are characteristically in the anterior temporal pole and external capsule but may involve C, D and E.

How well did you know this?
1
Not at all
2
3
4
5
Perfectly
104
Q

An 80-year-old man presents acutely with a dense hemiplegia. CT perfusion is performed soon after admission, which suggests that the entire involved arterial territory is beyond recovery. Which of the following options represents the most likely combination of cerebral blood flow, mean transit time and cerebral blood volume, respectively, seen within the affected brain parenchyma, compared with unaffected parenchyma? [B4 Q4]

a. increased, increased, increased
b. increased, increased, decreased
c. increased, decreased, decreased
d. decreased, decreased, decreased
e. decreased, increased, decreased

A

Decreased, Increased, Decreased
Cerebral perfusion CT can distinguish viable but ischaemic tissue (the penumbra) from tissue that is beyond recovery. Other uses include evaluation of vasospasm after subarachnoid haemorrhage, assessment of cerebrovascular reserve with acetazolamide (cerebral arteriole vasodilator) in cases of vascular stenosis, evaluation of collateral flow and cerebrovascular reserve in patients having temporary balloon occlusion and assessment of microvascular permeability of intracranial neoplasms. Cerebral perfusion CT utilizes the central volume principle. This states that CBF=CBV/MTT, where CBF is cerebral blood flow, CBV is cerebral blood volume and MTT is mean transit time. In practice, two CT perfusion techniques can be used. One is perfused-blood volume mapping, in which a quantity is assigned to cerebral blood volume by subtracting unenhanced CT data from CT angiographic data. It has the advantage of imaging the whole brain. The second technique is a dynamic, contrast-enhanced technique that acquires data from a limited number of axial slices and monitors the first pass of an iodinated contrast agent bolus through the cerebral circulation. This requires an unenhanced CT brain, followed by a dynamic CT performed during injection of 50 ml of iodinated contrast (300 mg I/ml) at 4 ml/s. The first pass of contrast is observed in the brain. Cerebral perfusion is related to the concentration of iodinated contrast, which is directly related to the attenuation measured. Several maps are produced, including the CBV, CBF and MTT. MTT is derived from arterial and venous enhancement curves, measured by using regions of interest placed on an artery (one that is not occluded as part of an acute event) and a venous sinus. CBV is the area under the enhancement curves, and CBF is obtained from the central volume equation. Differentiation of infarcted brain from penumbra is important because, while penumbra can be saved by timely thrombolysis, infarcted tissue has an increased risk of bleeding from thrombolysis with no chance of recovery. CBF is decreased in both ischaemia and infarction, MTT is longer (.6 s) in both, while CBV is decreased in infarct but increased (or normal) in the penumbra due to cerebral autoregulatory mechanisms. MTT is the most sensitive for stroke. So, this or CBF can be used to detect stroke while CBV is used to determine whether there is infarct or reversible ischaemia.

How well did you know this?
1
Not at all
2
3
4
5
Perfectly
105
Q

A 75-year-old man undergoes an aortic valve replacement. His GCS remains low as the general anaesthetic effect wears off, and a new left-sided weakness is observed. Unenhanced CT of the brain is performed. The CT reveals a hypodensity (of attenuation value–30 HU) within the first segment of the right middle cerebral artery. Which of the following options is most likely to explain the patient’s abnormal neurology? [B4 Q9]

a. right middle cerebral artery thromboembolic occlusion
b. cerebral embolism as part of the fat embolism syndrome
c. right middle cerebral artery fat embolism
d. right middle cerebral artery air embolism
e. right middle cerebral artery dissection

A

Right Middle Cerebral Artery Fat Embolism
The hypodense artery sign is described, representing a single, large, macroscopic fat embolus within the middle cerebral artery, giving rise to a stroke syndrome. It may occur during cardiac surgery, resulting in dislodgement of fat from the surrounding tissue. This is distinct from the shower of microscopic fat emboli that occurs in the fat embolism syndrome.

How well did you know this?
1
Not at all
2
3
4
5
Perfectly
106
Q

A 70-year-old man is referred for CT scan of the brain due to sudden onset of left-sided hemiparesis and clinical diagnosis of stroke. Unenhanced CT shows a rounded area of low attenuation in the right posterior frontal lobe with local gyriform swelling and sulcal effacement. Upon questioning, he reveals a history of lung resection for malignancy 18 months previously. Which of the following imaging investigations would you perform next? [B4 Q25]

a. no further imaging
b. CT of the thorax
c. CT of the thorax and abdomen
d. MRI of the brain

A

Contrast-Enhanced CT of the brain
In this scenario, the low attenuation and surrounding changes most likely represent brain oedema. This may be due to an evolving infarction or oedema around a metastatic deposit from the previous lung cancer. Differentiation between the two will immediately affect patient treatment, as anti-platelet therapy for ischaemic stroke will increase the risk of haemorrhage from a metastasis and therefore should be withheld if a metastatic deposit is suspected or diagnosed. The primary factor in determining whether a lesion will enhance on CT after administration of intravenous iodinated contrast is the integrity of the blood–brain barrier in that region of the brain substance. A large molecule such as iodinated contrast would not be able to enter the brain unless the integrity of the barrier were compromised. Many aggressive tumours, including metastases, will disrupt this barrier, and so contrast enhancement will be seen in the solid component of these lesions. Acute infarction will typically not show areas of enhancement.

How well did you know this?
1
Not at all
2
3
4
5
Perfectly
107
Q

Considering Moya-Moya disease in adults: [B3 Q41]
A. Infarct seen in cortical/subcortical areas
B. Multiple small flow voids are characteristic
C. Affects anterior circulation
D. Presentation with ischemia is more common in adults than in children
E. The supra-clinoid MCA is spared

A

B
Multiple flow voids are due to hypertrophied lenticulostriate arteries. Children more often present with ischemia and infarct in cortical/subcortical areas. Adults more often present with haemorrhage than children, but when infarcts do occur, they are most often in the deep white matter. The disease can involve the posterior circulation. The supra-clinoid MCA is the first to be involved.

How well did you know this?
1
Not at all
2
3
4
5
Perfectly
108
Q

Carotid Doppler examination is performed on a patient following a transient ischaemic attack. All other factors are in favour of surgical treatment for carotid stenosis. At what peak internal carotid artery velocity would ultrasound scan also support this management? [B4 Q68]
a. 50 cm/s
b. 75 cm/s
c. 100 cm/s
d. 200 cm/s
e. 250 cm/s

A

e. 250 cm/s
Carotid ultrasound scan can be used to assess the common, external, and internal carotid arteries and the carotid bulb, including the vessel walls and the presence of plaques and stenoses. Doppler scan can display velocity profiles and allow waveform analysis and peak velocity measurement. Flow velocity increases proportionally with the degree of stenosis, except when the affected vessel is almost completely or totally occluded. Here flow velocity drops off. Stenosis above 70% is considered for surgery. This corresponds to a flow rate of greater than 250 cm/s.

How well did you know this?
1
Not at all
2
3
4
5
Perfectly
109
Q

A 30-year-old male with recurrent Transient Ischemic Attacks (TIAs) and a history of migraine with aura undergoes CT brain. Subcortical infarcts are identified raising suspicion of cerebral autosomal dominant arteriopathy with subcortical infarcts (CADASIL). Which is the most characteristically involved location for subcortical infarcts? [B3 Q3]
A. Anterior temporal pole
B. Frontal lobe
C. Centrum semi-ovale
D. Deep grey matter structure
E. Pons

A

A. Anterior temporal pole
A young patient with migraines, auras, TIAs or subcortical strokes should raise suspicion of CADASIL. Subcortical infarcts are characteristically in the anterior temporal pole and external capsule but may involve C, D and E.

How well did you know this?
1
Not at all
2
3
4
5
Perfectly
110
Q

An 80-year-old man presents acutely with a dense hemiplegia. CT perfusion is performed soon after admission, which suggests that the entire involved arterial territory is beyond recovery. Which of the following options represents the most likely combination of cerebral blood flow, mean transit time and cerebral blood volume, respectively, seen within the affected brain parenchyma, compared with unaffected parenchyma? [B4 Q4]
a. increased, increased, increased
b. increased, increased, decreased
c. increased, decreased, decreased
d. decreased, decreased, decreased
e. decreased, increased, decreased

A

d. decreased, decreased, decreased
Cerebral perfusion CT can distinguish viable but ischaemic tissue (the penumbra) from tissue that is beyond recovery. Other uses include evaluation of vasospasm after subarachnoid haemorrhage, assessment of cerebrovascular reserve with acetazolamide (cerebral arteriole vasodilator) in cases of vascular stenosis, evaluation of collateral flow and cerebrovascular reserve in patients having temporary balloon occlusion and assessment of microvascular permeability of intracranial neoplasms. Cerebral perfusion CT utilizes the central volume principle. This states that CBF=CBV/MTT, where CBF is cerebral blood flow, CBV is cerebral blood volume and MTT is mean transit time. In practice, two CT perfusion techniques can be used. One is perfused-blood volume mapping, in which a quantity is assigned to cerebral blood volume by subtracting unenhanced CT data from CT angiographic data. It has the advantage of imaging the whole brain. The second technique is a dynamic, contrast-enhanced technique that acquires data from a limited number of axial slices and monitors the first pass of an iodinated contrast agent bolus through the cerebral circulation. This requires an unenhanced CT brain, followed by a dynamic CT performed during injection of 50 ml of iodinated contrast (300 mg I/ml) at 4 ml/s. The first pass of contrast is observed in the brain. Cerebral perfusion is related to the concentration of iodinated contrast, which is directly related to the attenuation measured. Several maps are produced, including the CBV, CBF and MTT. MTT is derived from arterial and venous enhancement curves, measured by using regions of interest placed on an artery (one that is not occluded as part of an acute event) and a venous sinus. CBV is the area under the enhancement curves, and CBF is obtained from the central volume equation. Differentiation of infarcted brain from penumbra is important because, while penumbra can be saved by timely thrombolysis, infarcted tissue has an increased risk of bleeding from thrombolysis with no chance of recovery. CBF is decreased in both ischaemia and infarction, MTT is longer (.6 s) in both, while CBV is decreased in infarct but increased (or normal) in the penumbra due to cerebral autoregulatory mechanisms. MTT is the most sensitive for stroke. So, this or CBF can be used to detect stroke while CBV is used to determine whether there is infarct or reversible ischaemia.

How well did you know this?
1
Not at all
2
3
4
5
Perfectly
111
Q

A 75-year-old man undergoes an aortic valve replacement. His GCS remains low as the general anaesthetic effect wears off, and a new left-sided weakness is observed. Unenhanced CT of the brain is performed. The CT reveals a hypodensity (of attenuation value–30 HU) within the first segment of the right middle cerebral artery. Which of the following options is most likely to explain the patient’s abnormal neurology? [B4 Q9]
a. right middle cerebral artery thromboembolic occlusion
b. cerebral embolism as part of the fat embolism syndrome
c. right middle cerebral artery fat embolism
d. right middle cerebral artery air embolism
e. right middle cerebral artery dissection

A

c. right middle cerebral artery fat embolism
The hypodense artery sign is described, representing a single, large, macroscopic fat embolus within the middle cerebral artery, giving rise to a stroke syndrome. It may occur during cardiac surgery, resulting in dislodgement of fat from the surrounding tissue. This is distinct from the shower of microscopic fat emboli that occurs in the fat embolism syndrome.

How well did you know this?
1
Not at all
2
3
4
5
Perfectly
112
Q

A 70-year-old man is referred for CT scan of the brain due to sudden onset of left-sided hemiparesis and clinical diagnosis of stroke. Unenhanced CT shows a rounded area of low attenuation in the right posterior frontal lobe with local gyriform swelling and sulcal effacement. Upon questioning, he reveals a history of lung resection for malignancy 18 months previously. Which of the following imaging investigations would you perform next? [B4 Q25]
a. no further imaging
b. CT of the thorax
c. CT of the thorax and abdomen
d. MRI of the brain
e. contrast enhanced CT of the brain

A

e. Contrast-Enhanced CT of the brain
In this scenario, the low attenuation and surrounding changes most likely represent brain oedema. This may be due to an evolving infarction or oedema around a metastatic deposit from the previous lung cancer. Differentiation between the two will immediately affect patient treatment, as anti-platelet therapy for ischaemic stroke will increase the risk of haemorrhage from a metastasis and therefore should be withheld if a metastatic deposit is suspected or diagnosed. The primary factor in determining whether a lesion will enhance on CT after administration of intravenous iodinated contrast is the integrity of the blood–brain barrier in that region of the brain substance. A large molecule such as iodinated contrast would not be able to enter the brain unless the integrity of the barrier were compromised. Many aggressive tumours, including metastases, will disrupt this barrier, and so contrast enhancement will be seen in the solid component of these lesions. Acute infarction will typically not show areas of enhancement.

How well did you know this?
1
Not at all
2
3
4
5
Perfectly
113
Q

An unenhanced CT scan of the brain is performed 3 hours after the onset of signs suggestive of ischaemic stroke. Which of the following image window parameters is most likely to reveal the early CT changes? [B4 Q34]
a. width 400 HU, centre 40 HU
b. width 80 HU, centre 20 HU
c. width 8 HU, centre 32 HU
d. width 0 HU, centre 0 HU
e. width 1500, centre–500 HU

A

c. Width 8 HU, centre 32 U

How well did you know this?
1
Not at all
2
3
4
5
Perfectly
114
Q

A 75-year-old woman presents with sudden onset left homonymous superior quadrantanopia. Head CT shows a subtle hypo-intensity in the right medial occipital lobe. On MRI, the occipital region shows high signal on FLAIR. Diffusion weighted images show high signal in the right medial occipital lobe consistent with reduced diffusion. What is the most likely diagnosis? [B5 Q32]
(a) Acute occipital lobe infarct in posterior cerebral artery territory
(b) Occipital lobe tumour
(c) Haemorrhage
(d) Old occipital lobe infarct
(e) Acute occipital lobe infarct in middle cerebral artery territory

A

Acute occipital lobe infarct in posterior cerebral artery territory
Homonymous visual field defects result from retrochiasmal lesions, which include the optic tracts, lateral geniculate body, optic radiations, and visual cortex. Unilateral posterior cerebral artery infarction results in isolated hemianopia. The superior portion of the visual field projects to the inferior occipital cortex. Thus, a partial or branch occlusion of the posterior cerebral artery may lead to an isolated superior or inferior visual field defect.

How well did you know this?
1
Not at all
2
3
4
5
Perfectly
115
Q

A 69-year-old lady was admitted 10 days ago following an acute intracerebral haematoma diagnosed on CT. What are the most likely radiological findings on the follow-up MRI scan of brain? [B1 Q4]
A. Haematoma hypointense to grey matter on T1WI, hyperintense on T2WI.
B. Haematoma hyperintense to grey matter on both T1WI and T2WI.
C. Haematoma hyperintense to grey matter on T1WI, hypointense on T2WI.
D. Haematoma hypointense to grey matter on both T1WI and T2WI.
E. Haematoma isointense to grey matter on both T1WI and T2WI.

A

Haematoma hyperintense to grey matter on both T1WI and T2WI.
The MRI appearances of intracranial haemorrhage are determined primarily by the state of the haemoglobin (Hb), which evolves with age. This can be staged as hyperacute (first few hours), acute (1–3 days), early subacute (3–7 days), late subacute (4–7 days to 1 month), or chronic (1 month to years).

How well did you know this?
1
Not at all
2
3
4
5
Perfectly
116
Q

A 68-year-old male attends the emergency department after being found slumped in his chair at home by a carer. CT head shows an intraparenchymal bleed. On MRI the lesion is hyperintense on T1 and hypointense on T2-weighted imaging. Which of the following stages of haemorrhage best correlates with the MRI findings? [B2 Q10]
a. Oxyhaemoglobin
b. Deoxyhaemoglobin
c. Intracellular methaemoglobin
d. Extracellular methaemoglobin
e. Haemosiderin

A

Intracellular methaemoglobin

How well did you know this?
1
Not at all
2
3
4
5
Perfectly
117
Q

Which of the following statements best describes the features of amyloid angiopathy? [B2 Q40]
a. Solitary, peripheral haemorrhagic focus in a hypertensive elderly patient
b. Solitary, central haemorrhagic focus in a normotensive young patient
c. Multiple, peripheral haemorrhagic foci in a normotensive elderly patient
d. Multiple, central haemorrhagic foci in a normotensive elderly patient
e. Multiple, central haemorrhagic foci in a hypertensive young patient

A

Multiple, peripheral haemorrhagic foci in a normotensive elderly patient
Multiple peripheral haemorrhages (corticomedullary junction) in normotensive elderly patients are suggestive of amyloid angiopathy. Hypertensive haemorrhage tends to occur centrally around the basal ganglia (putamen, external capsule), thalamus and pons.

How well did you know this?
1
Not at all
2
3
4
5
Perfectly
118
Q

Which of the following best describes the typical appearance of intracranial haemorrhage in the late subacute phase (8-30 days)? [B3 Q24]
A. Increased T1, increased T2
B. Increased T1, decreased T2
C. Isointense T1, decreased T2
D. Isointense T1, increased T2
E. Decreased T1, decreased T2

A

A
B represents early subacute haemorrhage, C represents acute (12-72 hours), D represents hyperacute (less than 12 hours) and E represents chronic haemorrhage (> 1 month).

How well did you know this?
1
Not at all
2
3
4
5
Perfectly
119
Q

A 62-year-old man with a history of falls and confusion undergoes MR of the brain. This demonstrates a subdural haematoma of high signal intensity on T1W images and of high signal intensity with a hypointense rim on T2W images. What is the most likely age of the haematoma? [B4 Q8]
a. <6 hours
b. 8–72 hours
c. 3 days to 1 week
d. 1 week to several months
e. several months to several years

A

1 week to several months
In the first 3–6 hours (hyperacute stage) following haemorrhage, the intact red cells contain mostly oxyhaemoglobin, which appears hyperintense on T2W images. Desaturation occurs peripherally, forming deoxyhaemoglobin, which is seen as hypointensity on T2W images. In the acute stage (8–72 hours), there is rapid deoxygenation of the oxyhaemoglobin to deoxyhaemoglobin, which, together with the high protein content of the clot and susceptibility effects, results in iso- to hypointensity on T1W images and hypointensity on T2W images. In the early subacute stage (3–7 days), oxidation of deoxyhaemoglobin to methaemoglobin occurs inside the red cell, resulting in characteristic hyperintensity on T1W images due to paramagnetic effects, and marked hypointensity on T2W images. In the late subacute stage (1 week to months), extracellular methaemoglobin results in persistent hyperintensity on T1W images, but increasing signal intensity on T2W images, with peripheral susceptibility effects causing a low intensity rim. In the chronic stage (months to years), iron atoms are deposited as haemosiderin and ferritin, which cause susceptibility effects resulting in low signal intensity on both T1W and T2W images.

How well did you know this?
1
Not at all
2
3
4
5
Perfectly
120
Q

Which of the following imaging features of an intraparenchymal haemorrhage is most likely to suggest that the haemorrhage originates from an underlying tumour? [B4 Q75]
a. homogeneous CT appearance
b. complete haemosiderin ring
c. complete resolution on follow-up imaging
d. solitary haemorrhage
e. post-contrast enhancement of non-haemorrhagic areas

A

Post-contrast enhancement of non-haemorrhagic areas
Haemorrhage occurs in 5–10% of tumours. Features suggesting tumour rather than benign haemorrhage include complex CT pattern, incomplete haemosiderin ring, persisting oedema or mass on follow-up studies, and multiple lesions.

How well did you know this?
1
Not at all
2
3
4
5
Perfectly
121
Q

A hypertensive, 75-year-old female admitted with an acute stroke is found on unenhanced CT of the brain, to have an acute basal ganglia haemorrhage. If an MRI were performed 2 weeks later, what signal characteristics would the region of haemorrhage return? [B4 Q79]
a. isointense on T1W images and hyperintense on T2W images
b. hyperintense on T1W images and hyperintense on T2W images
c. isointense on T1W images and hypointense on T2W images
d. hyperintense on T1W images and hypointense on T2W images
e. hypointense on T1W images and a hypointense rim on T2W images

A

Hyperintense on T1W images and hyperintense on T2W images
The MRI signal of blood depends first on whether it is moving or static, since on most sequences movement produces a signal void. When it is static, the signal returned by blood reflects the magnetic properties of the blood products and their location. Hyperacute haemorrhage is intracellular oxyhaemoglobin that is diamagnetic, returning an isointense T1 and bright T2 signal. At 1–2 days, deoxygenation has occurred, making the iron paramagnetic. It remains intracellular and returns an isointense T1 signal and is dark on T2W images. At 2–7 days, haemorrhage contains paramagnetic intracellular methaemoglobin. This is bright on T1W images and dark on T2W images. The methaemoglobin becomes extracellular from 1 week to 4 weeks, and the MRI signal is bright on both T1W and T2W sequences. Chronic haemorrhage contains haemosiderin/ferritin, which is ferromagnetic, is dark on T1W images and has a dark rim on T2W sequences.

How well did you know this?
1
Not at all
2
3
4
5
Perfectly
122
Q

A 34-year-old man undergoes MRI of brain after admission for head trauma. Which of the following sequences is most sensitive for subarachnoid haemorrhage?
A. T1WI.
B. T1WI with fat saturation.
C. T2WI.
D. FLAIR.
E. Proton density.

A

FLAIR.
Although CT is generally used for investigating acute subarachnoid haemorrhage (SAH), FLAIR sequence on MRI has been suggested as being as sensitive as or more sensitive than CT. It is particularly useful in regions where CT may be limited due to beam hardening artefacts or if there is a very small amount of blood. Acute SAH appears as high intensity on FLAIR within the cisterns and sulci. Subacute SAH may be better appreciated on MRI because of its high signal intensity when the blood is isointense to CSF on CT. SAH differs from intra-parenchymal haemorrhage in that the mix of blood with high-oxygen tension CSF delays generation of paramagnetic deoxyhaemoglobin, and oxyhaemoglobin remains present longer than in intra-parenchymal haemorrhage. This contributes to continued T2 prolongation. Beware that there are other pathological (meningitis, leptomeningeal metastases, acute stroke, fat-containing tumour/dermoid rupture) and benign (artefact, supplemental oxygenation) causes of FLAIR hyperintensity in the subarachnoid space. Chronic haemorrhage from SAH is best detected on GE sequences, resulting in marked subarachnoid low signal (the blooming of superficial siderosis).

How well did you know this?
1
Not at all
2
3
4
5
Perfectly
123
Q

A 72-year-old man from a nursing home presents to the accident and emergency department. Nurses have noticed increasing confusion following a fall six days earlier. His inflammatory markers are normal. A non-contrast CT scan of the head demonstrates a crescent-shaped collection in the left fronto-parietal region. The collection is iso-dense to CSF and there is no midline shift, nor hydrocephalus. On T1-weighted MR imaging the lesion is isointense to CSF. The most likely diagnosis is: [B2 Q19]
a. Subdural hygroma
b. Brain atrophy
c. Subdural empyema
d. Chronic subdural haematoma
e. Enlarged subarachnoid space

A

Subdural hygroma
This is a traumatic subdural effusion which shows up as a localised CSF-fluid collection within the subdural space. They present in the elderly or in young children usually 6–30 days following trauma. The majority are asymptomatic, but patients may present with increasing confusion or headaches. They are devoid of blood products on imaging, unlike chronic subdural haematomas. Subdural haematomas are also more likely to cause effacement of the ventricular system and loss of the normal sulci-gyral pattern. Normal inflammatory markers and lack of pyrexia lessen the probability of an empyema.

How well did you know this?
1
Not at all
2
3
4
5
Perfectly
124
Q

A 27-year-old man suffers a head injury. A CT brain is performed. Which of the following features favours a subdural haematoma (SDH) over an extradural haematoma (EDH)? [B1 Q32]
A. The haematoma measures 50 HU.
B. The presence of a temporal skull fracture.
C. The haematoma crosses the midline over the falx.
D. The collection has a biconvex configuration.
E. The haematoma crosses sutures.

A

The haematoma crosses sutures.
An SDH will cross suture lines and may extend over the whole cerebral hemisphere; an EDH will not. Only an EDH can cross the midline, and this usually occurs at the vertex in the setting of a venous EDH. EDHs are more usually arterial bleeds and associated with temporal skull fractures, which disrupt the adjacent middle meningeal artery. The latter are typically biconvex; SDHs are typically lentiform in shape. Both measure 50–60 HU if acute. Chronic SDHs may be iso- or hypodense to brain, although anaemia or clotting disorders can produce a similar appearance. EDH is a neurosurgical emergency more so than SDH because of the potential lucent period followed by sudden deterioration as arterial bleeding continues after having stripped the dura from the inner table of the skull.

125
Q

You are contacted by the A&E department requesting a CT brain for a 28-year-old female presenting with headache which reached peak intensity 15 minutes after onset. The headache came on during sexual activity and is associated with vomiting. Which feature in the clinical history is most predictive of subarachnoid haemorrhage? [B1 Q63]
A. Female sex.
B. Age <40 years.
C. 15-minute onset to peak intensity.
D. Onset during sexual activity.
E. Association with vomiting

A

Association with vomiting.
Headache accounts for approximately 2% of all A&E department visits, yet subarachnoid haemorrhage accounts for less than 3% of these headaches. Up to half of all patients with subarachnoid haemorrhage, however, will be alert and neurologically intact. A recent study prospectively assessed neurologically intact patients presenting with headache peaking within 1 hour to determine which variables are predictive of subarachnoid haemorrhage. High-risk variables include age >40, neck pain, witnessed loss of consciousness, onset with exertion (but not sexual activity), arrival by ambulance, vomiting, and blood pressure >160/100 mmHg. The presence of one or more of these findings in a patient with an acute non-traumatic headache reaching maximum intensity within 1 hour should prompt further investigation.

126
Q

A 42-year-old man is admitted with sudden onset headache suspicious of subarachnoid haemorrhage. There is no loss of consciousness. Unenhanced CT demonstrates blood in the cisterns around the midbrain and into the anterior part of the interhemispheric fissure. Subsequent CT angiogram (CTA) is negative. What is advised next for this patient? [B1 Q65]
A. Lumbar puncture.
B. MRA.
C. Catheter angiography.
D. Repeat CTA in 1 week.
E. No further investigation necessary (peri-mesencephalic haemorrhage).

A

Catheter angiography.
Peri-mesencephalic haemorrhage is confined to the cisterns around the midbrain. In some cases, the only blood is found anterior to the pons. Some sedimentation into the lateral ventricles may occur, but frank haemorrhage into the ventricles and/or extension into the sylvian fissure or anterior interhemispheric fissure (as in this case) indicates arterial haemorrhage. Catheter angiography following a negative CTA in the setting of a peri-mesencephalic bleed arguably does not have any extra diagnostic yield. However, catheter angiography remains the gold standard for detecting aneurysms, and in this case an anterior communicating artery aneurysm requires exclusion.

127
Q

A 36-year-old woman with known polycystic kidney disease presents with a history of sudden onset headache and has signs of meningism. A CT brain reveals subarachnoid haemorrhage with haematoma within the septum pellucidum. What is the most likely site for an intracerebral aneurysm? [B2 Q9]
a. Anterior communicating artery
b. Posterior communicating artery
c. A2 segment of an anterior cerebral artery
d. Tip of the basilar artery
e. Middle cerebral artery

A

Anterior communicating artery.
A clot in the septum pellucidum is virtually diagnostic of an aneurysm of the anterior communicating artery. Aneurysms of the distal anterior cerebral artery are less common.

128
Q

Which of the following best represents the decline in positive CT findings for a clinically suspected subarachnoid haemorrhage from scanning at 12 hours post-ictus to 3 days post-ictus? [B2 Q12]
a. 90% positive at 12 hours to 70% positive at 3 days
b. 90% positive at 12 hours to 60% positive at 3 days
c. 98% positive at 12 hours to 90% positive at 3 days
d. 98% positive at 12 hours to 75% positive at 3 days
e. 90% positive at 12 hours to 50% positive at 3 days

A

d.
98% positive at 12 hours to 75% positive at 3 days

129
Q

Which of the following features favours extradural rather than subdural haematoma? [B3 Q37]
A. Extra-axial collection
B. Mixed hyper and hypodense areas
C. Crossing of sutures
D. Crossing of dural attachment
E. Skull fractures

A

D
Both are extra-axial and can be of mixed attenuation. Although skull fractures are more common in EDH, this cannot be the single best answer as EDH cross dural attachments are limited by cranial sutures.

130
Q

A 60-year-old female admitted with severe, sudden-onset headache is found to have widely distributed subarachnoid haemorrhage. A saccular aneurysm is identified on CT angiography. From which of the following locations in the circle of Willis is this aneurysm most likely to arise? [B4 Q24]
a. basilar tip
b. middle cerebral artery bifurcation
c. junction of anterior cerebral and anterior communicating arteries
d. pericallosal artery
e. vertebral artery

A

c. junction of anterior cerebral and anterior communicating arteries
It is at this location that 35% of berry aneurysms are found. This is the same proportion that occurs at the junction of the internal carotid and posterior communicating arteries. Five per cent occur at the basilar tip and 20% at the middle cerebral artery bifurcation. Two per cent of the population have cerebral aneurysms. They are multiple in 20% of cases and giant in 25% (over 25 mm in diameter). They are caused by degenerative vascular changes, trauma, infection, tumour and vasculopathy. The incidence is increased in adult polycystic renal disease, aortic coarctation, fibromuscular dysplasia, and Marfan’s and Ehlers–Danlos syndromes.

131
Q

A 50-year-old female presents with a ‘thunder-clap’ headache and fluctuating level of consciousness. Unenhanced CT of the brain shows high-density material layered in the occipital horns of the lateral ventricles. Which of the following is the most appropriate next imaging investigation? [B4 Q53]
a. CT angiography
b. CT venography
c. MR angiography
d. catheter angiography
e. radionuclide regional cerebral blood flow imaging

A

a. CT angiography
Some 80–90% of subarachnoid haemorrhage is due to aneurysmal rupture. Therefore, when the diagnosis is proven or strongly suspected, the cerebral arterial circulation requires assessment. Sixteen-slice CT angiography (CTA) is as accurate for aneurysm detection as 2D catheter digital subtraction angiography (DSA). CTA can be used as primary imaging and DSA reserved for difficult cases. It has the advantages of being safe, immediate, swift and less likely to need sedation or general anaesthetic. CTA also offers anatomical information beyond just the cerebral vasculature, in contrast to DSA. With 16-slice CTA, the spatial resolution is similar to DSA: 0.40 mm for CTA and 0.32 mm in DSA (2D DSA 33 cm FOV, 1024×1024 matrix, resolution 0.32×0.32 mm). When DSA is performed with AP and lateral projections, aneurysms may be masked, but 3D rotational DSA avoids this problem. The CTA parameters are as follows: 0.5 rotation/s with a 10 mm table advance per rotation and a 5-scan, 16 rows of 0.75 mm reconstructed to 0.4 mm, kVp 120, mA 130.

132
Q

One week following a subarachnoid haemorrhage, a 50-year-old female, who had thus far been making a good recovery, develops intractable vomiting, vertigo, and ataxia. An unenhanced CT of the brain demonstrates unchanged ventricular size but a new hypodense region in a cerebellar hemisphere. Which of the following complications of subarachnoid haemorrhage is most likely to have occurred? [B4 Q55]
a. acute obstructive hydrocephalus
b. delayed communicating hydrocephalus
c. vasospasm and infarction
d. transtentorial herniation
e. rebleed from a berry aneurysm

A

c. vasospasm and infarction
Infarction in this case is cerebellar. Vasospasm is the leading cause of death and morbidity following subarachnoid haemorrhage. It occurs after 72 hours with a peak at 5–17 days. Acute obstructive hydrocephalus results from intraventricular blood or the resultant ependymitis obstructing the aqueduct of Sylvius or the outlet of the fourth ventricle. Communicating hydrocephalus usually occurs after 1 week when the haemorrhage causes impaired CSF absorption.

133
Q

A 14-year-old boy is hit by a vehicle while riding his bike and sustains a left-sided head injury. Paramedics report a transient loss of consciousness at the scene. On examination, he is confused with a GCS of 13/15. Unenhanced CT of the brain is performed, which shows a biconvex extra-axial collection in the left temporoparietal region with attenuation value of 65 HU. On bone windows, there is an undisplaced, linear, left temporoparietal skull fracture. What is the most likely diagnosis? [B4 Q93]
a. acute subdural haematoma
b. acute extradural haematoma
c. acute subarachnoid haemorrhage
d. acute intracerebral haematoma
e. cerebral cortical contusion

A

b. acute extradural haematoma
Acute extradural haematomas are traumatic in origin and tend to be commoner in younger patients. Two-thirds of cases involve the temporoparietal region, and 75–95% of patients have an associated skull fracture. Bleeding is usually from the underlying middle meningeal artery, though it may also be due to disruption of the middle meningeal vein, dural venous sinuses or diploic veins. Typical appearances are of a high-density lentiform or biconvex collection, which forms between the inner table of the skull and the dura mater. The dura is firmly bound to the skull at sutural margins, so extradural haematomas tend not to cross suture lines. Acute subdural haematomas usually follow severe trauma and have a poorer prognosis than acute extradural haematomas due to a high incidence of associated contusions and other brain injuries. They tend to be crescentic in shape and may freely extend across suture lines, being limited only by the interhemispheric fissure and tentorium. They are commonly bilateral, particularly in infants, and have no association with skull fractures.

134
Q

A 40-year-old man attends the Accident & Emergency Department with acute onset of the most severe headache of his life. CT of the head demonstrates subarachnoid haemorrhage in the left sylvian fissure and early hydrocephalus. What is the most likely site of the ruptured aneurysm? [B5 Q1]
(a) Basilar artery
(b) Right middle cerebral artery
(c) Left middle cerebral artery
(d) Anterior communicating artery
(e) Posterior cerebral artery

A

(c) Left middle cerebral artery (MCA)
Rupture of a left MCA aneurysm is likely to be seen as haemorrhage in the ipsilateral sylvian fissure.

135
Q

A 28-year-old unconscious man was admitted to the Accident & Emergency Department after a motorcycle accident. He briefly regained consciousness and then started to decline again. CT of the head shows a fracture of his right parietal bone over a lentiform extra-axial haematoma with midline shift. What is the most likely diagnosis? [B5 Q28]
(a) Subdural haematoma
(b) Extradural haematoma
(c) Subarachnoid haemorrhage
(d) Meningioma
(e) Intraparenchymal bleed

A

(b) Extradural haematoma
Classically an extradural haematoma is seen as a lentiform/biconvex hyperdense collection bounded by cranial sutures and associated with skull fracture. The collection may be heterogeneous if active bleeding is occurring.

136
Q

A 52-year-old man with a history of previous treated arteriovenous malformation in brain presents with bilateral sensorineural hearing loss. MRI of the brain shows that the contours of brain outlined by hypointense rim on T2 and T2* GRE images. What is the most likely diagnosis? [B5 Q38]
(a) Central nervous system siderosis
(b) MR sequence artefacts
(c) Brain surface vessels
(d) Neurocutaneous melanosis
(e) Meningoangiomatosis

A

(a) Central nervous system siderosis
Recurrent subarachnoid bleeds cause haemosiderin deposition on the surface of the brain, brainstem and leptomeninges. This is seen as hypointense signal on T2 and T2* GRE images.

137
Q

A 32-year-old woman of 38 weeks’ gestation presents with seizure following a headache. She is referred for CT querying venous thrombosis. Unenhanced CT brain demonstrates bilateral low attenuation change within the thalami. Given this location, where is the most likely site of thrombosis? [B1 Q61]
A. Superior sagittal sinus.
B. Transverse sinus.
C. Sigmoid sinus.
D. Straight sinus.
E. Cavernous sinus.

A

Straight sinus.
Pregnancy is a risk factor for both venous sinus thrombosis and hypertensive haemorrhage secondary to eclampsia. Hyperattenuating thrombus within the occluded sinus is classical on the unenhanced CT but is seen in only 25% of cases. On the contrast-enhanced study, a central filling defect surrounded by enhancing dura (empty delta sign) is present in over 30%. Most of the superior cerebrum is drained by the superior sagittal sinus. The transverse sinuses receive blood from the temporal, parietal, and occipital lobes. The transverse sinuses drain into the sigmoid sinuses and on into the internal jugular veins. The straight sinus forms from the confluence of the inferior sagittal sinus and vein of Galen. The vein of Galen is part of the deep venous system, which drains the corpus callosum, basal ganglia, thalami, and upper brainstem. Cavernous sinus thrombosis presents with proptosis and cranial nerve palsies (usually cranial nerve VI first). The cavernous sinus receives the petrosal sinuses and middle cerebral veins.

138
Q

A patient presents with diplopia and is found to have unilateral restricted eye movements. On MRI, the extraocular muscles of the same eye are seen to be enlarged and oedematous. Which of the following additional features favours a carotid–cavernous sinus fistula as the diagnosis, over Graves’ disease, lymphoma or orbital pseudotumour? [B4 Q30]
a. distal tapering of the swollen extraocular muscles
b. stranding in the orbital fat
c. extraocular muscles isointense to fat on T2W images
d. dilated superior ophthalmic vein
e. extraocular muscle swelling involving the tendons

A

Dilated superior ophthalmic vein
Carotid–cavernous sinus fistula can be caused by penetrating injury, base of skull fracture or rupture of an intracavernous internal carotid artery aneurysm. Drainage from the affected cavernous sinus is via the superior ophthalmic vein, contralateral cavernous sinus, petrosal sinus and, rarely, cortical veins. Pulsatile exophthalmos and orbital bruit are evident, as are chemosis, conjunctival oedema and increased intraocular pressure. On imaging, the cavernous sinus may be enlarged, and can cause erosion of the sella and enlargement of the superior orbital fissure. Regarding the differentials for extraocular muscle swelling, Graves’ disease typically spares the muscle attachments whereas inflammatory orbital pseudotumour affects the tendons while also causing orbital fat stranding.

139
Q

Which paired vein forms in the sylvian fissure and travels in the ambient cistern around the midbrain to enter the vein of Galen along with the internal cerebral vein? [B4 Q67]
a. superficial middle cerebral or sylvian vein
b. basal vein of Rosenthal
c. vein of Labbe
d. vein of Trollard
e. thalamo-striate vein

A

Basal vein of Rosenthal
These veins are all part of the supratentorial venous system. The superficial middle cerebral vein forms an arc along the surface of the sylvian fissure and is continuous with the sphenoparietal sinus. The veins of Trolard and Labbe are anastomotic veins that connect the superficial middle cerebral vein to the superior sagittal and transverse sinuses respectively. The thalamostriate vein is a subependymal vein that passes across the floor of the lateral ventricle, over the thalamus and into the internal cerebral vein behind the foramen of Monro. The paired internal cerebral veins run along the roof of the third ventricle and enter the vein of Galen with the paired basal veins of Rosenthal. The vein of Galen joins the inferior sagittal sinus and the straight sinus at the ‘venous confluence’ within the quadrigeminal plate cistern. The straight sinus lies along the junction of the falx and tentorium. The straight sinus, transverse sinus and superior sagittal sinus meet as the torcular herophili.

140
Q

A 38-year-old male presents with a history of recent head injury and unilateral, painful, pulsatile exophthalmos with reduced visual acuity. CT shows swelling of the extraocular muscles with dilatation of the superior ophthalmic vein and cavernous sinus on the ipsilateral side, both of which enhance avidly. What is the most likely diagnosis? [B4 Q87]
a. ophthalmic varix
b. arteriovenous malformation
c. lymphangioma
d. carotid–cavernous fistula
e. pseudotumour of orbit

A

Carotid-cavernous fistula
Carotid–cavernous sinus fistula may occur spontaneously (secondary to aneurysm rupture, dural sinus thrombosis or atherosclerosis) or after trauma/surgery. It presents with a classic triad of pulsatile exophthalmos, conjunctival chemosis and auscultatory bruit. However, reduction in visual acuity may be the only sign. Secondary findings such as proptosis, congestive extraocular muscle enlargement, and distension with enhancement of the superior ophthalmic vein and cavernous sinus are usually identified on routine CT or MRI. However, catheter angiography is usually required for lesion classification and treatment planning prior to embolization. Drainage may also occur into the contralateral cavernous sinus, resulting in dilatation and enhancement. Ophthalmic varices present with intermittent exophthalmos related to straining, with dilatation of superior and/or inferior ophthalmic veins, which may thrombose. Orbital pseudotumour is an idiopathic inflammatory condition affecting all orbital contents, but vascular dilatation is not a feature.

141
Q

A 30-year-old female with a past medical history of spontaneous pulmonary embolus presents 2 weeks after giving birth with severe headache, vomiting and drowsiness. Unenhanced CT of the brain shows areas of low attenuation with sulcal effacement and small areas of parenchymal haemorrhage. These changes do not conform to an arterial distribution. What is the most likely finding on the post-contrast CT brain? [B4 Q88]
a. ring enhancement of the low-attenuation regions
b. demonstration of a basilar tip aneurysm
c. ‘empty delta’ sign
d. anterior pituitary enlargement
e. vertebral artery dissection flap

A

Empty delta sign
The patient has a venous sinus thrombosis causing congestion and venous infarction. Veno-occlusive disease is commoner in the first 3 weeks postpartum, especially if there is underlying hypercoagulability, including factor V Leiden abnormality, antiphospholipid antibody syndrome, and protein C, protein S or antithrombin III deficiency. On unenhanced CT, there may be hyperdense veins, grey–white matter junction haemorrhage and brain oedema. On CT venography, an ‘empty delta’ can be seen because thrombus rather than iodinated contrast occupies the affected dural venous sinus.

142
Q

A 36-year-old man is admitted following a seizure. Unenhanced CT demonstrates a right frontal mixed attenuation lesion, which avidly enhances post contrast. Multiple flow voids are seen on MRI of brain. What finding on catheter angiography differentiates arteriovenous malformation (AVM) from other vascular lesions of the brain?

A. Early venous drainage.
B. Arterial stenoses of feeder vessels.
C. External carotid trans-dural supply.
D. Dilated medullary veins (caput medusa).
E. Dilated cortical veins.

A

Early venous drainage.

Brain AVMs are abnormal connections between arteries that would normally supply the brain tissue and veins that normally drain the brain, resulting in shunting with an intervening network of vessels within the brain parenchyma. The finding of early venous drainage is important in differentiating brain AVMs from other vascular lesions. Cortical venous drainage may be seen in superficial lesions. Recruitment of transdural supply is observed in large lesions, although this is a more typical feature of proliferative angiopathy, which is a diffuse type of AVM. Classically, in this condition, normal brain parenchyma is interspersed between the abnormal vessels. Stenoses of feeder vessels are also often identified in proliferative angiopathy. The caput medusa of dilated medullary veins is a feature of DVAs, a normal variant. Dilated cortical veins are seen in dural arteriovenous fistulas (DAVFs), which are abnormal connections between arteries that normally supply the meninges (but not the brain) and small venules within the dura.

143
Q

A 54-year-old female is admitted following a seizure. CT of the brain demonstrates a rounded, 2-cm hyperdense lesion within the right temporal lobe, which exhibits calcification. No enhancement is seen post contrast. On follow-up MRI of brain, the lesion is of mixed intensity on T1WI. The lesion has a hypointense rim on T2WI. Prominent susceptibility effect is noted on T2* GE imaging. DWI is normal and no enhancement is demonstrated on T1WI post gadolinium. Based on the imaging findings, what is the most likely diagnosis?

A. AVM.
B. Haemorrhagic neoplasm.
C. Cavernous malformation (cavernoma).
D. Amyloid angiopathy.
E. Capillary telangiectasia.

A

Cavernous malformation (cavernoma).

This is a benign vascular hamartoma, which can occur anywhere throughout the CNS, although spinal cord involvement is rare. It is typically a discrete, lobulated mass <4 cm containing blood products at different stages of evolution. On MRI they are described as having a ‘popcorn’ appearance of mixed signal intensity with a hypointense haemosiderin rim. They display little to no enhancement. Prominent susceptibility effect on T2* GE is typical. AVMs will demonstrate multiple flow voids. Haemorrhagic neoplasms will exhibit strong enhancement and an incomplete haemosiderin rim. Multiple ‘black dots’ are a feature of amyloid angiopathy. Capillary telangiectasias are poorly demarcated lesions, usually <1 cm, demonstrating ‘brush-like’ enhancement on T1WI post contrast. They are not usually identified on CT.

144
Q

A 29-year-old woman undergoes a brain CT for severe worsening headache. She is 20 weeks pregnant. No acute pathology is demonstrated. However, an incidental finding of a small right parietal arteriovenous malformation is noted. This is confirmed on subsequent MRI. You are asked to counsel the patient. What is the most appropriate advice?

a. Normally the risk of haemorrhage is 2–3% per year but this increases by a factor of 10 during pregnancy. Recommend monthly follow-up with brain MRI
b. Normally the risk of haemorrhage is 10% per year but haemorrhage is likely during labour. Recommend angiography with a view to coil embolization
c. Normally the risk of haemorrhage is 10% per year but there is no significant increase in risk during pregnancy. Recommend annual brain MRI for surveillance
d. Normally the risk of haemorrhage is 2–3% per year but there is no significant increase in risk during pregnancy. Recommend review after delivery to discuss treatment options
e. Normally the risk of haemorrhage is 15% per year but there is no significant increase in risk during pregnancy. Recommend review after delivery to discuss treatment options

A

d. Normally the risk of haemorrhage is 2–3% per year but there is no significant increase in risk during pregnancy. Recommend review after delivery to discuss treatment options.

Arteriovenous malformations (AVMs) are congenital lesions consisting of multiple arteries and veins, connecting as a fistula without an intervening normal capillary bed. The majority are solitary lesions except when associated with hereditary haemorrhagic telangiectasia. Approximately 90% occur in the supratentorial region (parietal > frontal > temporal). About half of AVMs present with haemorrhage (intracerebral > subarachnoid > intraventricular), approximately 25% present with focal or generalised seizures and 15% present with headache. The overall risk of haemorrhage for an unruptured AVM is about 2–3% per year. This increases to 6–17% in the first year following first haemorrhage and then decreases to a baseline level by the third year. Following a second haemorrhage, the annual risk of re-bleed rises to 25% in the following year. An approximate lifetime risk for patients with previously unruptured AVMs can be calculated as 105 minus the patient’s age in years. Current data suggests that pregnancy does not increase the risk of an AVM haemorrhage, however, when possible, AVMs should be treated before pregnancy. If the lesion is noted during pregnancy and there is no haemorrhage, treatment risks should be considered higher than the risk of haemorrhage, and treatment may be deferred until after pregnancy.

145
Q

A 40-year-old male has a CT brain to investigate seizures. Which of the following best describes a cavernoma?

A. Popcorn appearance with high SI centre on T1+T2
B. Most commonly occurs in the cerebellum
C. Usually considerable mass effect
D. Hypodense region on CT
E. Rarely calcifies

A

A.

Cavernomas are most located in subcortical regions and are hypodense areas which can calcify. They are associated with minimal mass effect/oedema. Cavernomas can undergo haemorrhagic change and are associated with seizures.

146
Q

A 20-year-old woman has a right-sided proptosis, chemosis and bruit over the orbit. On MRI she is found to have dense enhancement of the right-sided extra-ocular muscles with dilatation of the right cavernous sinus and periorbital soft tissues. Angiography reveals abnormal early opacification of a dilated left cavernous sinus and superior ophthalmic vein. Which is the most likely diagnosis?

A. Carotid-cavernous fistula
B. Thyroid eye disease
C. Orbital pseudotumor
D. Retro-orbital metastases
E. Optic nerve plexiform neurofibroma

A

A. Carotid-cavernous fistula.

MR angiography shows reversal of flow through the Superior Ophthalmic Vein (SOV), rapid progress of proptosis, visual loss or raised intracranial pressure requiring emergency treatment.

147
Q

A 70-year-old man has severe rhinorrhoea and then develops a cough with haemoptysis. A chest radiograph shows a large nodule in the right lung, and a subsequent CT thorax demonstrates a cavitary, left-lung nodule. A cerebral catheter angiogram, undertaken to investigate a focal motor deficit, is most likely to reveal which of the following?

a. a giant berry aneurysm
b. multiple aneurysms with stenoses and occlusions
c. dural arteriovenous malformation
d. contrast extravasation
e. numerous collaterals supplying the anterior circle of Willis

A

b. multiple aneurysms with stenoses and occlusions.

Nasal and paranasal involvement and migratory lung nodules, which can be cavitary, are typical features of Wegener’s granulomatosis. Cerebral vasculitis can also be a feature. MRI findings are non-specific and include hyperintensities on T2W images, infarcts and haemorrhage. Angiography may demonstrate occlusion, stenoses and aneurysms.

148
Q

A 19-year-old man with axillary freckling has a cerebral angiogram for recurrent hemiparetic episodes. There is bilateral occlusion of the distal internal carotid arteries extending into the proximal anterior and middle cerebral arteries. This is associated with an extensive network of upper brain-stem collaterals. Which of the following is the most likely cause of the angiographic findings in this patient?

a. sickle cell disease
b. atherosclerosis
c. radiation vasculopathy
d. Moyamoya syndrome
e. amyloid angiopathy

A

d. Moyamoya syndrome.

Moyamoya disease is progressive arteritis typically affecting both supraclinoid internal carotid arteries and the anterior two vessels of the circle of Willis – the anterior and middle cerebral arteries. It is termed ‘Moyamoya syndrome’ when associated with neurofibromatosis, bacterial meningitis, head trauma, tuberculosis, oral contraceptives, atherosclerosis or sickle cell anaemia. Sickle cell anaemia and atherosclerosis are not associated with axillary freckling, unlike neurofibromatosis type 1. Amyloid angiopathy should be suspected in elderly normotensive patients with multiple areas of intra-axial haemorrhage sparing the basal ganglia.

149
Q

A 65-year-old woman with a history of previous subarachnoid haemorrhage presents with slowly progressive cognitive decline and worsening gait. A CT brain reveals ventricular dilatation with rounded frontal horns and periventricular hypodensity. What is the most likely diagnosis? [B1 Q33]

A. Heavy metal toxicity secondary to aneurysm clip.
B. Parkinson’s disease.
C. Alzheimer’s disease.
D. Normal pressure hydrocephalus.
E. Binswanger’s disease.

A

D. Normal pressure hydrocephalus.

Fifty per cent of cases of normal pressure hydrocephalus (NPH) are idiopathic, while 50% have a cause, e.g. meningitis, neurosurgery, head trauma, or, as in the current vignette, previous subarachnoid haemorrhage. The imaging reveals dilated ventricles with rounded frontal horns, which are enlarged out of proportion to the degree of sulcal enlargement (as one would see in cerebral atrophy). On CT there may be periventricular low attenuation in the frontal and occipital regions, representing transependymal CSF flow. The MRI equivalent is periventricular high signal and there are other MRI findings, including upward bowing of the corpus callosum and the aqueductal flow void sign (reflecting increased CSF velocity through the cerebral aqueduct). The bradykinesia and gait apraxia of NPH may mimic Parkinson’s disease, although there is typically no associated tremor. Binswanger’s disease (also known as subcortical arteriosclerotic encephalopathy) is a continuous, irreversible ischaemic degeneration of periventricular and deep white matter. MRI reveals extensive periventricular and deep white matter hyperintensities, which reflect microinfarctions and demyelination, and enlarged ventricles.

150
Q

A 62-year-old man is referred for MRI of brain after presenting with cognitive decline, gait apraxia, and urinary incontinence. There is a preceding history of chronic headache. Ventriculomegaly is noted on initial CT. Which of the following conventional MRI findings distinguishes aqueductal stenosis from normal pressure hydrocephalus? [B1 Q71]

A. Periventricular T2WI hyperintensity.
B. Normal sulci.
C. Aqueduct flow void.
D. Funnel-shaped aqueduct.
E. Relatively normal calibre fourth ventricle.

A

D. Funnel-shaped aqueduct.

Aqueductal stenosis is a focal reduction in aqueduct size, which can be congenital or acquired. Stenosis occurs at the level of the superior colliculi or at the inter-collicular sulcus. The best diagnostic clue is a funnel-shaped aqueduct. There is resultant ballooning of the lateral and third ventricles. The fourth ventricle is normal distal to the obstruction. The most specific finding is lack of CSF flow through the aqueduct on phase contrast MRI. Other findings include thinning of the corpus callosum and downward displacement of the internal cerebral veins and third ventricular floor. In older patients it can present similarly to NPH. Imaging findings in NPH include ventriculomegaly (with relative sparing of the fourth ventricle) out of proportion to sulcal enlargement, with normal hippocampi. The aqueductal ‘flow-void’ sign reflects increased CSF velocity through the aqueduct, although this can be observed in normal individuals.

151
Q

A 24-year-old male boxer is admitted with concussion following a head injury. His admission CT does not demonstrate any evidence of intracranial injury, but the A&E physician asks you about a midline CSF space. You explain that this is a cavum velum inter-positum. What distinguishes this CSF space from cavum velum inter-positum and cavum vergae? [B1 Q72]

A. Position between the frontal horns of the lateral ventricles.
B. Posterior extension between the fornices.
C. Does not extend anterior to foramen of Monro.
D. Mildly hyper-density to CSF in lateral ventricles.
E. Absent septum pellucidum.

A

C. Does not extend anterior to foramen of Monro.

Cavum septum pellucidum (CSP), cavum vergae (CV), and cavum vellum interpositum (CVI) are all considered normal variants, although CSP is possibly more prevalent in boxers due to repeated head trauma, most famously referred to in Rocky V. CSP is universal in foetuses, but decreases with age. CSP is an elongated finger-shaped CSF collection between the frontal horns of the lateral ventricles. Posterior extension between the fornices is referred to as CV. CV almost never occurs in the absence of CSP. CVI, however, is a triangular-shaped CSF space between the lateral ventricles that does not extend anterior to the foramen of Monro. Absent septum pellucidum can look like CSP/CV on sagittal imaging. It is commonly associated with other congenital anomalies.

152
Q

Routine first-trimester antenatal ultrasound scan reveals a large posterior fossa cyst and ventriculomegaly. Foetal MRI demonstrates dysgenesis of the corpus callosum, a large posterior fossa and hypoplasia of the cerebellar vermis. What is the most likely diagnosis? [B2 Q8]

a. Dandy–Walker malformation
b. Dandy–Walker variant
c. Megacisterna magna
d. Arachnoid cyst
e. Porencephaly

A

a. Dandy–Walker malformation

Dandy–Walker malformation is characterised by an enlarged posterior fossa (not seen in Dandy–Walker variant) with high-rising tentorium cerebelli, dys/agenesis of the cerebellar vermis (intact in megacisterna magna) and cystic dilatation of the fourth ventricle (normal in arachnoid cyst). Ventriculomegaly is also common.

153
Q

A CT brain of a 25-year-old male with a head injury but no focal neurology shows no acute abnormality. A thin cerebrospinal fluid (CSF) density is noted between the frontal horns of the lateral ventricles in the midline. Which is the diagnosis? [B3 Q3]

A. Cavum septi pellucidi
B. Cavum vergae
C. Cavum veli interpositum
D. Colloid cyst
E. Arachnoid cyst

A

A. Cavum septi pellucidi

Occurs in 80% of term infants and 15% of adults. Rarely may dilate and cause obstructive hydrocephalus.

154
Q

What is the most sensitive sign on non-contrast CT for detecting early hydrocephalus? [B4 Q22]

a. cortical sulcal effacement
b. uncal herniation
c. enlarged third ventricle
d. enlarged fourth ventricle
e. enlarged temporal horns of the lateral ventricles

A

e. enlarged temporal horns of the lateral ventricles

In many cases of hydrocephalus due to subarachnoid haemorrhage, the temporal horns of the lateral ventricles become dilated sooner than the frontal horns. Dilatation of the temporal horn is often particularly conspicuous, as it is frequently not visualized at all on CT of normal brains. Uncal herniation is herniation of the medial temporal lobe into a subtentorial location, where it may exert pressure on the brain stem and is a late sign of raised intracranial pressure, often presenting with oculomotor nerve palsy resulting in a fixed dilated pupil.

155
Q

A 31-year-old woman attends neurology clinic with a history of orthostatic headache, worst on standing, which sometimes induces vomiting, and relieved by lying down. MRI of the brain and cervical spine shows crowding of the foramen magnum due to low-lying cerebellar tonsils, elongation of the fourth ventricle, effacement of the prepontine cistern and a prominent pituitary gland. In the spine, an extradural fluid-signal collection is identified ventral to the cord. Administration of intravenous gadolinium reveals smooth areas of intracranial pachy meningeal enhancement. Which of the following diagnoses is best supported by these findings?

a. intracranial hypotension
b. intracranial hypertension
c. migraine
d. Chiari I malformation
e. Dandy–Walker malformation

A

a. intracranial hypotension

Spontaneous intracranial hypotension is a syndrome of low CSF pressure characterized by postural headaches in patients without any history of dural puncture or penetrating trauma. It is thought to arise from an occult CSF leak due to dural defects reducing CSF volume and subsequently pressure. Intracranial findings include downward displacement of the brain, subdural effusions, engorgement of other venous structures (including the hyperaemic pituitary) and low-lying cerebellar tonsils. There may be flattening of the pons as the brain sags against the skull base. Diffuse pachymeningeal enhancement is due to increased venous supply to maintain intracranial volume and therefore pressure, according to the Monro–Kellie doctrine. In the spine, extradural fluid collection is indicative of an occult leak. Treatment is with an epidural blood patch where autologous blood is introduced into the extradural (epidural) space in an attempt to seal the microscopic dural defects.

156
Q

A 13-year-old child with a history of perinatal infection presents with headaches and vomiting. CT of the brain shows gross dilatation of the lateral and third ventricles with a normal 4th ventricle. No tumour masses are seen. The most likely diagnosis is? [B5 Q23]

(a) Aqueduct stenosis
(b) Klippel–Feil syndrome
(c) Chiari I malformation
(d) Dandy–Walker malformation
(e) Neurofibromatosis

A

(a) Aqueduct stenosis

There are various causes for aqueductal stenosis, however the imaging features of this condition are dilated lateral and third ventricles with a normal 4th ventricle. This is the most frequent cause of congenital hydrocephalus. Its aetiology may be classified as post-inflammatory (commonest), congenital or neoplastic (rare).

157
Q

A 16-year-old male with a history of epilepsy is investigated via MRI. Axial T2WI demonstrates a cystic space within the left frontal lobe isointense to CSF. This is causing local mass effect and there is adjacent enlargement of the left lateral ventricle. What is the most likely diagnosis? [B1 Q68]

A. Porencephalic cyst.
B. Arachnoid cyst.
C. Schizencephaly.
D. Hydranencephaly.
E. Ependymal cyst.

A

Porencephalic cyst.

  • Porencephaly is a congenital/acquired cystic cavity within the brain parenchyma with adjacent enlargement of the lateral ventricle. They develop in utero or early infancy.
  • Arachnoid cysts are also CSF isointense, but are extra-axial, displacing the brain away from the adjacent skull.
  • Ependymal cysts are intra-ventricular and the surrounding brain is usually normal.
  • Schizencephaly is characterized by an intraparenchymal cleft extending from the ventricular surface to the brain surface lined by gray matter.
  • Hydranencephaly results from an early destructive process of the developing brain. The cranial vault is CSF filled with absence of the cortical mantle and ventricles (water-bag brain). Death in infancy is typical.
158
Q

Routine first-trimester antenatal ultrasound scan reveals a large posterior fossa cyst and ventriculomegaly. Fetal MRI demonstrates dysgenesis of the corpus callosum, a large posterior fossa and hypoplasia of the cerebellar vermis. What is the most likely diagnosis? [B2 Q8]

a. Dandy–Walker malformation
b. Dandy–Walker variant
c. Megacisterna magna
d. Arachnoid cyst
e. Porencephaly

A

Dandy–Walker malformation

Dandy–Walker malformation is characterised by an enlarged posterior fossa (not seen in Dandy–Walker variant) with high-rising tentorium cerebelli, dys/agenesis of the cerebellar vermis (intact in megacisterna magna) and cystic dilatation of the fourth ventricle (normal in arachnoid cyst). Ventriculomegaly is also common.

159
Q

A five-year-old boy presents with visual fixation. The ophthalmologist suspects optic nerve hypoplasia. Brain CT confirms optic nerve hypoplasia, an absent septum pellucidum and a third cerebral abnormality. What is the third abnormality likely to be? [B2 Q48]

a. Schizencephaly
b. Cortical dysplasia
c. Polymicrogyria
d. Agenesis of the corpus callosum
e. Type II Chiari malformation

A

Schizencephaly

The combination of absent septum pellucidum and optic nerve hypoplasia is indicative of septo-optic dysplasia (SOD). Most patients present in infancy with visual disturbance, seizures, or endocrine abnormalities (pituitary dysfunction is seen in approximately 50% of cases). Additional abnormalities are often present with schizencephaly being the most common (50%).

160
Q

On review of an MRI of a complex case marked hypointensity of the globus pallidus is noted on T2W1, surrounding a higher intensity centre, ‘eye of the tiger sign’. Which is the most likely cause? [B3 Q13] [? Wrong Key]

A. Dandy-Walker malformation
B. Multiple sclerosis
C. Hypoparathyroidism
D. Wilson’s disease
E. Hypoglycaemia

A

D. Wilson’s disease

Due to excess iron accumulation, a central gliosis is associated with Haltervorden-Spatz disease, dementia, retinitis pigmentosa, X-linked disorders with mental retardation, atrophy and Parkinsonian disorders.

161
Q

Which of the following associations favour Chiari I rather than Chiari II malformation?

A. Klippel-Feil anomaly
B. Myelomyeocele
C. Dysgenesis corpus callosum
D. Absence of septum pellucidi
E. Excessive cortical gyration

A

A. Klippel-Feil anomaly

Syringohydromyelia, hydrocephalus, malformation of small bones and spine are also associations of Type I Chiari malformation.

162
Q

A child who undergoes MR of the brain for clinically apparent facial abnormalities is shown to have a defect of midline cleavage of the brain. What structure is abnormal or absent in all forms of holoprosencephaly, and therefore is the most sensitive indicator of a midline cleavage abnormality? [B4 Q2]

a. falx cerebri
b. third ventricle
c. fourth ventricle
d. corpus callosum
e. septum pellucidum

A

e. septum pellucidum

Holoprosencephaly is failure of the primitive brain to cleave into two hemispheres and is commonly associated with midline facial abnormalities (ranging from cyclopia to hypertelorism) and absence of many intracranial midline structures. There are three types, the most severe being the alobar form, which shows no cleavage at all, with absence of the falx cerebri and third ventricle, fusion of the cerebral hemispheres and thalami, and a single large lateral ventricle. The semilobar form has variable cleavage with a partially formed falx, rudimentary third ventricle, and variable cleavage of the thalami, lateral ventricles and cerebral hemispheres. In the lobar type of holoprosencephaly, brain formation may be nearly normal, but the septum pellucidum is always absent, as in all forms. The falx, corpus callosum and ventricular system may be normal in the lobar type.

163
Q

A 9-month-old boy presents to the paediatric neurologist with general developmental delay, left-sided weakness from birth and tonic–clonic seizures. MRI performed under anaesthetic reveals a thin cleft containing CSF extending from the right lateral ventricle to the cortical surface of the right frontal lobe. The margins of the cleft are opposed and lined with heterotopic grey matter that shows polymicrogyria. Which disorder of cortical formation is described? [B4 Q35]

a. porencephaly
b. schizencephaly
c. holoprosencephaly
d. lissencephaly
e. hemimegalencephaly

A

b.schizencephaly

The development of the cerebral cortex takes place in three stages: cell proliferation, cell migration and cortical organization. Schizencephaly is a disorder of the final stage where there is a CSF-containing cleft lined with grey matter (which is often polymicrogyric) connecting the subarachnoid CSF space with the ventricular system. Differentiation can be made from the cyst or cavity of porencephaly, by the presence in schizencephaly of a lining of heterotopic grey matter, whereas a porencephalic cyst is lined by white matter. To result in schizencephaly, the insult must involve the entire thickness of the cerebral hemisphere during cortical organization, as in injuries due to prenatal infection, ischaemia or chromosomal abnormalities. Clinical presentation varies but often includes developmental delay, motor impairment and seizures.

164
Q

A severely hypoplastic cerebellar vermis in an enlarged bony posterior fossa, with associated hydrocephalus and communication of the fourth ventricle with a posterior midline CSF cyst, are features of which of the following posterior fossa malformations? [B4 Q65]

a. mega cisterna magna
b. Dandy–Walker malformation
c. Dandy–Walker variant
d. Arnold–Chiari malformation
e. Joubert’s syndrome

A

b. Dandy-Walker malformation

Classically, the Dandy–Walker malformation consists of partial or total absence of the cerebellar vermis, dilatation of the fourth ventricle into a large cystic mass, an enlarged posterior fossa, hydrocephalus (in 75% of cases) and torcular–lambdoid inversion (elevation of the torcular herophili above the lambdoid suture). The proposed aetiology is obstruction of CSF outflow at the foramina of Magendie and Luschka. The vermis abnormality is the key component in all forms of the Dandy–Walker complex. The variant is less severe with a better prognosis. Chiari malformations have the fundamental abnormality of an underdeveloped, small posterior fossa, in contrast to the Dandy Walker complex where it is normal or enlarged.

165
Q

A 27-year-old female presents with emotional lability and headaches. MRI of the brain demonstrates a well-defined mass in the pericallosal region. The mass is hyperintense on T1W images and demonstrates no enhancement with intravenous gadolinium. There is associated agenesis of the corpus callosum. What is the most likely diagnosis? [B4 Q86]
a. dermoid
b. lipoma
c. interhemispheric arachnoid cyst
d. epidermoid
e. lymphoma

A

Lipoma
Intracranial lipomas appear as well-circumscribed masses of fat density on CT with occasional rim calcification. On MR scan, characteristic appearances are of hyperintensity on T1W images, with chemical shift artefact or signal suppression on fat-saturated sequences. Approximately 30% of intracranial lipomas occur in the pericallosal region, and there is a high incidence of associated congenital anomalies, most commonly agenesis of the corpus callosum, but also encephalocele and cutaneous frontal lipomas. Interhemispheric arachnoid cysts may occur in association with agenesis of the corpus callosum, but they are of CSF density and, like epidermoid, appear hypointense on T1W images. Dermoids have signal characteristics following those of fat but are usually more heterogeneous and not associated with callosal anomalies. Lymphoma may involve the corpus callosum but appears iso or hypointense on T1W images.

166
Q

A 27-year-old woman presents to the Accident & Emergency Department with headaches. A CT scan of the head shows widely spaced lateral ventricles, dilatation of the trigones and occipital horns of lateral ventricles with an upward displacement of the dilated 3rd ventricle. The underlying abnormality in the brain is? [B5 Q16]
(a) Midline arachnoid cyst
(b) Agenesis of the corpus callosum
(c) Prominent cavum septum pellucidum
(d) Hydrocephalus
(e) Lobar holoprosencephaly

A

Agenesis of corpus callosum
This is associated with parallel, widely spaced lateral ventricles that may appear crescent shaped. There is dilatation of trigones and the occipital horn of lateral ventricles, along with a high riding 3rd ventricle. Callosal agenesis is associated with Dandy–Walker syndrome, Chiari malformations and fetal alcohol syndrome.

167
Q

A 30-year-old man presents with loss of sensation in his toes. MRI of the cervical spine shows cerebellar ectopia of 4 mm below the foramen magnum and syringomyelia of the cervical cord. The most likely diagnosis is? [B5 Q22]
(a) Chiari I malformation
(b) Chiari II malformation
(c) Chiari III malformation
(d) Chiari IV malformation
(e) Dandy–Walker malformation

A

Chiari I malformation
Chiari I malformation is characterised by cerebellar ectopia and is frequently an isolated hindbrain abnormality without supratentorial abnormalities. 20–30% of cases are associated with syringomyelia.

168
Q

A patient presents with recent onset neurological symptoms suspicious of an acute presentation of multiple sclerosis (MS). Which of the following anatomical sites of plaque involvement is least consistent with this? [B1 Q2]
A. Corpus callosum.
B. Spine involvement in the absence of brain involvement.
C. Cerebral cortex.
D. Symmetrical involvement of cerebral white matter.
E. Floor of the fourth ventricle.

A

Symmetrical involvement of cerebral white matter.
Symmetrical involvement of the cerebral hemispheres or cerebellar peduncles is unusual in MS and is occasionally seen in acute disseminated encephalomyelitis (ADEM). ADEM can mimic an acute presentation of MS both clinically and in terms of imaging findings. The monophasic nature of ADEM can be deduced both from the uniformity of lesional oedema and contrast enhancement in the acute phase. Lesions in ADEM resolve on follow-up, and although enhancing and non-enhancing lesions can coexist for a period, new lesions should not appear. MS plaques are classically seen in the periventricular and juxtacortical white matter. The other options in the question are all common plaque locations. Involvement of the corpus callosum is characteristic. Other common supratentorial sites include the white matter abutting the temporal horns and trigones of the lateral ventricles. Cortical lesions are less conspicuous on MRI than white matter lesions, but their detection is improved by the inclusion of a FLAIR sequence. Juxtacortical white matter lesions are highly suggestive of MS, as lesions are not commonly seen in this region in normal ageing. Twelve per cent of patients have lesions on MRI limited to the spine without brain involvement.

169
Q

A 34-year-old female presents with neurological symptoms suggestive of MS and is referred for an MRI of brain by the neurology team. Which of the following sequences is most useful for determining if there are plaques of differing ages (i.e. dissemination in time)? [B1 Q35]
A. FLAIR.
B. T2WI.
C. Pre- and post-contrast T1WI.
D. Proton density.
E. STIR.

A

Pre- and post-contrast T1WI.
MS plaques are generally most conspicuous on FLAIR sequences. However, if there are enhancing and non-enhancing plaques on T1WI post-contrast, this indicates plaques of differing ages, i.e. dissemination in time. While this is not diagnostic of MS (imaging alone never is), it is very suggestive. Proton density sequences are useful, but not quite as useful as FLAIR sequences, in showing periventricular lesions. T2WI sequences are superior to FLAIR sequences when it comes to detection of lesions in the posterior fossa and spinal cord. T2W STIR is probably superior to standard T2WI sequences in detecting spinal cord lesions but is not routinely used. STIR sequences or fat suppression are useful when imaging the optic nerves, as contrast between lesions and the surrounding orbital fat is increased.

170
Q

An immunocompetent 24-year-old patient presents with an acute history of right-sided limb weakness which has rapidly progressed. The patient now also has mixed sensory symptoms and poorly controlled seizures. Basic observations and initial blood results are normal. An MRI is carried out which shows a large mass-like lesion in the right posterior frontal lobe. There is mild mass effect associated with this lesion on the adjacent parenchyma, with mild compression of the right lateral ventricle. The lesion is also noted to involve the corpus callosum and cross the midline into the left cerebral hemisphere. This lesion is bright on FLAIR imaging. On post-contrast T1WI the lesion has peripheral rim enhancement. Vascular structures are noted to pass through the lesion. There is no evidence of necrosis or haemorrhage. On DWI, the b1000 sequence reveals increased signal in the lesion. Other high T2WI lesions are noted in the cerebellum, with a further lesion in the spinal cord. What is the most likely diagnosis based on this information? [B1 Q60]
A. MS.
B. Glioblastoma multiforme.
C. High-grade astrocytoma.
D. Lymphoma.
E. Abscess

A

MS.
The key feature in this clinical vignette is the identification of the mass lesion crossing the midline. Only a few lesions demonstrate this ability, and they are GBM, lymphoma, metastases, MS and other white matter disorders, lipoma, stroke, and shearing injuries. Abscess is thus excluded and would be unlikely given the clinical features. GBM is unlikely given the absence of necrosis and would be rare in a patient of this age group. High-grade astrocytomas do not involve the corpus callosum and cross the midline. Lymphoma exhibiting these features would be uncommon in immunocompetent patients, being more typically associated with HIV-related lymphoma. Tumefactive MS typically presents with an acute clinical history and frequently displays the imaging features described. The presence of vascular structures crossing the lesion also indicates this diagnosis, as these would be displaced by neoplastic lesions.

171
Q

Which is the preferred sequence to use when attempting to identify posterior fossa lesions on MRI in patients with multiple sclerosis? [B2 Q24]
a. T1-weighted spin-echo
b. T2-weighted spin-echo
c. FLAIR
d. Gradient-echo
e. Proton density

A

T2-weighted spin-echo
Multiple sclerotic plaques can be located anywhere in the central nervous system but typically they form at the junction of the cortex and white matter and periventricularly. FLAIR sequences are excellent at identifying lesions located in the cerebral hemispheres, but T2-weighted spin-echo sequences tend to be superior in detecting lesions located in the posterior fossa. With the extensive degree of variation seen in the posterior fossa structures, a combination of T1-weighted and T2-weighted sequences can be useful.

172
Q

Which of the following MR sequences is most sensitive for detecting subcortical and periventricular lesions in multiple sclerosis? [B4 Q38]
a. axial T1W
b. axial T2W
c. axial FLAIR
d. axial proton density
e. axial T1W post-gadolinium contrast

A

Axial FLAIR
The FLAIR MR sequence is a heavily T2-weighted inversion recovery sequence designed to nullify the signal from CSF. This highlights any T2 high-signal lesions that may be masked on standard T2W images by adjacent high signal of CSF. This is particularly beneficial in the periventricular region and corpus callosum, and studies have shown that FLAIR has higher accuracy than intermediate and T2W sequences in detecting supratentorial cortical, subcortical, and periventricular multiple sclerosis lesions. Some lesions can be best seen on standard T2W images, particularly in the posterior fossa, brain stem, and spinal cord.

173
Q

A 31-year-old woman presents to the neurologist with a transient episode of facial numbness. MRI shows several small areas of low signal on T1W and high signal on T2W images in the optic nerves and cerebellar peduncles. The FLAIR sequence demonstrates multiple, ovoid, high-signal areas in the corpus callosum and periventricular white matter. These imaging findings support which of the following diagnoses? [B4 Q81]
a. internal carotid artery dissection
b. multiple sclerosis
c. progressive multifocal leukoencephalopathy
d. acute disseminated encephalomyelitis
e. Wallerian degeneration

A

Multiple sclerosis
MRI is a crucial tool for supporting the clinical diagnosis of multiple sclerosis, an inflammatory autoimmune disease characterized by demyelination. The lesions appear high signal on T2W sequences (including FLAIR) and are commonly found in the periventricular white matter, often resembling Dawson’s fingers. The presence of these high-signal lesions in the corpus callosum and periventricular white matter, alongside the clinical symptoms, strongly indicates multiple sclerosis.

174
Q

A 35-year-old woman with spastic paraparesis presents with right facial paralysis. FLAIR images show multiple foci of hyperintense signal in the periventricular distribution, aligned at a right angle to the ventricles. Contrast-enhanced T1 images show multiple enhancing lesions in the cervical cord. What is the most likely diagnosis? [B5 Q36]
(a) Diffuse axonal injury
(b) Small vessel ischemic change
(c) Vasculitis
(d) Multiple sclerosis
(e) Neurosarcoidosis

A

Multiple sclerosis
MS typically presents with multiple white matter lesions that are hyperintense on T2 and FLAIR. These lesions align perpendicularly to the ventricles, and contrast enhancement may indicate active lesions.

175
Q

A 34-year-old woman presents with a seizure. MRI shows small multifocal frontal and parietal subcortical white matter T2WI hyperintensities, with areas of restricted diffusion in the right cerebral hemisphere on DWI. What is the most likely diagnosis? [B1 Q36]
A. Multiple sclerosis (MS).
B. SLE.
C. Small vessel ischaemia.
D. Susac syndrome.
E. Lyme disease.

A

SLE.
The presence of white matter hyperintensities and focal infarcts, combined with established neurological symptoms in a young woman, strongly suggests cerebral lupus. Imaging findings include small multifocal T2WI or FLAIR hyperintense white matter lesions and potentially enhancing lesions. SLE can cause a variety of neuropsychiatric symptoms due to vasculitis and thrombosis.

176
Q

A patient known to have AIDS presents with malaise and confusion. CT brain shows multiple cerebral hypoattenuating nodular lesions with surrounding oedema and enhancement post-contrast. Which condition can be confidently removed from the differential diagnosis? [B2 Q31]
a. Tuberculosis
b. Pyogenic abscesses
c. Progressive multifocal leukoencephalopathy
d. Lymphoma
e. Toxoplasmosis

A

Progressive multifocal leukoencephalopathy
PML is characterized by single or multiple hypoattenuating white matter lesions without oedema or mass effect in AIDS patients. Other conditions listed would present with enhancement and surrounding oedema. Toxoplasmosis is the most common CNS mass lesion in AIDS, but PML lesions do not exhibit enhancement and are typically non-edematous.

177
Q

Progressive multifocal leukoencephalopathy has a predilection for: [B3 Q46]
A. Cerebellum
B. Parietal lobe
C. Occipital lobe
D. Parieto-occipital region
E. Frontal lobe

A

D. Parieto-occipital region
PML has a predilection for the parieto-occipital region, often presenting with characteristic imaging findings in this area.

178
Q

A 39-year-old man known to have HIV and with clinically deteriorating dementia undergoes MRI of the brain with a provisional diagnosis of HIV encephalopathy. Images show abnormal high signal on T2W and FLAIR images affecting the white matter, caudate nucleus, and basal ganglia on a background of brain atrophy. Which additional MR characteristic of these lesions is likely to suggest progressive multifocal leukoencephalopathy over HIV encephalopathy? [B4 Q37]
a. frontal white matter preponderance
b. clustering around the basal ganglia
c. involvement of subcortical U fibres
d. areas of haemorrhagic necrosis
e. enhancement with intravenous gadolinium

A

Involvement of subcortical U fibres
This is characterised by a symmetrical, non-inflammatory demyelination of the basis pontis. The underlying cause is rapidly corrected hyponatraemia. MRI appearances are characteristic with a sparing of peripheral rim of tissue. The corticospinal tracts are generally spared as well.

179
Q

An eight-year-old boy with skin hyperpigmentation presents with deteriorating vision and hearing loss. CT shows large symmetric low-density lesions in the parietal-occipital white matter. T2-weighted MR imaging demonstrates confluent symmetric hyperintensity within the parietal-occipital white matter extending across the splenium of the corpus callosum. There is relative sparing of the frontal lobes. Follow-up MRI six months later shows cerebral atrophy and more extensive white matter change with involvement of the frontal lobes and cerebellum. At this point the patient had developed spastic quadriplegia. What is the most likely diagnosis? [B2 Q43]
a. Multiple sclerosis
b. Lymphomatoid granulomatosis
c. Acute disseminated encephalomyelitis
d. Adrenoleukodystrophy
e. Autoimmune vasculitis

A

Adrenoleukodystrophy (ALD)
ALD is an inherited metabolic disorder characterised by progressive demyelination of cerebral white matter. It is commonly X-linked recessive and boys present between the ages of 3 and 12 years with ataxia, deteriorating vision, hearing loss, altered behaviour and mental deterioration. It is associated with adrenal insufficiency (Addison’s disease). Predominantly, there is posterior white matter involvement with the disease advancing toward the frontal lobes and cerebellum. Imaging shows CT hypodensity, MR T1 hypointensity and T2 hyperintensity. Administration of contrast shows enhancement of the lateral margins of the lesions, indicating areas of active demyelination.

180
Q

An 18-month-old toddler presents with progressive gait disturbance, motor developmental delay and muscle weakness. Biochemical tests show abnormally low levels of arylsulphatase A enzyme in peripheral blood leukocytes and urine. MRI demonstrates symmetrical, confluent areas of high signal on T2W images affecting the periventricular and cerebellar white matter, corpus callosum and corticospinal tracts with sparing of the subcortical U fibres. The periventricular abnormality shows a striped ‘tigroid’ pattern. There is no enhancement with administration of intravenous gadolinium. What is the most likely condition? [B4 Q32]
a. Alexander’s disease
b. Canavan’s disease
c. acute disseminated encephalomyelitis
d. mucopolysaccharidosis
e. metachromatic leukodystrophy

A

Metachromatic leukodystrophy
HIV encephalopathy is a progressive subcortical dementia caused by direct infection of the central nervous system by HIV. It is therefore not an opportunistic infection and is seen in up to 60% of AIDS cases. The most striking feature is cerebral atrophy and diffuse myelin pallor, manifested as ill-defined, confluent areas of high signal in the white matter on T2W sequences or low attenuation on CT. Changes are usually bilateral but asymmetrical, with characteristic sparing of the grey matter. Progressive multifocal leukoencephalopathy occurs in only 2–4% of AIDS cases and is a reactivation of the JC (John Cunningham) virus, resulting in destruction of oligodendrocytes. The most common location is posterior whereas HIV encephalopathy is more commonly frontal; however, this is an unreliable discriminator. Progressive multifocal leukoencephalopathy characteristically involves the subcortical U fibres and also affects grey matter. It has a very poor prognosis, death occurring within 2–5 months.

181
Q

A chronic alcoholic was admitted with dehydration, acute confusion, and electrolyte imbalance. The patient was admitted and treated vigorously with intravenous fluids. The next day the patient went into a coma. MRI shows a round abnormality in the central area of the pons which returns low signal on T1 and high on T2. The most likely cause of the abnormality is? [B5 Q7]
(a) Acute pontine infarct
(b) Pontine haemorrhage
(c) Wilson’s disease
(d) Glioma
(e) Central pontine myelinolysis

A

Pontine myelinolysis
Also known as osmotic myelinolysis. This condition is seen in people with rapidly corrected hyponatraemia with intravenous fluids. The rapid correction of sodium releases myelinotoxic compounds resulting in destruction of myelin sheaths. This is particularly common in alcoholics.

182
Q

A 60-year-old man with a history of chronic alcoholism is admitted with confusion and dysarthria. MRI shows a 2 cm area of abnormality in the central pons which returns high T2 signal and low on T1. There is a rim of normal tissue around this lesion and prominence of the cerebellar folia. What is the most likely diagnosis? [B5 Q27]
(a) Multiple sclerosis
(b) Central pontine myelinosis
(c) Infarction
(d) Neoplasm
(e) Acute disseminated encephalomyelitis

A

Central pontine myelinosis
Dysmyelinating diseases (leukodystrophies) are a wide spectrum of inherited neurodegenerative disorders affecting myelin in the brain and peripheral nerves. Most fall into one of three categories: lysosomal storage diseases, peroxisomal disorders and diseases caused by mitochondrial dysfunction. The most common, metachromatic leukodystrophy, is an autosomal recessive disorder caused by a deficiency of the lysosomal enzyme arylsulphatase A, which leads to accumulation of sulphatides in tissues. It usually manifests as a late infantile subtype in children at 12–18 months of age and is characterized by motor signs of peripheral neuropathy followed by deterioration in intellect, speech and coordination, leading to death within a few years. On T2W MRI, symmetrical confluences of high signal intensity are seen in the periventricular white matter with sparing of the subcortical U fibres. Sparing of the perivascular white matter gives a striped or tigroid appearance to the periventricular area of abnormality, particularly well seen in the centrum semiovale. Other sites often affected are the corpus callosum, internal capsule and corticospinal tracts.

183
Q

A 14-year-old boy presents with a grossly abnormal gait, kyphoscoliosis, and upper limb tremors. MRI shows mild atrophy of the cerebellum and much more marked cervico-medullary junction thinning with decreased anteroposterior diameter of the upper cervical spinal cord, which returns normal signal. What is the most likely diagnosis? [B4 Q12]

a. Friedreich’s ataxia
b. Creutzfeldt–Jakob disease
c. olivopontocerebellar atrophy
d. Huntington’s disease
e. tuberous sclerosis

A

Friedreich’s Ataxia

Loss of myelinated fibres and gliosis in the posterior and lateral columns of the spinal cord are the histopathological hallmarks of Friedreich’s ataxia. On MRI, the predominant radiological finding is thinning of the cervical spinal cord and medulla, and there may be associated mild cerebellar atrophy. Conversely, in most other forms of ataxia such as early onset cerebellar ataxia and olivopontocerebellar atrophy, atrophy of the cerebellum and pons predominates with relative sparing of the cord. Although cerebellar atrophy may be seen in Friedreich’s ataxia, it is less pronounced than in these other diseases.

184
Q

A 58-year-old patient is found at home with a reduced GCS. CT brain reveals atrophy only. MRI brain reveals hyperintensity in the tegmentum (except for the red nucleus) and hypo-intensity of the superior colliculus on T2WI, as well as hyperintensity in the basal ganglia. What is the most likely cause? [B1 Q31]

A. Cocaine abuse.
B. Methanol poisoning.
C. Primary basal ganglia haemorrhage.
D. Wilson’s disease.
E. Carbon monoxide poisoning.

A

Wilson’s disease.

Hyperintensity in the tegmentum (except for the red nucleus) and hypointensity of the superior colliculus are described as the ‘face of the giant panda sign’ and are seen in axial T2WI sections of the midbrain in Wilson’s disease. A ‘double panda sign’ has also been described, with a second ‘panda cub face’ in the pons. Abnormal signal can also be seen in the basal ganglia and thalamus in Wilson’s disease (putamen most commonly). The signal abnormalities are due to copper deposition. Signal is generally reduced on T1WI sequences, although it may be increased due to the paramagnetic effects of copper and due to the hepatic component of Wilson’s disease (a portocaval shunt can produce this latter finding). Signal is generally increased on T2WI sequences, but it can be of mixed or reduced intensity. Similarly, carbon monoxide poisoning and methanol poisoning can cause increased or reduced signal on T1WI. Methanol poisoning typically causes abnormal signal in the putamen, with haemorrhagic necrosis being more typical, whereas carbon monoxide poisoning typically affects the globus pallidus. The latter would be expected to cause low attenuation in the basal ganglia on CT.

185
Q

A 56-year-old man with chronic alcohol dependence presents with progressive cognitive impairment, gait disturbance, and signs of interhemispheric disconnection. An MRI scan of brain demonstrates increased T2WI signal lesions without mass effect within the corpus callosum and dorsal part of the external capsule. Which of the following is the most likely diagnosis? [B1 Q58]

A. Wernicke’s encephalopathy.
B. Osmotic myelinolysis.
C. Marchifava–Bignami disease.
D. Alcohol withdrawal syndrome.
E. Chronic hepatic encephalopathy.

A

Marchifava–Bignami disease.

This is a rare complication of chronic alcohol consumption characterized by demyelination and necrosis of the corpus callosum, although other white matter tracts (such as the external capsule) may be involved. Bilateral and symmetrical T2WI hyperintensities within the thalami, periaqueductal grey, and mammillary bodies are seen in Wernicke’s encephalopathy. Osmotic myelinolysis is usually secondary to rapid changes in serum sodium and results in increased T2WI signal within the central pons. Extrapontine myelinolysis is rare. In alcohol withdrawal syndrome, patients present with seizures and delirium tremens. MRI may show volume loss in the temporal cortex and anterior hippocampus. Hepatic encephalopathy secondary to hepatocellular failure may produce T1WI hyperintensity within the caudate nucleus, globus pallidus, putamen, and anterior midbrain due to increased concentration of manganese.

186
Q

A 32-year-old man presents with schizophrenic-like psychosis and parkinsonian-type movement disorder. There is a family history of neuropsychiatric disturbance. An initial CT is requested which demonstrates heavy bilateral, symmetric calcifications within the globus pallidus, thalami, putamen, and cerebellum. There is no enhancement post contrast. Which of the following suggests a diagnosis of Fahr disease over pseudohypoparathyroidism? [B1 Q69]

A. Involvement of the globus pallidus.
B. Involvement of the thalami.
C. Involvement of the putamen.
D. Involvement of the cerebellum.
E. Normal calcium-phosphorus metabolism

A

Normal calcium-phosphorus metabolism.

Fahr disease is a rare degenerative neurological disorder characterized by extensive bilateral basal ganglia calcifications that can lead to progressive dystonia, parkinsonism, and neuropsychiatric manifestations. CT has higher diagnostic specificity for basal ganglia calcification over MRI. The distribution of calcifications is similar in endocrinological disorders such as hyperparathyroidism, hypoparathyroidism, and pseudohypoparathyroidism. The calcium-phosphorus metabolism is usually normal in Fahr disease. In pseudohypoparathyroidism the serum calcium is low with an appropriately high parathyroid hormone (PTH), due to PTH resistance.

187
Q

A 14-year-old boy presents with a progressive history of gait and speech disturbance. On both T1- and T2-weighted MR imaging, the globus pallidi are markedly hypointense except for a small central region of high signal intensity. The findings are more pronounced on T2-weighted imaging. What is the most likely diagnosis? [B2 Q15]

a. Leigh’s disease
b. Hallervorden–Spatz syndrome
c. Wilson disease
d. Mitochondrial encephalomyelopathy
e. Parkinson’s disease

A

Hallervorden–Spatz (HS) syndrome

The finding described on MRI is the ‘eye-of-the-tiger’ sign. This is closely associated with HS. HS is a progressive neurodegenerative metabolic disorder characterised by extrapyramidal and pyramidal signs. The condition (for which the pathophysiology is unclear) results in the accumulation of iron within the globus pallidi and brainstem nuclei. Two clinical entities exist: familial and sporadic. The familial (classic) form shows earlier onset and rapid progression. The sporadic (atypical) form is characterised by a later onset, often in teenage years, with slower progression. Although the ‘eye-of-the-tiger’ sign is closely associated with HS, it has been demonstrated in other rare extrapyramidal parkinsonian disorders including cortical-basal ganglionic degeneration, early-onset levodopa-unresponsive parkinsonism and progressive supranuclear palsy.

188
Q

A three-year-old girl undergoes a CT scan of her head following trauma. No acute pathology is demonstrated but there is basal ganglia calcification. Which of the following can be excluded as a cause of the calcification? [B2 Q27]

a. Down’s syndrome
b. Neurofibromatosis
c. Birth hypoxia
d. Wilson disease
e. Congenital rubella

A

Wilson disease

Causes of basal ganglia calcification include:

  • Physiological aging.
  • Infections/inflammatory: TORCH infection, TB, cysticercosis, measles, chickenpox, pertussis, Coxsackie B virus, AIDS, SLE.
  • Toxins: lead, carbon monoxide, birth anoxia/hypoxia, chemotherapy/radiotherapy, nephrotic syndrome.
  • Congenital: Cockayne’s, Fahr’s and Down’s syndromes, neurofibromatosis, tuberous sclerosis, methaemaglobinopathy.
  • Endocrine: hypothyroidism, hypoparathyroidism, pseudo-hypoparathyroidism, pseudo-pseudo-hypoparathyroidism, hyperparathyroidism.
  • Metabolic: Leigh disease, mitochondrial cytopathies.
  • Trauma: infarction
189
Q

A homeless adult male is admitted with a change in mental state and a metabolic abnormality. CT shows a focus of reduced attenuation in the pons. On MRI the focus is hypointense on T1 and hyperintense on T2. There is restricted diffusion on diffusion weighted imaging and there is no enhancement post-contrast. What is the most likely diagnosis? [B2 Q45]
a. Brainstem glioma
b. Metastasis
c. Infarction
d. Tuberculosis
e. Osmotic myelinolysis

A

e. Osmotic myelinolysis
Osmotic myelinolysis is also referred to as central pontine myelinolysis or osmotic demyelination syndrome. It is classically seen in an alcoholic, hyponatraemic patient in which rapid correction of sodium releases myelinotoxic compounds leading to destruction of myelin sheaths. The pons is the commonest site, although extra-pontine areas such as basal ganglia, cerebellar white matter and thalami may also be involved. Some patients completely recover but the six-month survival rate is only 5–10%.

190
Q

A 40-year-old unkempt male is admitted with disorientation and ataxia. FLAIR, T2 and diffusion-weighted MR imaging reveal abnormal high signal in both medial thalami, the hypothalamus and peri-aqueductal gray matter. There is atrophy of the right mamillary body. What is the most likely diagnosis? [B2 Q58]
a. Lymphoma
b. Viral encephalitis
c. Creutzfeldt–Jacob disease
d. Wernicke’s encephalopathy
e. Venous thrombosis

A

d. Wernicke’s encephalopathy
WE refers to an acute or subacute syndrome characterised by disorientation, gaze paralysis, ataxia and nystagmus. It invariably occurs in chronic alcoholics but it is caused by thiamine deficiency. Involvement of the medial thalami is characteristic and typically the mamillary bodies, peri-aqueductal grey matter and hypothalamus are also involved. In the acute stages of the disease, haemorrhage, necrosis and oedema are encountered, whereas in the latter stages these regions tend to atrophy, particularly the mamillary bodies.

191
Q

Which is the cause of low attenuation in the basal ganglia? [B3 Q12]
A. Wilson’s disease
B. Hypoparathyroidism
C. Pseudohypoparathyroidism
D. Hypothyroidism
E. Radiation therapy

A

A. Wilson’s disease
Other causes increase carbon monoxide poisoning, barbituate intoxication, hypoxia, hypoglycaemia and lacunar infarcts.

192
Q

Bilateral globus pallidus injury manifest radiologically as high signal on T2W MR sequences is indicative of poisoning by which of the following substances? [B4 Q76]
a. lead
b. methanol
c. carbon monoxide
d. carbon dioxide
e. mercury

A

c. carbon monoxide
Carbon monoxide poisoning results in irreversible formation of carboxyhaemoglobin in the blood, causing anoxic ischaemic encephalopathy. These changes are usually bilateral and affect the basal ganglia, most commonly the globus pallidus. Injury is demonstrated as high signal on T2W and FLAIR images and shows restricted diffusion on DWI. Areas less commonly affected acutely are the putamen (which is characteristically involved in methanol poisoning) and caudate nucleus. Involvement elsewhere can occur but is less common than basal ganglia changes. Delayed post-anoxic encephalopathy may develop several weeks after carbon monoxide poisoning, and MRI then shows further high T2 signal changes in the corpus callosum, subcortical U fibres, and internal and external capsules, with low T2 signal changes in the thalamus and putamen.

193
Q

A 73-year-old has been referred for assessment of cognitive decline. A CT brain reveals cerebral atrophy, and a dementia specialist refers her for PET-CT brain. Which of the following findings is most consistent with early Alzheimer’s disease? [B1 Q37]
A. Diffuse reduced activity.
B. Reduced activity in the precuneus and posterior cingulate gyrus.
C. Reduced activity in the frontotemporal regions.
D. Reduced activity in the caudate and lentiform nuclei.
E. Reduced activity bilaterally in the occipital cortex.

A

B. Reduced activity in the precuneus and posterior cingulate gyrus.
FDG-PET-CT has been shown to have a sensitivity and specificity of 93% for mild to moderate Alzheimer’s disease. The technique has been shown to provide important prognostic information so that a negative PET-CT scan is indicative of unlikely progression of cognitive impairment for a mean follow-up of 3 years in those patients who initially present with cognitive symptoms of dementia. The more specific findings on PET-CT in Alzheimer’s disease are early reduced activity in the precuneus/posterior cingulate gyrus and the superior, middle, and inferior temporal lobe gyrus, with relative sparing of the primary sensorimotor and visual cortex, and sparing of the striatum, thalamus, and cerebellum.

194
Q

A 62-year-old man presents with tremor and incontinence. Examination reveals bradykinesia and gait ataxia. He is also noted to have a reduced mini mental state examination (MMSE) and postural hypotension. He has an MRI scan of brain as part of the diagnostic workup. On the axial T2WI a cruciform hyperintensity in the pons is noted. Which of the following is the most likely diagnosis? [B1 Q57]
A. Parkinson’s disease.
B. Multisystem atrophy (MSA).
C. Progressive supranuclear palsy (PSP).
D. Cryptobasal degeneration.
E. Lewy body dementia

A

B. Multisystem atrophy (MSA).
T2WI cruciform hyperintensity or the ‘hot cross bun’ sign within the pons, is suggestive of MSA, although it is not specific, as it also occurs in spinocerebellar ataxia. Other MRI features include hyperintensity within the putamen. Patients with MSA typically have parkinsonian features poorly responsive to levodopa therapy and autonomic disturbance.

195
Q

A 52-year-old female presents with increasing short-term memory problems. CT and MR scans are normal. A perfusion scintigraphy study of the brain shows reduced perfusion in the posterior parietal regions bilaterally. What is the most likely diagnosis? [B4 Q21]
a. Alzheimer’s disease
b. vascular dementia
c. Lewy body dementia
d. frontotemporal dementia
e. Parkinson’s disease

A

Alzheimer’s disease
Alzheimer’s disease typically presents with atrophy and reduced perfusion to the mesotemporal regions bilaterally, but, in younger patients, it often presents with posterior parietal perfusion abnormalities. Vascular dementia shows patchy cortical and subcortical perfusion defects, with involvement of the cerebellum. Lewy body dementia is like Alzheimer’s disease but with less occipital sparing; Parkinson’s dementia is also similar but with less mesiotemporal change and more involvement of the visual cortex.

196
Q

A 22-year-old man presents with a 6-week history of progressive worsening confusion. Initial CT examination is normal. Following neurological review, the patient undergoes MRI, at which the only abnormality seen is high signal in the posterior portions of the thalami bilaterally on T2W and FLAIR images. What is the most likely condition? [B4 Q23]
a. acute disseminated encephalomyelitis
b. variant Creutzfeldt–Jakob disease
c. multiple sclerosis
d. metachromatic leukodystrophy
e. Rasmussen’s encephalitis

A

Variant Creutzfeldt-Jakob Disease
Variant Creutzfeldt–Jakob disease is a rare but important, transmissible, rapidly progressive spongiform encephalopathy and a cause of dementia and death in young patients. The transmissible agent is thought to be a prion protein (proteins devoid of nucleic acid), and the condition is causally linked to bovine spongiform encephalopathy. CT is typically normal, but symmetrical hyperintensity in the pulvinar nuclei of the posterior thalami (pulvinar sign) on T2W or FLAIR MR images has been described as a specific sign for the variant form. The classic form affects an older age group and is often genetically linked, whereas the variant form is sporadic. The pulvinar sign is seen only in the variant form, with the classic type typically showing similar hyperintense abnormalities located in the caudate nucleus and putamen.

197
Q

With ageing, iron accumulates in tissues of the brain and is seen as low signal on both T1W and T2W sequences at MRI. Which of the following locations for iron deposition is pathological in a 60-year-old man and may indicate onset of dementia? [B4 Q28]
a. putamen
b. globus pallidus
c. red nucleus
d. dentate nucleus
e. pars reticulata

A

Putamen
Non-haem brain iron is normally found within oligodendroglia and astrocytes, with smaller amounts in neurons and myelinated axons. Half is found within the mitochondria and 10% in the nuclei, and 40% represents a soluble fraction. The areas of maximum iron concentration in normal adults are found in the globus pallidus, red nucleus, pars reticulata of the substantia nigra and dentate nucleus of the cerebellum. The brain concentration is independent of general body stores, and the mechanism by which it crosses the blood–brain barrier is not fully understood. By the eighth decade, the concentration of iron may increase in the caudate nucleus and putamen to levels similar to those seen in the globus pallidus. Excessive or premature accumulation in these areas has been seen in various forms of senile dementia including Alzheimer’s disease. Iron deposition is identified on T1W and T2W sequences as signal hypointensity due to magnetic susceptibility.

198
Q

An elderly man undergoes CT for gradual onset of confusion and memory loss. A pattern of generalized global brain atrophy showing marked temporal lobe predominance with an increase in size of the hippocampal–choroidal fissure is seen. Coronal T2W images show atrophy of the hippocampus bilaterally. Which condition is most likely to result in such a pattern of volume loss? [B4 Q42]
a. Pick’s disease
b. Alzheimer’s disease
c. Parkinson’s disease
d. Lewy body dementia
e. multi-infarct dementia

A

Alzheimer’s disease
Alzheimer’s disease is a progressive neurodegenerative disorder and the most common cause of dementia in elderly people. Current thinking is that imaging can help in the early diagnosis by documenting or quantifying atrophy in certain regions of the brain such as the hippocampus and entorhinal cortex. However, conventional structural imaging has a relatively low sensitivity in Alzheimer’s disease and may be normal. Therefore, use is limited to identification of patterns of atrophy and the exclusion of other potential causes of dementia such as normal pressure hydrocephalus, vascular dementia or space-occupying lesion. Patterns of atrophy, when seen, may be relatively specific for different causes of dementia, with hippocampal atrophy a cardinal radiological sign of Alzheimer’s disease. Pick’s disease has a frontal and temporal preponderance and is part of the rarer group of diseases under the umbrella term ‘frontotemporal lobar degeneration’. Lewy body dementia typically affects the parietal and occipital lobes and the cerebellum, which are usually spared in Alzheimer’s disease. Multiinfarct dementia is vascular in origin and will show a patchy distribution of change. The different patterns of atrophy have also been recently demonstrated with 18 FDG PET.

199
Q

A 34-year-old liver transplant recipient presents to hospital with confusion and seizures. A CT brain reveals low attenuation in the deep and subcortical white matter of the occipital and parietal lobes bilaterally. There is no abnormal enhancement post IV contrast administration. As the reporting radiologist, you advise that the clinical team first: [B1 Q34]
A. measure blood glucose
B. measure serum alpha-feta protein
C. measure blood pressure
D. send coagulation screen
E. measure d-dimer

A

Measure blood pressure
The CT findings are consistent with PRES. This is a usually reversible neurological syndrome with a variety of presenting symptoms ranging from altered mental status to seizures, headache, and loss of vision. Common causes include hypertension, eclampsia and preeclampsia, immunosuppressive medications such as cyclosporine, various antineoplastic agents (including interferon), SLE, and various causes of renal failure. Hypertension is common in PRES, but may be mild and is not universally present, especially in the setting of immunosuppression. However, in the vignette given, hypertension is a possible cause and should be sought. Cyclosporin or tacrolimus might be causes; the former is thought to result in PRES both via a direct neurotoxic effect and by causing hypertension. The condition is not always reversible and may result in haemorrhagic infarcts. The classic MRI finding is of hyperintensity on FLAIR in the parieto-occipital and posterior frontal cortical and subcortical white matter. Less commonly the brainstem, basal ganglia, and cerebellum are involved. Atypical imaging appearances include contrast enhancement, haemorrhage, and restricted diffusion on MRI. Abnormalities can often be seen on CT, as described in the vignette.

200
Q

A 29-year-old female complains of increasing headache, followed by generalized seizure activity 24 hours after giving birth. She is noted to be hypertensive. MRI demonstrates bilateral parieto-occipital T2WI hyperintense cortical and subcortical lesions. ADC is elevated. Similar lesions are noted in the anterior watershed zones. What is the most likely diagnosis? [B1 Q70]
A. Hypotensive cerebral infarction.
B. Postpartum cerebral angiopathy.
C. Posterior reversible encephalopathy syndrome (PRES).
D. Progressive multifocal leukoencephalopathy (PML).
E. Gliomatosis cerebri

A

Posterior reversible encephalopathy syndrome (PRES)
This is a disorder of cerebrovascular autoregulation and is associated with eclampsia, which is defined clinically as seizure or coma with pregnancy-induced hypertension. There is a predilection for the posterior circulation and watershed zones, possibly due to its sparse vasomotor sympathetic innervation. T2WI hyperintense lesions within the posterior cortex and subcortical white matter, without diffusion restriction, are typical. Diffusion restriction/low ADC will be found in infarction. The diagnosis of postpartum cerebral angiopathy should be considered in normotensive postpartum women presenting with intracerebral haemorrhage. Reversible high T2WI signal abnormalities may also be seen in the cortex and subcortical white matter. Multifocal stenoses and beading of the intracranial vasculature are also a feature. PML presents with asymmetrical T2WI hyperintensity in the periventricular and subcortical white matter, with cortical sparing. It is seen in the immunocompromised. Gliomatosis cerebri is a diffusely infiltrating tumour involving two or more lobes and is frequently bilateral. Enlargement of the involved structures is seen. It can mimic brainstem PRES.

201
Q

A 40-year-old with headache, confusion, seizures, and visual disturbance has a CT showing low attenuation involving the cortex and subcortical white matter in both occipital lobes. Diffusion weighted MR shows no restriction. Which is the most likely diagnosis? [B3Q4]
A. Posterior reversible encephalopathy syndrome
B. Bilateral occipital infarction
C. CADASIL
D. Sinus venous thrombosis
E. Hypoxic brain injury

A

A
These are typical imaging findings but the abnormalities are not always reversible or posterior.

202
Q

A 36-year-old patient with known AIDS is admitted to your hospital with a right-sided hemiparesis. His wife gives a history of cognitive decline, headache, and lethargy for the past few weeks, but with an acute deterioration in the last 36 hours. His CD4 counts are between 200 and 500 (category B). An MRI is carried out which shows multiple ring-enhancing lesions in the brain. These lesions are high signal on T2WI/FLAIR but demonstrate a low intensity peripheral ring. Lesions are noted in the basal ganglia bilaterally. The cortical region and subcortical U-fibres are spared. A thallium SPECT is also carried out that shows abnormal uptake in the brain, which correlates with the areas identified on MRI. What is your primary diagnosis? [B1 Q9]
A. Nocardia abscesses.
B. Toxoplasmosis.
C. Progressive multifocal leukoencephalopathy.
D. Primary CNS lymphoma.
E. Cerebral cryptococcus infection

A

Toxoplasmosis.
This is the most common intracranial opportunistic infection in AIDS patients and the most common cause of intracranial mass lesions in this population, with lymphoma second and cryptococcus third. Differentiating toxoplasmosis from lymphoma has been the subject of much debate in the literature and ultimately there is no definitive means of differentiating them on imaging alone. As described toxoplasmosis causes multiple ring-enhancing lesions and often affects the basal ganglia. Lymphoma tends to involve the subependymal regions and can cause leptomeningeal enhancement. Lymphoma can cause encasement of the ventricle, which is not seen with toxoplasmosis. Toxoplasmosis tends to be higher intensity on T2WI/FLAIR than lymphoma. PET and thallium SPECT have also been reported to show abnormal uptake in lymphoma, but not toxoplasmosis, although exceptions exist. Response to anti-toxoplasma treatment is often used as a clinical differentiator. Cryptococcus classically causes dilatation of Virchow–Robin spaces. PML is a demyelinating condition secondary to JC virus of oligodendrocytes and is differentiated from other HIV-related demyelinating conditions by affecting the subcortical U fibres.

203
Q

A 13-year-old boy is brought to your paediatric hospital with a recent history of headache and high fever. The child is becoming progressively drowsy and demonstrates rigidity and tremor on neurological examination. An MRI is requested. Relevant past medical history on the request form is of recent travel to Asia, history of measles as a 6-year-old and recent viral infection. The MRI scan shows increased signal on T2WI and FLAIR sequences in the hippocampal regions of the temporal lobes. There is also increased FLAIR signal in the thalami and putamina bilaterally. Small foci of increased T1WI signal within these regions are felt to represent haemorrhage. What diagnosis would you place at the top of your differential list? [B1 Q13]
A. Herpes simplex type 1 encephalitis.
B. Herpes simplex type 2 encephalitis.
C. Japanese encephalitis.
D. Varicella Zoster encephalitis.
E. Subacute sclerosing panencephalitis (SSPE)

A

Japanese encephalitis.
Japanese encephalitis and herpes simplex virus (HSV) encephalitis both present with similar acute and rapidly progressive neurological symptoms. The key differentiator is involvement of the basal ganglia, which is typical in Japanese encephalitis but rare in HSV. Both commonly involve the hippocampi, this being the classical appearance of HSV encephalitis. HSV type 1 is the subtype that affects adult and older children. HSV type 2 causes neonatal and in utero infection. Herpes varicella zoster virus (VZV) infection can be seen in immunocompromised children, but in the immunocompetent population is more typically seen in elderly patients, often, but not exclusively, in the presence of cutaneous shingles. It causes a vasculitis, which can be seen angiographically and causes bilateral increased T2WI/FLAIR foci and gyriform enhancement in the distribution of the vasculitis. SSPE presents with a more protracted history of neurological decline.

204
Q

The husband of a 33-year-old woman takes her to the local accident and emergency department, stating that she is becoming acutely confused and is not her normal self. Her past medical history is unremarkable apart from a flu-type illness a few days earlier. Her GCS is 13 (E– 4, V– 3, M– 6). Initial non-contrast-enhanced CT of the brain shows low-density change within the left temporal lobe. MRI demonstrates abnormal low signal on T1 and high signal on T2 within the left temporal cortex. The insula is involved but there is sparing of the putamen. There is mild mass effect with partial effacement of the lateral ventricles. What is the most likely diagnosis? [B2 Q28]
a. Herpes simplex encephalitis
b. Low-grade glioma
c. Brain abscess
d. Post-viral leukoencephalopathy
e. Dural sinus thrombosis leading to cerebral infarction

A

Herpes simplex encephalitis (HSE)
There are two main types of herpes simplex virus (HSV) – HSV type I and HSV type II. Type I (oral herpes) tends to affect older infants, children, and adults whereas type II (genital herpes) is the usual cause of HSE in neonates. HSE is a necrotising meningoencephalitis which has a predilection for the limbic system (temporal lobes, insula, cingulated gyri). Characteristically, there are poorly defined areas of low attenuation in one or both temporal lobes/limbic system on unenhanced CT, low signal on T1 (gyral oedema) and high signal on T2. The T2 high signal typically spares the putamen and forms a sharply defined border. Changes may initially appear unilateral but contralateral disease invariably follows. This sequential bilaterality is characteristic of HSE. Haemorrhage is typically a late finding. Mortality ranges from 30% to 70% but is reduced with early antiviral therapy.

205
Q

A 33-year-old male with no significant past medical history presents with headache, drowsiness, and confusion. CT shows a hypodense lesion with a smooth regular wall centred over the left lentiform nucleus. There is surrounding oedema and mass effect with effacement of the ipsilateral Sylvian fissure. On T2-weighted MR imaging, the lesion is hyperintense and is surrounded by a hypointense rim and hyperintense oedema. There is peripheral enhancement post-contrast injection, and diffusion-weighted imaging demonstrates restricted diffusion within the lesion. What is the most likely diagnosis? [B2 Q46]
a. Glioblastoma multiforme
b. Pyogenic abscess
c. Toxoplasmosis
d. Lymphoma
e. Metastasis

A

Pyogenic abscess
The differential diagnosis for a solitary ring-enhancing lesion of the brain includes (‘MAGICAL DR’): Metastasis; Abscess; Glioma/Glioblastoma multiforme; Infarction; Contusion; AIDS (toxoplasmosis); Lymphoma (often AIDS-related); Demyelinating disease; Resolving haematoma/Radiation necrosis. Classically, abscesses are located at the corticomedullary junction in the frontal and temporal lobes. The most common causative organism is Streptococcus. The wall is generally smooth and regular with relative thinning of the medial wall secondary to a poorer blood supply from white matter (neoplastic lesions usually have a thick, nodular, irregular rim). In this scenario, the enhancing, T2-hypointense rim suggests abscess. Restricted diffusion is also highly suggestive of an abscess. Lymphoma may be hyperdense on CT due to a high nuclear-to-cytoplasmic ratio and typically shows solid homogeneous enhancement in immunocompetent patients.

206
Q

A four-month-old male undergoes investigation for microcephaly and hearing loss. Unenhanced CT brain shows several periventricular subependymal cysts and multiple coarse periventricular and parenchymal white matter calcifications. There is diffuse hypoplasia of the cerebellum. What is the most likely diagnosis? [B2 Q57]
a. Tuberous sclerosis
b. Sturge–Weber syndrome
c. Cytomegalovirus infection
d. Venous sinus thrombosis
e. Congenital rubella

A

Cytomegalovirus infection
This is the most common intrauterine infection and the leading cause of brain disease and hearing loss in children. Typical imaging findings include periventricular subependymal cysts representing focal areas of necrosis and glial reaction, periventricular post inflammatory calcifications, scattered calcifications in basal ganglia and brain parenchyma, microcephaly due to disturbance of cell proliferation and hypoplasia of the cerebellum. There may also be lissencephaly, cortical dysplasia, polymicrogyria and schizencephaly due to disturbed neuronal migration.

207
Q

In herpes simplex virus (HSV) type 1 encephalitis: [B3 Q30]
A. Early CT findings include hyper-density in one or both temporal lobes
B. Avid contrast enhancement in the temporal lobes is often present on C7
C. MR shows avid contrast enhancement in the temporal lobes
D. The basal ganglia are typically spared
E. Signal abnormalities are usually persistent even after drug treatment

A

D
Early CT findings may show medial temporal hypodensity. Areas poorly enhance on CT and MR. Signal abnormalities often decrease in response to treatment.

208
Q

Which is the most common location for toxoplasmosis? [B3 Q45]
A. Basal ganglia
B. Frontal lobe
C. Occipital lobe
D. Temporal lobe
E. Brainstem

A

A
75% of cases are in the basal ganglia.

209
Q

A 37-year-old man with AIDS presents with confusion, lethargy, and memory loss. CT of the brain demonstrates multiple supratentorial enhancing masses. Which imaging feature favours a diagnosis of toxoplasmosis rather than primary CNS lymphoma? [B4 Q3]
a. subependymal distribution
b. lesions hyperdense on unenhanced CT
c. lesion size > 3cm
d. hypo-vascularity on MR perfusion study
e. increased uptake of thallium-201 on SPECT

A

d
Hypo-vascularity on MR perfusion study
Toxoplasmosis is the most common cause of a cerebral mass lesion in patients with AIDS. Typical appearances are of multiple, hypoattenuating, 2 cm lesions with a predilection for the basal ganglia. Lymphoma is the second commonest mass lesion, with characteristic features including hyperdense lesions (though less frequently than in non-AIDS lymphoma) in a periventricular location with subependymal spread. Lesions in both conditions may show solid or ring enhancement. Haemorrhage is unusual in lymphoma but may be seen in toxoplasmosis.

210
Q

A 30-year-old woman presents acutely with seizures, fever, and headache, followed by rapid deterioration to coma. Emergency MRI shows asymmetrical swelling of the anterior temporal lobes on T1W images. T2W images reveal concordant asymmetrical but bilateral areas of high signal in the anterior temporal lobes, insular cortices and hippocampi. There is no enhancement following administration of intravenous gadolinium. What is the most likely condition? [B4 Q84]
a. lymphoma
b. HIV encephalitis
c. cytomegalovirus encephalitis
d. herpes simplex encephalitis
e. toxoplasmosis

A

d
Herpes simplex encephalitis
Herpes simplex virus is the most common cause of fatal endemic encephalitis, often leaving survivors with severe memory and personality problems. Both oral (type 1) and genital (type 2) strains may produce encephalitis with a multimodal distribution, affecting neonates (due to cross-infection with type 2 from the mother during birth), children and adults. Childhood and adult infection is caused by the type 1 virus and results in fulminant necrotizing encephalitis presenting with acute confusion and deteriorating rapidly to coma. Focal neurological deficits are seen in only 30% of cases. The virus asymmetrically affects the temporal lobes, insula, orbitofrontal region and cingulate gyrus, causing oedema. This is seen as high signal on T2W/FLAIR images, with DWI appearances variable depending on the presence of infarction. The putamen is characteristically spared, and the areas of encephalitis typically do not show enhancement on CT or MRI.

211
Q

A 20-year-old immigrant from South America presents with seizure. CT of the brain shows multiple cystic lesions, with some of them showing calcification. On MRI, there are multiple fluid-containing cysts in the brain, some of which contain small nodules. There is mild surrounding oedema. The most likely diagnosis is? [B5 Q26]
(a) Multiple brain abscesses
(b) Neurocysticercosis
(c) Tuberculomas
(d) Metastases
(e) Sarcoidosis

A

(b)
Neurocysticercosis
This is the most common parasitic infection of the brain and is particularly prevalent in South America, Asia and Africa. In the vesicular stage, CT and MRI show multiple, thin-walled cysts containing scolex with minimal surrounding oedema.

212
Q

A 7-year-old girl is brought into A&E with a history of headache of 4 hours’ duration, associated with neck stiffness. The parents noted a petechial rash and are concerned about the possibility of meningitis. There is no photophobia, and no other neurological signs are present. A&E have requested a CT scan to rule out raised intracranial pressure and to diagnose meningitis, whilst also commencing antibiotic therapy for presumed meningitis. The CT scan is normal. As it is a normal working day, A&E have also requested an MRI scan to diagnose meningitis. This was not carried out as the child was agitated. The following day the child develops a pyrexia and a right unilateral aspect to the headache, with decreased GCS. An MRI is carried out and demonstrates features of developing right temporal lobe empyema. Which of the following is an appropriate indication for imaging in the investigation of meningitis? [B1 Q12]
A. CT to rule out raised intracranial pressure.
B. CT to diagnose meningitis.
C. MRI to diagnose meningitis.
D. MRI due to deterioration in clinical course.
E. All of these

A

D
MRI due to deterioration in clinical course.
In the absence of clinical features of raised intracranial pressure a CT is not necessarily indicated prior to lumbar puncture to exclude raised intracranial pressure. CT is also insensitive at diagnosing meningitis, being normal in most uncomplicated cases. MRI is also negative in 50% of cases of uncomplicated meningitis. The value of imaging is in the detection of complications of meningitis, including hydrocephalus, abscess, cerebritis, venous thrombosis, and infarction.

213
Q

A male patient presents with severe back pain following surgery seven months previously. MRI shows extensive clumping of roots of the cauda equina to the right of the midline, suggesting an intrathecal mass. There is no enhancement post-gadolinium administration. What is the most likely diagnosis? [B2 Q52]
a. Intrathecal metastases
b. Nerve root sheath tumour
c. Arachnoiditis
d. Extradural haematoma
e. Prolapsed intervertebral disc

A

Arachnoiditis

Arachnoiditis is an intrathecal inflammatory reaction which usually results from iatrogenic injury. Previously, it was associated with the injection of contrast agents during myelography. Now, it is most seen following lumbar surgery. Intradural infections (TB, fungal, parasitic) are rare causes.
Radiologically, arachnoiditis appears as nerve root adhesion. Commonly the nerves are clumped together simulating a solid mass, although the nerves may also adhere to the dura, giving the appearance of an ‘empty thecal sac’. In severe cases, the thecal sac may become loculated and compartmentalised.
Clumped nerve roots suggest arachnoiditis and the lack of enhancement makes tumours unlikely.

214
Q

A 60-year-old man presents with back pain and progressive paraparesis. T1-weighted MR imaging shows loss of T9–T10 disc space with hypointense signal involving multiple contiguous vertebral bodies. Skip involvement of T3 and L1 vertebral bodies is evident. T2-weighted imaging shows a large paraspinal and prevertebral mass. An anterior epidural collection is seen compressing the cord. On post-contrast T1-weighted imaging the vertebral bodies show inhomogeneous enhancement and the paraspinal mass shows peripheral enhancement with central necrosis. What is the most likely cause? [B2 Q56]
a. Pyogenic infection
b. Lymphoma
c. Sarcoidosis
d. Tuberculosis
e. Metastases

A

d
Tuberculosis
The vertebral column is the most common site of osseous involvement of tuberculosis (TB), usually as the result of haematogenous spread from pulmonary involvement. The lower thoracic and upper lumbar vertebrae are the most frequently affected. It can be difficult to differentiate between pyogenic and TB spondylodiscitis on imaging but several features favour a diagnosis of TB: multilevel involvement/skip lesions, relative sparing of the disc spaces, large pre/paravertebral collections which are more likely to calcify, subligamentous spread and meningeal involvement.

215
Q

A 38-year-old male with Human Immunodeficiency Virus (HIV) stopped taking his retrovirals 6 months ago and now presents with confusion. CT brain shows non-enhancing hypodensities, with apparent dilated perivascular spaces, although these were not present on a CT brain from 2 years ago. What is the most likely cause? [B3 Q3]
A. Cryptococcus
B. Progressive multifocal leukoencephalopathy
C. Tuberculosis
D. CMV encephalitis
E. Toxoplasmosis

A

A
More commonly cryptococcus meningitis but cryptococcus or gelatinous pseudocysts reside in dilated perivascular spaces.

216
Q

Sequelae and complications of meningitis that may be identified on contrast-enhanced CT of the brain in the acute phase of the disease include all but which of the following? [B4 Q71]
a. venous sinus thrombosis
b. leptomeningeal enhancement
c. encephalomalacia
d. ring-enhancing mass lesion
e. infarction

A

Encephalomalacia
Diagnosis of meningitis is made by clinical examination and examination of CSF. Imaging is reserved for complications and to identify contraindications to lumbar puncture. The mechanism of spread is usually haematogenous, common organisms being Neisseria meningitidis (meningococcus) in young adults, Escherichia coli and Haemophilus influenzae. Complications that can be identified on CT include subdural empyema (sterile effusion can be seen with H. influenzae), venous sinus thrombosis, infarction, cerebritis and abscess formation. Leptomeningeal thickening and enhancement may be seen, but its absence does not exclude a diagnosis of uncomplicated meningitis. CT may also identify a potential source of infection such as otitis media, mastoiditis, sinusitis or orbital cellulitis.

217
Q

In brain imaging performed to characterize a chronic subdural collection, which feature is more likely to favour a diagnosis of subdural empyema over a sterile effusion? [B4 Q97]
a. isointense MR signal on T1W images
b. hyperintense MR signal on T2W images
c. sub-falcine herniation
d. restricted diffusion on MR DWI
e. low attenuation on CT

A

Restricted diffusion on MR DWI
Abscess cavities and empyemas are homogeneously hyperintense on MR DWI and low on ADC map due to the increased viscosity of the purulent material that they contain, resulting in restricted diffusion of water. Sterile effusions are hypointense on DWI and have an ADC appearance similar to that of CSF due to their lower viscosity and free macro-diffusion of water. Diffusion images can therefore be important when deciding whether to intervene surgically or conservatively manage subdural collections, as empyema requires timely surgical drainage. Contrast enhancement of the wall of the collection may be seen on CT and MRI, but its absence does not exclude the diagnosis of a purulent collection. Likewise, one may expect to see additional mass effect with an infected collection, but this too is not sensitive enough to exclude infection by its absence.

218
Q

A patient with a known diagnosis of a neurocutaneous syndrome is having a routine follow-up MRI scan. This patient is noted to have a history of right retinal calcifications. On MRI a retinal lesion is noted in the right eye, which is increased signal on T1WI. The scan shows a 2-cm lesion in the left lateral cerebellar hemisphere. This is predominantly low on T1WI and high on T2WI, but has a peripheral nodular area, which has a central signal void and demonstrates enhancement on T1WI. A further small enhancing lesion is noted in the cervical spinal cord, with an associated syrinx. The appearances are unchanged from previous imaging. Based on these findings, which neurocutaneous syndrome does this patient have? [B1 Q7]
A. Neurofibromatosis type 1 (NF-1).
B. Neurofibromatosis type 2 (NF-2).
C. Tuberous sclerosis.
D. Sturge Weber syndrome.
E. Von Hippel–Lindau syndrome.

A

Von Hippel–Lindau syndrome.
This syndrome is characterized by retinal lesions variously described as being retinal angiomas, retinal haemangiomas, or retinal hamartomas. These lesions (Lindau tumours) cause retinal calcification, although this is also seen in NF-1 and tuberous sclerosis (TS) from other causes. The findings describe the classical appearance of a cerebellar haemangioblastoma. These lesions can also be entirely cystic or solid in a minority of cases. A signal void and lack of dural enhancement help differentiate this from other infratentorial masses in adults. Twenty per cent of haemangioblastomas are associated with Von Hippel–Lindau syndrome and 80% of Von Hippel–Lindau syndrome patients have this tumour. The spinal finding also indicates a further haemangioblastoma in the cord. Unlike in the posterior fossa, these seldom have flow voids, but are associated with the development of a syrinx.

219
Q

A 42-year-old male patient of African-Caribbean ethnicity presents to the neurologists with a history of facial paraesthesia and slight weakness, headache, lethargy, and neck stiffness. He also has a history of mild dyspnoea. An MRI is requested, which shows left frontal leptomeningeal thickening, with an apparent dural mass noted along the falx. There is increased T2WI/FLAIR signal in the cortex adjacent to this area of meningeal disease. There is also thickening, and enhancement noted of the right trigeminal and facial nerves on post-contrast T1WI. Further enhancement is also noted along the parasellar region on the left. Multiple non-enhancing high T2WI/FLAIR periventricular white matter lesions are noted. Abnormality is noted in the skull vault, where there is an expanded diploic space over the right occipital region, with associated apparent thinning of the outer table, although this is hard to define on MRI. Due to concern with regard to multifocal meningioma the patient is started on steroids to reduce the mass effect from the frontal lesion and referred to neurosurgery. A CT is non-contributory. An MRI is repeated, which shows only mild meningeal thickening. What is the most likely diagnosis? [B1 Q56]
A. NF-2.
B. Neurosarcoid.
C. Neurotoxoplasmosis.
D. Tuberous sclerosis.
E. Multiple sclerosis

A

Neurosarcoid.
Whilst primary neurosarcoid is rare, it is not uncommon for the systemic disease to present via its neurological manifestations. The findings of neurosarcoid are non-specific, with the common manifestations mirroring those described in this patient. Leptomeningeal thickening and enhancement are classical. Involvement of the perivascular spaces adjacent to this can cause increased signal in the adjacent cortex. This appearance can mimic meningioma. Multifocal meningiomas and nerve sheath schwannomas are features of NF-2, but this would not be expected to resolve with steroid therapy. The features described are not typical of TS. The meningeal disease would make MS unlikely, even given the periventricular high-signal lesions, which are the most common intraparenchymal finding in neuro-sarcoid. Sarcoid involving the skull table is not directly related to the neurosarcoid and is more commonly seen with other manifestations of musculoskeletal sarcoid. Toxoplasmosis, in the absence of immunocompromise, is unlikely. Overall, the response to steroid, along with the wide variation of CNS disease described, are the strongest indicators for sarcoid. Whilst neurosarcoid does typically respond well to steroid therapy, it has a high recurrence rate.

220
Q

A 16-year-old female is referred for MRI after presenting with an increasing number of cutaneous neurofibromata. As a child she had been noted to have a cafe-au-lait spot on her back. What MRI finding would confirm the diagnosis of NF-1? [B1 Q67] {will repeat again in the orbit section}
A. Multiple hyperintense white matter foci on T2WI.
B. Bilateral vestibular schwannomas.
C. Meningioma.
D. Optic nerve glioma.
E. Multiple ependymomas.

A

Optic nerve glioma.
NF-1 is an autosomal dominant neurocutaneous disorder. The gene locus is located on 17q11.2. Diagnosis of NF-1 requires two or more of the following: six or more cafe-au-lait spots, two or more neurofibromas (or one plexiform neurofibroma), axillary/inguinal freckling, optic nerve glioma, sphenoid wing dysplasia or a first-degree relative with NF-1. The classic imaging appearance is multiple focal areas of white matter and deep gray matter signal abnormality. Other potential findings include intracranial stenoses and moyamoya type proliferation. NF-2 characteristically presents with multiple intracranial schwannomas, meningiomas and ependymomas (MISME). Bilateral vestibular schwannomas are diagnostic of NF-2.

221
Q

Which of the following neurological imaging findings would offer a clear differential diagnosis between tuberous sclerosis and neurofibromatosis Type I? [B2 Q32]
a. Basal ganglia calcification
b. Optic nerve glioma
c. Cerebral astrocytoma
d. Enhancing heterotopic grey matter
e. Multiple foci of hyperintensity on T2-weighted imaging

A

Enhancing heterotopic grey matter
All of the other features may be found in either condition. Giant cell astrocytomas located in the region of the foramen of Monro in tuberous sclerosis may degenerate into high-grade astrocytomas. Cortical tubers show as multiple nodules which are hyperintense on T2/FLAIR imaging. CNS hamartomas (occurring in up to 75–90% of NF1) also display these characteristics they are often termed ‘unidentified bright objects.’

222
Q

A three-year-old girl undergoes further investigation for refractory seizures. Contrast-enhanced T1-weighted imaging shows diffuse pial enhancement of variable thickness over the parieto-occipital region of the right cerebral hemisphere. There is atrophy of the underlying cerebrum and the right choroid plexus is enlarged. Several hypointense foci are seen within the gyri and adjacent white matter. There is also bilateral well-defined orbital choroidal enhancement. T2-weighted imaging demonstrates prominent superficial cortical veins. What is the most likely diagnosis? [B2 Q54]
a. Klippel–Trenaunay syndrome
b. Sturge–Weber syndrome
c. Wyburn–Mason syndrome
d. Neurofibromatosis
e. Tuberous sclerosis

A

Sturge–Weber syndrome
Sturge–Weber syndrome is a congenital disease characterised by capillary venous angiomas involving the face (port-wine stain, usually ophthalmic division of trigeminal nerve), choroid of the eye and leptomeninges. Clinical manifestations include focal seizures (80% in the first year of life), developmental delay, hemiparesis and homonymous hemianopia. Seizures typically become refractory to medication. Leptomeningeal angiomas are confined to the pia mater and occur primarily within the parieto-occipital region. There is cortical hemiatrophy beneath the angioma due to local anoxia and usually after the age of two years there is cortical calcification manifesting as low signal intensity on T1 post-contrast images. Other findings include enlargement of the ipsilateral choroid plexus, dilatation of the transparenchymal veins that communicate between the superficial and deep cerebral venous systems, and orbital choroidal haemangiomas.

223
Q

A 35-year-old female presents with worsening headache and facial droop. Her chest radiograph is abnormal. Non-contrast-enhanced CT brain is unremarkable. MRI demonstrates leptomeningeal thickening and enhancement, particularly around the basal cisterns. Enhancement extends along the right seventh cranial nerve and along the optic nerves. There are several small superficial parenchymal enhancing lesions found bordering the basal cisterns. What is the most likely diagnosis? [B2 Q55]
a. Sarcoidosis
b. Tuberculosis meningitis
c. Carcinomatous meningitis
d. Lymphoma
e. Behcet syndrome

A

Sarcoidosis
Neurologic manifestations of sarcoidosis occur in 5% of patients. In over 80% of established cases, the chest radiograph is abnormal. Sarcoidosis commonly involves the leptomeninges, especially the basal meninges and midline structures, including the hypothalamus and optic chiasm. The facial nerve is the most commonly involved cranial nerve clinically, while the optic nerves are often affected radiologically. Dural thickening may also be present, along with enhancing granulomas in the superficial parenchyma.

224
Q

A combination of subependymal nodules, giant cell astrocytomas, white matter lesions, and retinal phakomatoses suggests: [B3 Q25]
A. Tuberous sclerosis
B. NF I
C. NF II
D. Sturge-Weber syndrome
E. Von Hippel-Lindau

A

A. Tuberous sclerosis
These findings are characteristic of tuberous sclerosis.

225
Q

Which is the most common imaging finding in neurosarcoidosis? [B3 Q29]
A. Leptomeningeal contrast enhancement
B. Hyperintense white matter T2 lesion
C. Grey matter lesions enhancing on MR
D. Involvement of the hypothalamus
E. Focal epidural masses

A

A. Leptomeningeal contrast enhancement
This is a common finding; however, other features can also be present in neurosarcoidosis.

226
Q

A patient with a genetic condition and known bilateral renal cysts undergoes MRI brain showing a cystic lesion with mural nodule. This finding is associated with: [B3 Q31]
A. VHL
B. Tuberous sclerosis
C. Sturge-Weber
D. Hereditary Hemorrhagic Telangiectasia (HHT)
E. Neurofibromatosis

A

A. VHL
Approximately one-third of hemangioblastomas are associated with cystic lesions and mural nodules.

227
Q

Tram track gyriform calcification is most likely to be seen in: [B3 Q32]
A. NF I
B. Tuberous sclerosis
C. Von Hippel-Lindau
D. HHT
E. Sturge-Weber

A

E. Sturge-Weber
This condition is associated with tram track calcifications, cortical atrophy, and choroid plexus enlargement.

228
Q

A 20-year-old male patient has an MR scan of his spine for investigation of back pain. He has low IQ and multiple skin patches. The MR scan shows bilateral branching tubular paraspinal masses, with widening of the intervertebral foramina and posterior scalloping of the vertebral bodies. A sharply angulated kyphosis is present at the thoracolumbar junction. What is the most likely diagnosis? [B4 Q63]
a. neurofibromatosis type 1
b. neurofibromatosis type 2
c. tuberous sclerosis
d. Marfan’s syndrome
e. Ehlers–Danlos syndrome

A

Neurofibromatosis type 1
This condition is characterized by plexiform neurofibromas and vertebral abnormalities such as posterior scalloping and dural ectasia.

229
Q

A 30-year-old man with a bimalar rash and learning difficulties was shown to have bilateral renal angiomyolipomas. The most likely diagnosis is? [B5 Q6]
(a) Tuberous sclerosis
(b) Peutz–Jeghers syndrome
(c) Sturge–Weber syndrome
(d) Neurofibromatosis
(e) Fibrous dysplasia

A

Tuberous sclerosis
The classical triad includes adenoma sebaceum, mental retardation, and seizures, along with bilateral renal angiomyolipomas.

230
Q

A 15-year-old boy presents with a history of epilepsy and visual loss. A CT scan shows ‘tram track’ gyriform cortical calcifications in the right parieto-occipital lobe. MRI shows cortical atrophy in the region of calcifications. Post-gadolinium T1 demonstrates focally enhancing leptomeninges and enlarged ipsilateral choroid plexus in the occipital horn. What is the most likely underlying condition? [B5 Q9]
(a) Tuberous sclerosis
(b) von Hippel–Lindau
(c) Klippel–Trenaunay syndrome
(d) Sturge–Weber syndrome
(e) Neurofibromatosis

A

Sturge–Weber syndrome
Characterized by meningeal angiomatosis and associated findings such as cortical atrophy and calcifications.

231
Q

A 9-year-old boy with inguinal freckling presents with visual problems and epilepsy. MRI shows homogeneous enhancement of bilaterally enlarged optic nerves. What is the most likely diagnosis? [B5 Q29]
(a) Neurofibromatosis type 1
(b) Neurofibromatosis type 2
(c) Tuberous sclerosis
(d) von Hippel–Lindau disease
(e) Sturge–Weber syndrome

A

Neurofibromatosis type 1
This condition includes optic nerve gliomas and other characteristic lesions like café au lait spots and freckling.

232
Q

A 21-year-old boy with neurofibromatosis type 1 complains of visual difficulties. MRI shows abnormal enlargement of the optic chiasm and intense and homogeneous enhancement with gadolinium. The abnormality extends into the left optic tract. What is the most likely diagnosis? [B5 Q33]
(a) Craniopharyngioma
(b) Lymphoma
(c) Neurosarcoidosis
(d) Chiasmal glioma
(e) Tuberculosis

A

Chiasmal glioma
This condition is associated with NF1 and typically presents with optic chiasm enlargement.

233
Q

A 70-year-old woman presents with a sudden onset of right-sided hemiplegia and expressive dysphasia. She is otherwise well and has normal blood pressure on examination. Emergency CT shows a small subcortical acute haemorrhage in the inferior posterior left frontal lobe. Elsewhere, throughout the brain, there are smaller acute and subacute haemorrhages in the subcortex and a background of lacunar infarction. There is also marked brain atrophy more than that expected for the patient’s age. Changes consistent with diffuse leukoencephalopathy are also seen. What is the most likely underlying pathological condition? [B4 Q1]
a. malignant hypertension
b. acute disseminated encephalomyelitis
c. neurosarcoidosis
d. neuroamyloidosis
e. haemorrhagic metastases

A

Neuroamyloidosis
Cerebral amyloid angiopathy is characterized by deposition of b-amyloid protein in cortical, subcortical and leptomeningeal vessels. It usually occurs in sporadic form and increases in frequency and severity with increasing age. The condition produces a wide variety of clinical symptoms and varied imaging appearances. Many cases are asymptomatic but progressive neurological symptomatology and cognitive decline may be a feature. Chronic haemorrhage in a distinctive distribution, or catastrophic acute intracerebral/subarachnoid haemorrhage, may also occur. Amyloid angiopathy should be strongly considered in elderly normotensive patients with spontaneous intracranial haemorrhage, particularly when associated with leukoencephalopathy and atrophy related to small-vessel cerebrovascular disease. Definitive diagnosis is usually only made postmortem, with a presumptive diagnosis made on clinical presentation and imaging findings.

234
Q

A 32-year-old man of African descent presents to the neurologist with a several-month history of headache followed by rapid onset of bilateral facial weakness and blurring of vision. An MRI shows thickening of the basal leptomeninges, which enhance with intravenous gadolinium. Abnormal areas are identified in the pituitary gland, and optic and facial nerves, which all return low signal on T1W images and high signal on FLAIR images. What is the most likely diagnosis? [B4 Q15]
a. multiple sclerosis
b. tuberculous meningitis
c. neuroamyloidosis
d. neurosarcoidosis
e. lymphoma

A

Neurosarcoidosis
Neurosarcoidosis is seen in up to 8% of cases of sarcoidosis and has a tendency to involve the base of the brain. Cranial nerve involvement occurs frequently and nerve palsies are a common presenting feature. Bilateral facial nerve palsies, particularly in young adults, are most commonly due to neurosarcoidosis. The most common imaging manifestation is diffuse basomeningeal thickening, which typically shows enhancement with intravenous gadolinium. Also frequently seen are meningeal nodules and deep white matter lesions with high T2 signal. Multiple or solitary parenchymal masses may also occur, which can have a ring-like appearance and may therefore mimic primary or metastatic neoplasia.

235
Q

Following a large postpartum haemorrhage, a 25-year-old woman develops a severe headache and sudden visual field defect. What is the most likely diagnosis? [B4 Q66]
a. intracerebral haemorrhage
b. reversible posterior leukoencephalopathy
c. subarachnoid haemorrhage
d. Sheehan’s syndrome
e. vertebral artery dissection

A

Sheehan’s Syndrome
Many of the acute neurological conditions of pregnancy occur with rising blood pressure. Sheehan’s syndrome results from haemorrhage-induced hypotension causing pituitary infarction. Early on, this appears as an enlarged homogeneous pituitary with low T1 signal, high T2 signal and post-contrast ring enhancement. Later, there is an empty sella. Clinical manifestations include visual field loss, headache, ophthalmoplegia and pituitary dysfunction (diabetes insipidus).
Reversible posterior leukoencephalopathy produces cortical blindness, headaches, confusion and seizures. Those affected are often taking immunosuppressant treatment. Imaging features can be identical to eclampsia, peripartum cerebral angiopathy and hypertensive encephalopathy, but with a posterior predominance. On CT, there is low attenuation change. On MRI, there is high signal on T2W/FLAIR images. ADC maps can differentiate between likely reversible vasogenic oedema (high signal on ADC map showing unrestricted diffusion) and cytotoxic oedema (low signal due to restricted diffusion), which is more likely to progress to infarct.
Microangiopathic haemolytic anaemias, such as thrombotic thrombocytopenic purpura and haemolytic uraemic syndrome, give widespread ischaemia/infarction and haemorrhagic transformation. There is no increased risk in pregnancy of vasculitis such as systemic lupus erythematosus, Takayasu’s syndrome or Moyamoya syndrome. Arteriovenous malformation is no more likely to bleed in pregnancy, but there is an increased risk with arterial aneurysms. Haemorrhage, sepsis and pulmonary embolism cause hypotension that can cause watershed infarction as well as Sheehan’s syndrome.

236
Q

Regional cerebral blood flow imaging is required to localize an epileptic focus. Of the following radiopharmaceuticals, which is the most appropriate for this purpose? [B4 Q83]
a. 99mTc-labelled MAA
b. 99mTc-labelled DTPA
c. 99mTc-labelled pertechnetate
d. 99mTc-labelled glucoheptonate
e. 99mTc-labelled ECD

A

99m Tc-labelled ECD
Conventional radionuclide brain scans are not indicated when CT or MRI can be used. Brain accumulation of radiotracer occurs at sites of blood–brain barrier disruption and increased vascularity such as cerebral metastases, meningioma and high-grade glioma. Reagents used are 99m Tc-labelled pertechnetate, 99m Tc-labelled DTPA, and, when it was available, 99m Tc-labelled glucoheptonate. Radionuclide imaging is also able to provide regional blood flow mapping, which is used to localize epileptic foci, map cerebrovascular disease, investigate dementia, assess treatments and confirm brain death. 99m Tc-labelled HMPAO is the most used radiopharmaceutical overall. It is lipophilic and therefore crosses the blood–brain barrier, and in doing so is distributed in proportion to cerebral blood flow. 99m Tc-labelled ECD is another regional blood flow imaging tracer and can therefore also be used in the localization of epileptic foci. 18 FDG PET tracers are distributed through the brain according to metabolic activity.

237
Q

The Wada test is performed before surgical treatment for medically refractory epilepsy. It involves intra-arterial injections of sodium amobarbital with 99mTc-labelled HMPAO followed by SPECT imaging to map the distribution of the barbiturate. What is the main indication for this test? [B4 Q94]
a. chemical ablation of epileptic foci
b. localization of speech and memory centres
c. mapping symmetry of brain vascular distribution
d. identification of interhemispheric collateral circulation
e. evaluation of cerebrovascular reserve

A

Localization of speech and memory centres
SPECT of the central nervous system allows assessment of the collateral circulation during balloon occlusion, mapping the extent and distribution of vasospasm associated with subarachnoid haemorrhage, evaluation of cerebrovascular reserve prior to carotid endarterectomy, identification of seizure foci and guiding of stereotactic brain biopsy in tumour recurrence. The Wada test is used to predict the probability of memory and speech complications in patients before they undergo anterior temporal lobectomy with amygdalohippocampectomy, a surgical treatment for medically refractory epilepsy. The test involves injection of radiopharmaceutical and short-acting barbiturate (usually sodium amobarbital) into each of the carotid arteries in turn. Loss of memory or speech following one of the injections indicates that these centres lie in the ipsilateral temporal lobe. The results of the Wada test can be negated by vessels crossing the midline, and the presence of such collateral circulation may not be readily apparent without SPECT.

238
Q

A 2-year-old boy presents with involuntary saccadic eye movements (opsoclonus) and myoclonus of the trunk and limbs. What are the most likely findings on MRI of the brain? [B4 Q100]
a. normal appearances
b. cystic lesion in the posterior fossa with enhancing mural nodule
c. cerebellar atrophy
d. central symmetrical lesions in the pons of high signal intensity on T2W images
e. caudally displaced brain stem and fourth ventricle with tonsillar herniation

A

Normal appearances
Opsoclonus–myoclonus syndrome is characterized by opsoclonus in combination with myoclonus of the trunk, limbs or head. It may be idiopathic or occur as a paraneoplastic syndrome, when it may follow a relapsing–remitting course. In adults, it is most commonly associated with breast and squamous cell lung carcinoma, while in children it is associated with neuroblastoma. The syndrome usually precedes diagnosis of the underlying malignancy, and in children should prompt investigation to identify an underlying neuroblastoma. MIBG whole-body scintigraphy may be helpful in identifying occult disease if conventional imaging is negative. MR scan of the brain is usually normal.

239
Q

Which one of the following orbital pathologies typically arises from the intraconal compartment? [B1 Q18]
A. Cavernous haemangioma.
B. Adenocystic carcinoma.
C. Rhabdomyosarcoma.
D. Dermoid.
E. Orbital pseudotumour.

A

Cavernous haemangioma.
The orbital compartments are split up into extraconal, conal, intraconal, and globe. The extraconal compartment consists of fat, lacrimal gland, and bony orbit. Pathology in this region includes infection, neurofibroma, adenocarcinoma, mucoepidermoid and adenoid cystic carcinoma, neoplasia of bone, and lymphoma.
Conal pathology (muscle) includes rhabdomyosarcoma, thyroid eye disease, and idiopathic orbital inflammation (pseudotumour).
The intraconal compartment consists of fat, lymph nodes, vessels, nerves, and the optic nerve sheath complex. Pathology in this region includes venolymphatic malformation, haemangioma, arteriovenous malformation, optic nerve meningioma/glioma, and lymphoma.
Pathology in the globe includes retinoblastoma, metastasis, and melanoma.

240
Q

A 25-year-old man presents with painless progressive proptosis. CT shows a soft tissue lesion with microcalcifications in the left orbit. On MRI, T1 axial and coronal images show a soft tissue mass isointense to adjacent muscles in the extraconal plane while the T2 images show the hyperintense septated lesion. The lesion avidly enhances with gadolinium. What is the most likely diagnosis? [B5 Q19]
(a) Cavernous haemangioma
(b) Orbital pseudotumour
(c) Neurofibroma
(d) Dermoid cyst
(e) Haemangiopericytoma

A

Cavernous haemangioma
This is the most common orbital mass in adults and is the most common vascular malformation of the orbit. CT demonstrates the location of the lesion and microcalcifications. Remodelling of the bone may be seen. MRI features are typical as given.

241
Q

A 54-year-old lady with a history of exophthalmos undergoes a CT scan of orbits for assessment. There is spindle-shaped enlargement of extra-orbital musculature that does not involve the tendinous insertions. Which pair of muscles is most likely to be involved? [B1 Q20]
A. Superior and inferior.
B. Superior and medial.
C. Superior and lateral.
D. Inferior and medial.
E. Inferior and lateral.
F. Medial and lateral.

A

Inferior and medial.
The question refers to Graves ophthalmopathy, where enlargement of the extraocular muscles is observed, with sparing of the tendinous insertions. This is in distinction to idiopathic orbital inflammatory syndrome, where the muscle swelling typically involves the tendinous insertions.
In Graves ophthalmopathy, the extra-orbital muscles are involved with decreasing frequency as follows: inferior, medial, superior, and lateral recti.

242
Q

A 7-year-old boy presents with rapid onset right proptosis. CT shows an extraconal mass in the right orbit with irregular margins. There is evidence of intraconal and intracranial extension. On MRI, the lesion is of decreased signal on T1WI, increased signal on T2WI, and shows relatively uniform enhancement. What is the most likely diagnosis? [B1 Q21]
A. Retinoblastoma.
B. Non-Hodgkin lymphoma.
C. Capillary haemangioma.
D. Ruptured dermoid cyst.
E. Rhabdomyosarcoma.

A

Rhabdomyosarcoma.
The clinical and radiological features described are typical of rhabdomyosarcoma. It usually presents in the first decade of life with a rapidly growing mass causing proptosis. The globe is often distorted and displaced, but rarely invaded. The lesion is aggressive and there may be extension into the sinuses or intracranial compartment, and bony destruction can occur.
NHL is usually found in older adults, but it can occur in older children or adolescents. Unlike rhabdomyosarcoma, NHL commonly causes lacrimal gland involvement, may be hypointense on T2WI, and encases rather than distorts the globe.
Capillary haemangioma typically presents in the first few months of life. They have a proliferative phase for approximately 12 months following diagnosis and then are stable, before gradually involuting over a period of years. These lesions show rapid and persistent enhancement on CT and MRI. On unenhanced MRI, it is typically iso- to hyperintense relative to muscle on T1WI and moderately hyperintense on T2WI, with flow voids at the periphery of or within the tumour.
A ruptured dermoid cyst can mimic a rhabdomyosarcoma both clinically and radiologically, but one might expect to see calcification or fatty or cystic components in the native lesion. Retinoblastoma is an intraorbital lesion.

243
Q

A 10-year-old girl presents with gradual onset proptosis in the left eye and is noted to have restricted left eye movements. An MRI scan is performed, and this demonstrates a poorly circumscribed lobulated lesion that involves both the intraconal and extraconal compartments. The lesion has mixed signal intensity on T1WI, with areas of increased and decreased signal. It is mostly of increased signal on T2WI and there are areas containing a fluid-fluid level. Post contrast, there is heterogenous enhancement. What is the most likely diagnosis? [B1 Q22]
A. Venous-lymphatic malformation.
B. Orbital varix.
C. Cavernous haemangioma.
D. Capillary haemangioma.
E. Haemangiopericytoma.

A

Venous-lymphatic malformation.
Venous-lymphatic malformation (previously known as a lymphangioma) typically has a presentation in childhood. MRI is the modality of choice for diagnosis, as fluid–fluid levels are almost pathognomic of this condition and these represent areas of haemorrhage of varying ages. The MRI findings are as those described and there are typically no large feeding vessels or flow voids. The lesion is predominantly extraconal but can insinuate into the intraconal compartment.
Capillary haemangioma also presents in childhood but may be apparent clinically and on MRI there are not typically fluid–fluid levels, but there may be flow voids within the lesion from prominent vessels. Also, enhancement is usually more rapid and persistent in capillary haemangioma than in venous-lymphatic malformation. Orbital varices usually present in young adults and on imaging have the appearance of dilated, tortuous vessels.
Cavernous haemangioma typically presents in adulthood. They are usually well-circumscribed and most often occur in the lateral aspect of the retrobulbar intraconal space. On CT, they are slightly hyperattenuating prior to contrast and may contain phleboliths. On MRI, they are isointense to muscle on T1WI and hyperintense on T2WI. They show progressive enhancement on multiphase dynamic imaging.
Haemangiopericytomas are rare vascular tumours that are more often seen in adults than children. Large lesions may show bony erosion and they do not typically have fluid–fluid levels on CT or MRI.

244
Q

A 20-year-old female with a history of neurofibromatosis presents with reduced visual acuity in the right eye. She subsequently has CT and MR imaging of the orbits to assess for a tumour relating to the right optic nerve. Which of the following findings on imaging would be more suggestive of the presence of an optic nerve glioma, rather than a meningioma arising from the optic nerve sheath? [B1 Q23]
A. Presence of the ‘tram-track’ sign.
B. Presence of optic canal widening.
C. Presence of marked intense tumour enhancement.
D. Presence of calcification.
E. Presence of bony hyperostosis

A

Presence of optic canal widening.
The presence of a widened optic nerve canal occurs in up to 90% of cases of optic nerve glioma. While it can also occur in meningioma, it is more common in glioma and some cases of meningioma may even have a narrowed canal secondary to bony hyperostosis.
The ‘tram-track’ sign is typically associated with meningioma and refers to the more avidly enhancing meningioma surrounding the non-enhancing optic nerve on axial CT and MR imaging of the orbit. Although both meningioma and glioma enhance following intravenous contrast, it is meningioma that is more typically associated with marked intense enhancement.
Calcification and bony hyperostosis are features associated with meningioma. Calcification is rare in gliomas, unless they have previously undergone radiotherapy.

245
Q

A 16-year-old female is referred for MRI after presenting with an increasing number of cutaneous neurofibromata. As a child she had been noted to have a cafe-au-lait spot on her back. What MRI finding would confirm the diagnosis of NF-1? [B1 Q67]
A. Multiple hyperintense white matter foci on T2WI.
B. Bilateral vestibular schwannomas.
C. Meningioma.
D. Optic nerve glioma.
E. Multiple ependymomas.

A

Optic nerve glioma.
NF-1 is an autosomal dominant neurocutaneous disorder. The gene locus is located on 17q11.2. Diagnosis of NF-1 requires two or more of the following: six or more cafe-au-lait spots, two or more neurofibromas (or one plexiform neurofibroma), axillary/inguinal freckling, optic nerve glioma, sphenoid wing dysplasia or a first-degree relative with NF-1. The classic imaging appearance is multiple focal areas of white matter and deep gray matter signal abnormality. Other potential findings include intracranial stenoses and moyamoya type proliferation.
NF-2 characteristically presents with multiple intracranial schwannomas, meningiomas and ependymomas (MISME). Bilateral vestibular schwannomas is diagnostic of NF-2.

246
Q

A 50-year-old woman presents with visual loss. Examination reveals retinal detachment and an ocular lesion. On MRI, the lesion is hyperintense on T1 and hypointense on T2 relative to the vitreous. The lesion enhances post-gadolinium injection. The most likely diagnosis is: [B2 Q6]
a. Metastases from breast cancer
b. Metastases from lung cancer
c. Choroidal haemangioma
d. Malignant melanoma
e. Vitreous lymphoma

A

Malignant melanoma
This is the most common primary intraocular neoplasm in adult Caucasians. The typical age range is 50–70 years old. They are almost always unilateral and located in the choroid, although ciliary body and iris melanomas are not uncommon.
Presentation can be with retinal detachment, vitreous haemorrhage, astigmatism or glaucoma. MRI shows a sharply circumscribed hyperintense lesion on T1, due to the paramagnetic properties of melanin, and hypointense relative to the vitreous body on T2.

247
Q

A 35-year-old previously well female consults an ophthalmologist with a history of progressive loss of visual acuity over several months. Retinal examination reveals papilloedema. Unenhanced CT shows tubular thickening of the optic nerve associated with dense calcifications. Post-contrast injection shows a non-enhancing optic nerve surrounded by a markedly enhancing soft-tissue mass. The remainder of the brain is normal. The most likely diagnosis is: [B2 Q7]
a. Optic nerve glioma
b. Perioptic meningioma
c. Sarcoidosis
d. Lymphoma
e. Multiple sclerosis

A

Perioptic meningioma

248
Q

A middle-aged female presents with unilateral proptosis. CT of the orbits reveals an intraconal mass with involvement of the lateral rectus muscle to its point of tendinous insertion. The lesion enhances post-contrast injection. MRI shows a mass which is hypointense to fat on T2. What is the most likely diagnosis?
a. Thyroid opthalmopathy
b. Lymphoma
c. Cavernous haemangioma
d. Capillary haemangioma
e. Pseudotumour

A

Pseudotumour
Pseudotumour is the commonest cause of an intra-orbital mass lesion in adults. It is an idiopathic inflammatory condition and about 10% of cases occur in association with autoimmune conditions such as retroperitoneal fibrosis.
It usually presents with unilateral painful opthalmoplegia. It can be acute (more common) or chronic. The former has a more favourable prognosis as it is responsive to steroids. In contrast, the chronic type frequently requires chemotherapy and radiotherapy. On MRI, pseudotumour is hypointense to fat on T2, whereas true tumours are hyperintense.
Grave’s disease is the most common cause of uni/bilateral proptosis in adults (85% bilateral, 15% unilateral). However, involvement of extra-ocular muscles tends to maximally affect the midportion with relative sparing of the tendinous insertions. This gives rise to the so-called ‘Coke-bottle’ sign.

249
Q

A 65-year-old woman is investigated for enophthalmos and headache. She is cachetic, anaemic and you suspect a metastatic process. CT head demonstrates an infiltrative retrobulbar mass. What is the most likely site of primary disease? [B2 Q33]
a. Breast
b. Lung
c. Renal
d. Melanoma
e. Ovarian

A

Breast
Most retrobulbar metastases are extraconal (outside the muscle cone). Neuroblastoma and Ewing’s sarcoma are the most common in children and produce smooth extraconal masses related to the posterior lateral wall of the orbit. In adults, an infiltrative retrobulbar mass and enophthalmos is characteristic of scirrhous carcinoma of the breast (invasive ductal carcinoma). Enophthalmia is also considered to be one of the earliest signs of metastatic breast cancer.

250
Q

A three-week-old girl is investigated for a right orbital mass. CT shows a diffuse, poorly marginated mass in the superior temporal quadrant of the orbit. It is separate from the globe and the mass shows diffuse enhancement post-contrast injection. On MRI, the mass is hypointense to fat on T1 but hyperintense on T2. On both sequences, low-signal curvilinear foci are seen within it. What is the most likely diagnosis? [B2 Q42]
a. Lymphangioma
b. Rhabdomyosarcoma
c. Cavernous haemangioma
d. Retinoblastoma
e. Capillary haemangioma

A

Capillary haemangioma
Capillary haemangiomas are the most common vascular orbital masses in children. They typically present at birth or shortly after. CT shows a diffuse, usually poorly marginated mass which shows diffuse contrast enhancement. On MRI the mass is hypointense to fat on T1, and on T2 it is hyperintense to muscle and fat but hypointense compared to fluid. Curvilinear flow voids representing blood vessels are typical within the mass. They tend to grow rapidly over the first six months of life, stop growing during the second year and then slowly involute over subsequent years.

251
Q

A 60-year-old woman with mild proptosis undergoes a CT which shows enlargement of the bellies of the medial, and to a lesser extent, lateral rectus muscle. Tendons appear normal. What is the likely diagnosis? [B3 Q27]
A. Idiopathic orbital myositis
B. Carotid cavernous fistulae
C. Retro-orbital metastases
D. Graves ophthalmopathy
E. Plexiform neurofibroma of optic nerve

A

D
Sparing of muscle tendons is typical. Most cases are bilateral.

252
Q

Which of the following is an extraconal extra-orbital lesion, rather than an extraconal intra-orbital lesion? [B3 Q47]
A. Squamous cell carcinoma of the sinus
B. Teratoma
C. Dermoid cyst
D. Capillary haemangioma
E. Lymphangioma

A

A
Other causes are lymphoma, adenocarcinoma, adenoid cystic carcinoma, mucoceles and paranasal sinusitis.

253
Q

A 25-year-old man presents with unilateral proptosis, chemosis, reduced visual acuity and a bruit over his right orbit. Gadolinium-enhanced MRI of the orbits shows abnormal contrast enhancement of the right periorbital soft tissues and extraocular muscles. The superior ophthalmic vein is also dilated. What is the most likely diagnosis? [B5 Q35]
(a) Carotid-cavernous fistula
(b) Graves’ disease
(c) Orbital pseudo-tumour
(d) Optic nerve glioma
(e) Cavernous haemangioma orbit

A

Carotid-cavernous fistula
Important imaging findings include: extraocular muscle enlargement, proptosis, chemosis and dilatation of ipsilateral cavernous sinus and superior ophthalmic vein.
Graves’ disease affects older ages and bilateral involvement is typical. An orbital pseudotumour is a non-specific inflammation involving any area of orbit. Orbital bruit and a dilated superior ophthalmic vein are not usually seen.

254
Q

A 42-year-old woman presents with bilateral proptosis. CT of the orbits shows increased bulk of the rectus muscles and a dilated superior ophthalmic vein. What is the most likely diagnosis? [B5 Q48]
(a) Graves’ disease
(b) Orbital cellulitis with myositis
(c) Orbital pseudotumour
(d) Sarcoidosis
(e) Non-Hodgkin’s lymphoma

A

Graves’ disease
Graves’ ophthalmopathy (thyroid eye disease) features bilateral exophthalmos with bilateral enlargement of the extraocular muscles. The condition particularly affects the inferior, medial and superior rectus muscles.

255
Q

A 35-year-old man presents with tinnitus and hearing loss in the right ear. Investigations include an MRI of the internal auditory meati. This demonstrates an expansile lesion in the right petrous apex, without bone destruction. The lesion is of increased signal on T1WI, increased signal on T2WI, and non-enhancing. What is the most likely diagnosis? [B1 Q19]
A. Cholesteatoma.
B. Petrous apex cephalocele.
C. Mucocele.
D. Petrous apicitis.
E. Cholesterol granuloma.

A

Cholesterol granuloma.
The findings described are typical of cholesterol granuloma. A critical distinction is between this and a petrous carotid artery aneurysm, which may show similar features of increased T1WI signal. However, the presence of flow void, lesion centred on the carotid canal, and additional complex areas of signal due to blood products of varying ages might be expected in an aneurysm.
A petrous apex cephalocele is an area of fluid signal (hypointense T1WI, hyperintense T2WI) adjacent to the petrous apex that is in communication with Meckel’s cave.
A mucocele of the petrous apex may cause benign bony expansion and is typically of decreased signal on T1WI and increased signal on T2WI and is non-enhancing.
A cholesteatoma may have similar T1WI and T2WI signal characteristics, but these entities may be differentiated by DWI. On DWI, cholesteatoma is typically of increased signal, whereas a mucocele is of decreased signal.
Petrous apicitis is an aggressive process that typically enhances with gadolinium and is secondary to infection. There may be an accompanying history of infection, middle ear disease, diabetes, or immunocompromise.

256
Q

A 45-year-old man has a severe head injury and is noted to have a left facial nerve palsy. Following stabilization, a subsequent HRCT scan of the temporal bone is performed. This demonstrates a fracture of the left temporal bone, involving the course of the left facial nerve. Which orientation of fracture and which segment of the facial nerve are most likely to be involved? [B1 Q24]
A. Transverse/internal auditory canal.
B. Longitudinal/internal auditory canal.
C. Transverse/labyrinthine.
D. Longitudinal/labyrinthine.
E. Transverse/mastoid.
F. Longitudinal/mastoid.

A

Transverse/labyrinthine.
Transverse temporal bone fractures are more commonly associated with facial nerve paralysis (approximately up to 50%) than longitudinal temporal bone fractures (approximately up to 20%). The labyrinthine segment is the most likely segment of the facial nerve to be associated with facial paralysis.

257
Q

A 54-year-old man presents with hearing loss in the left ear, which is of the sensori-neural type. He undergoes an MRI scan of the internal auditory meati and subsequently a full MRI scan of brain. A large extra-axial mass lesion is identified at the left cerebellopontine angle. Which of the following features on MR imaging will be most helpful in indicating that this is probably a large vestibular schwannoma, rather than a meningioma? [B1 Q74]
A. Enhancing dura adjacent to the mass.
B. Erosion of the adjacent porus acousticus.
C. Tumour within the adjacent internal auditory meatus.
D. Intense enhancement within the mass.
E. Hyperostosis in the adjacent petrous temporal ridge.

A

Erosion of the adjacent porus acousticus.
A large meningioma at the cerebellopontine angle can easily grow into the adjacent internal auditory meatus (IAM), but typically does so without causing any enlargement or erosion of this structure. Most vestibular schwannomas arise from the inferior vestibular nerve within the IAM and as they grow they smoothly erode the posterior edge of the porus acousticus.
Although dural enhancement has been described adjacent to vestibular schwannomas, an enhancing ‘dural tail’ would be a finding more commonly associated with a lesion arising from the dura, typically a meningioma.
Bony hyperostosis is a finding that is associated with meningioma. Both meningiomas and vestibular schwannomas typically show avid enhancement with intravenous gadolinium.

258
Q

A 45-year-old woman undergoes investigation for conductive hearing loss. History reveals several previous ear infections. Direct visualisation with an otoscope shows a mass behind an intact tympanic membrane. Coronal CT imaging demonstrates a soft-tissue mass located between the lateral attic wall and the head of the malleus. There is blunting of the scutum. The mass does not enhance post-contrast. What is the most likely diagnosis?
a. Chronic otitis media
b. Cholesterol granuloma
c. Cholesteatoma
d. Rhabdomyosarcoma
e. Squamous cell carcinoma

A

Cholesteatoma
A cholesteatoma consists of a sac lined with stratified squamous epithelium and filled with keratin – essentially ‘skin growing in the wrong place’.
They can be acquired (98%) or congenital (2%). Most acquired cholesteatomas arise in the superior portion of the tympanic membrane (pars flaccida) and extend into Prussak’s space where they can cause medial displacement of the head of the malleus and erosion of the bony scutum.
The characteristic imaging feature of a cholesteatoma is bone erosion associated with a non-enhancing soft-tissue mass.
Complications can be intratemporal and intracranial:
Intratemporal: ossicular destruction, facial nerve paralysis, labyrinthine fistula, complete hearing loss, automastoidectomy.
Intracranial: meningitis, sinus thrombosis, abscess, CSF rhinorrhea.

259
Q

Which of the following is the most common cause of pulsatile tinnitus? [B3 Q40]
A. Glomus tumour
B. Cholesterol granuloma
C. Dehiscent jugular bulb
D. Carotid artery dissection
E. Meningioma

A

A
Glomus tumour is the most common cause of pulsatile tinnitus.

260
Q

A 24-year-old male patient presents following a head injury with GCS of 13. There is bruising over the right temporal region. A CT scan shows no intracranial haemorrhage but does identify a longitudinal fracture through the petrous temporal bone. What complication should be considered? [B4 Q49]
a. sensorineural hearing loss
b. conductive hearing loss
c. vertigo
d. carotid artery injury
e. sigmoid sinus injury

A

Conductive hearing loss
Longitudinal fractures of the temporal bone represent 75% of temporal bone fractures and run parallel to the axis of the petrous pyramid. They may cause dislocation of the auditory ossicles, usually the incus, causing a conductive deafness. Sensorineural hearing loss is associated with transverse fractures of the temporal bone, as is vertigo. Facial nerve palsy is seen in both fracture types, but is less common in longitudinal fractures, where it frequently recovers spontaneously. Carotid artery and major sinus injuries are not directly associated with petrous temporal fractures.

261
Q

A 30-year-old female presents with left-sided hearing loss and facial nerve palsy. CT shows a solid mass in the left middle ear behind an intact tympanic membrane, occupying an enlarged attic and eroding the scutum and middle-ear ossicles. What is the most likely diagnosis? [B4 Q56]
a. Bell’s palsy
b. cholesteatoma
c. malignant otitis externa
d. acoustic neuroma
e. squamous cell carcinoma

A

Cholesteatoma
Cholesteatoma is an abnormal collection of keratinized debris arising from an ingrowth of stratified squamous epithelium and occurs in primary (2%) or acquired (98%) types. The acquired type can be further subdivided, with the most common being a primary, acquired, epidermoid-type lesion of the pars flaccida, located in the attic of the middle ear. Cholesteatomas are benign lesions but cause bone erosion, including the auditory ossicles, resulting in conductive hearing loss. Local extension may compress the geniculate ganglion in the facial canal.

262
Q

A 49-year-old presents with a history of painless discharge, hearing loss and fullness in the left ear. CT shows a soft tissue mass in the middle ear with intact jugular fossa. Coronal reconstructions show erosion of the epitympanic ossicular chain with intact scutum. MRI shows that the soft tissue mass is hypointense on T1 and intermediate signal on T2 with no enhancement with gadolinium. The most likely diagnosis is? [B5 Q14]
(a) Secondary cholesteatoma
(b) Chronic otitis media
(c) Granulation tissue
(d) Squamous cell carcinoma
(e) Acute otitis media

A

Secondary cholesteatoma
Definitive diagnosis of cholesteatoma requires CT evidence of bone erosion, either of the ossicular chain or the walls of tympanic cavity. Pars tensa cholesteatoma typically spares the scutum and results in lateral displacement of the head of the malleus and the incus. Granulation tissue shows high signal on T2 and enhances with gadolinium. Squamous cell carcinoma also shows contrast enhancement.

263
Q

A 45-year-old man presents with deafness and left ear discharge. CT of the petrous and mastoids shows a soft tissue mass in the attic with erosion of the scutum. No contrast enhancement is seen. What is the most likely diagnosis? [B5 Q45]
(a) Glomus tympanicum
(b) Pars tensa cholesteatoma
(c) Pars flaccida cholesteatoma
(d) Cholesterol granuloma
(e) Congenital cholesteatoma

A

Pars flaccida cholesteatoma
This typically causes erosion of the scutum, ossicles or the lateral epitympanic wall. Pars flaccida is a small superior portion of the tympanic membrane. Scutum erosion is common and three-quarters of cases may have erosion of the ear ossicles.

264
Q

A 52-year-old woman presents with hearing loss in the right ear. Examination reveals a non-pulsating, bluish discoloration of the ear drum. CT shows a smooth expansile mass lesion in the middle ear which bulges the tympanic membrane laterally. On MRI, the lesion returns high signal on T1 and T2 sequences. What is the most likely diagnosis? [B5 Q46]
(a) Cholesterol granuloma
(b) Chronic otitis media
(c) Glomus jugulare
(d) Cholesteatoma
(e) Haemorrhagic otitis media

A

Cholesterol granuloma
This is caused by recurrent haemorrhage into the middle ear cavity forming an inflammatory mass of granulation tissue. Diagnosis is made by demonstration of bony expansion on CT and high signal on T1 and T2.

265
Q

A 27-year-old woman with facial skin lesions presents with chronic hearing loss in the right ear. CT shows expansion of the petrous and mastoid bones with a ‘ground-glass’ appearance. What is the most likely diagnosis? [B5 Q47]
(a) Osteopetrosis
(b) Fibrous dysplasia
(c) Otosclerosis
(d) Paget’s disease
(e) Giant cell tumour

A

Fibrous dysplasia
CT appearances are typical showing increased volume of bone with ‘ground-glass’ appearance.

266
Q

A 50-year-old male presents with a history of intermittent epistaxis, nasal obstruction, and frontal headache. He undergoes a CT of the sinuses that demonstrates an iso-dense soft-tissue mass filling the right maxillary antrum with extension through the infundibulum into the nasal cavity. There is associated bony remodelling of the infundibulum. On MRI, the mass is isointense to muscle on T1WI and T2WI and demonstrates a convoluted cerebriform pattern on enhanced T1WI. The remainder of the sinuses are unremarkable. What is the diagnosis? [B1 Q39]
A. Juvenile angiofibroma.
B. Inverted papilloma.
C. Antrochoanal polyp.
D. Invasive fungal sinusitis.
E. Nasal carcinoma.

A

Inverted papilloma
Inverted papillomas are uncommon benign epithelial neoplasms with significant malignant potential. They arise from the lateral nasal wall or maxillary sinus. They are most seen in 40–70-year-olds with a male to female ratio of 2–4:1. CT demonstrates a soft-tissue mass centred in the middle meatus associated with bone remodelling. Stippled calcification is seen in 20% of cases. On MRI, it is isointense on T1WI and iso/hypointense on T2WI. Heterogeneous enhancement is seen in 50% of cases. A convoluted cerebriform pattern on T2WI or enhanced T1WI is typical of inverted papillomas. Juvenile angiofibromas are seen in younger patients and are in the posterior nasal cavity. They demonstrate intense enhancement and flow voids on MRI. Antrochoanal polyps are homogenously hyperintense on T2WI. Invasive fungal sinusitis is primarily seen in immunosuppressed patients. They are frequently bilateral. CT findings include complete opacification of sinuses by hyperdense mass, erosion or remodelling of sinuses, and intra-sinus calcification. They are hypointense on T2WI. Nasal carcinomas cause bone erosion/destruction rather than remodelling.

267
Q

A 45-year-old female patient presents with recurrent frontal sinusitis following functional endoscopic sinus surgery (FESS). Which of the following CT findings is not commonly associated with postoperative frontal recess stenosis? [B1 Q40]
A. Inadequate removal of the agger nasi and frontal recess cells.
B. Retained superior portion of the uncinate process.
C. Medialization of middle turbinate.
D. Osteoneogenesis due to chronic inflammation or mucosal stripping.
E. Scarring or inflammatory mucosal thickening.

A

Medialization of middle turbinate.
FESS is the treatment of choice for patients with medically refractory sinusitis. Obstruction of the frontal recess is the main cause for medically refractory frontal sinusitis. The frontal recess is considered a difficult area to treat with FESS and it is also prone to re-stenosis. All of the above options are commonly associated with postoperative frontal recess stenosis except medialization of middle turbinate. Medialization (‘Bolgerization’) is performed to treat a lateralized middle turbinate, which is a well-recognized cause of frontal recess stenosis.

268
Q

A 52-year-old man with known chronic myeloid leukaemia complains of left-sided facial pain. Plain radiographs show a poorly defined lytic lesion centred over the left maxilla. Further imaging with CT and MR demonstrates an enhancing, homogeneous mass with infiltrative margins, which returns intermediate signal on T1 and T2 sequences. The most likely diagnosis is: [B2 Q17]
a. Granulocytic sarcoma
b. Lymphoma
c. Osteomyelitis
d. Myeloma
e. Neuroblastoma

A

Granulocytic sarcoma
Granulocytic sarcoma is a rare complication of acute and chronic myeloid leukaemia (AML & CML), occurring in approximately 3% of patients. It is a soft-tissue infiltrate of immature myeloid elements. The mean age at presentation is 48 years and most patients present with a solitary lesion. In the head and neck, they have been reported in the skull, face, orbit and paranasal sinuses. Extramedullary lesions have been reported in the tonsils, the oral and nasal cavities and within the lacrimal, thyroid and salivary glands. They are also referred to as chloromas, a term used to describe their green colour seen on sectioning. The prognosis of patients with AML does not change in the presence of a chloroma, however, in patients with CML it may represent worsening of the disease as their presence is associated with the acute or blastic phase. Approximately 30% of patients with chloromas have no haematological disease at the time of presentation.

269
Q

A 58-year-old man presents with impaired vision and intractable headaches. He has a history of recurrent sinusitis. Examination reveals left-sided proptosis and a palpable mass in the superomedial aspect of the orbit. CT demonstrates a soft-tissue mass causing expansion and erosion of the left frontal sinus. There is peripheral enhancement post-contrast injection. The next radiological step should be: [B2 Q29]
a. Referral to an appropriate clinician– you have made a confident diagnosis of a benign aetiology
b. MRI of the head and neck– you have made a confident diagnosis of a malignant aetiology and wish to stage its local spread
c. MRI of the head and neck– you are unsure of the nature of the aetiology and want to further characterise it
d. Perform a staging CT– you are concerned this may be metastatic disease
e. Perform an ultrasound scan of the orbit– to further characterise the lesion

A

Referral to an appropriate clinician – you have made a confident diagnosis of a benign aetiology
This is almost certainly a mucocoele. Mucocoeles represent the end stage of a chronically obstructed sinus. They most commonly affect the frontal sinus (60%), with ethmoid (30%), maxillary (10%) and sphenoid (rare) following respectively. Patients present with symptoms as described in the question. Increased intrasinus pressure results in expansion and erosion of the sinus walls. There may be a surrounding zone of bone sclerosis. Contrast injection typically reveals rim enhancement, which helps to differentiate from the more solid enhancement pattern of neoplasms.

270
Q

In normal anatomy of the nasal cavity, which structure opens into the inferior meatus below the inferior nasal turbinate? [B4 Q82]
a. anterior ethmoidal ostium
b. posterior ethmoidal ostium
c. frontal sinus ostium
d. maxillary sinus ostium
e. nasolacrimal duct

A

Nasolacrimal duct
The lateral nasal wall is separated into superior, middle, and inferior meatuses by three curled bony shelves called turbinates (or conchae). The nasolacrimal duct opens into the anterior aspect of the inferior meatus and is usually the only opening seen there. The other ostia all open into the middle meatus, with the exceptions of the posterior ethmoidal ostia (superior meatus) and sphenoidal ostia (posterior to the superior turbinate in the sphenoethmoidal recess). The sphenopalatine foramen lies inferior to the sphenoethmoidal recess posterior to the middle turbinate.

271
Q

A 20-year-old man presents with swelling around his left eye. A CT scan shows a high attenuation mass lesion which expands the ethmoid air cells with bony erosion. There are small punctate calcifications seen within the mass. On MRI, the mass returns low signal on T1 and T2. The most likely diagnosis is? [B5 Q17]
(a) Fungal sinusitis
(b) Chronic sinonasal polyposis
(c) Nasopharyngeal carcinoma
(d) Juvenile angiofibroma
(e) Chronic sinusitis

A

Fungal sinusitis
CT findings are typical, showing a hyperdense lesion with calcifications and bony erosion. The ethmoid sinus is most involved and bony expansion with erosion is characteristic. On MRI, the lesion is low signal on T1 and T2 due to high fungal mycelial iron, magnesium and manganese from amino acid metabolism.

272
Q

A 15-year-old girl presents with symptoms of chronic sinusitis. CT of the paranasal sinuses shows a low-density mass opacifying the right maxillary antrum and extending to the posterior choana. No bony destruction is seen. The left maxillary antrum also shows mucosal thickening. The most likely diagnosis is? [B5 Q18]
(a) Juvenile angiofibroma
(b) Antro-choanal polyp
(c) Fungal sinusitis
(d) Inverted papilloma
(e) Intranasal glioma

A

Antrochoanal polyp
These are seen as low attenuating masses from the maxillary antrum extending through a sinus ostium to the choana. They show non-aggressive features with peripheral enhancement. MRI shows a hypo to variable signal on T1 with hyperintense on T2 with peripheral enhancement. Juvenile angiofibroma is seen in adolescent males with an intensely enhancing mass extending to the posterior nasopharynx. Inverted papilloma are seen in older males as a locally aggressive mass of the middle meatus extending into the maxillary sinus. Intranasal glioma usually presents at birth as a very soft mass centred at the nasal dorsum.

273
Q

A 9-year-old boy presents with chronic right facial pain. Radiography shows an opaque right maxillary antrum. A CT scan of the paranasal sinuses shows that the right maxillary antrum is filled with soft tissue with destruction of medial and posterior bony walls. No significant sinus mucosal disease is seen in the other paranasal sinuses. What is the most likely diagnosis? [B5 Q49]
(a) Fungal infection
(b) Allergic sinusitis
(c) Rhabdomyosarcoma
(d) Antrochoanal polyp
(e) Acute sinusitis

A

Rhabdomyosarcoma
Bone destruction suggests an aggressive lesion at this site and rhabdomyosarcoma would be the most likely diagnosis in a young person.

274
Q

A 40-year-old migrant worker of Chinese origin presents with a chronic history of nasal congestion and recent epistaxis. CT of the paranasal sinuses shows a large soft tissue mass in the post-nasal space. There is also bony destruction with erosion of the basisphenoid. What is the most likely diagnosis? [B5 Q50]
(a) Chronic polyposis
(b) Juvenile angiofibroma
(c) Nasopharyngeal carcinoma
(d) Fungal infection
(e) Pharyngeal abscess

A

Nasopharyngeal carcinoma
The lesion is aggressive with bony destruction and is likely to be malignant. Nasopharyngeal carcinomas have a high incidence in the Chinese population. It can feature bony destruction of the basisphenoid, basiocciput or the petrous tip. There may also be cranial nerve involvement, either at their exit through the foramina or secondary to intracranial extension.

275
Q

15-year-old male presents with a history of recurrent epistaxis and nasal obstruction. MRI demonstrates a lesion centred at the sphenopalatine foramen, which is hypointense on T1WI and heterogeneously intermediate signal on T2WI. Intense lesional enhancement and multiple flow voids are noted on post-gadolinium T1WI. What is the diagnosis? [B1 Q38]
A. Ludwig angina.
B. Nasopharyngeal carcinoma.
C. Inverted papilloma.
D. Juvenile angiofibroma.
E. Glomus jugulare

A

Juvenile angiofibromas
Juvenile angiofibromas are benign but locally aggressive tumours with high vascularity. They typically occur in adolescent boys and present with recurrent epistaxis and nasal obstruction. They are centred within the sphenopalatine foramen and involve the pterygopalatine fossa, producing a bowed appearance of the posterior wall of maxillary sinus and widening of pterygopalatine fossa, inferior orbital, and pteryogomaxillary fissures. Osseous erosion is commonly seen. The specific differentiating feature on MRI is the presence of multiple flow voids on T2WI and enhanced T1WI.

276
Q

A 50-year-old male undergoes an MR carotid angiogram on which an incidental soft-tissue mass is noted in right parapharyngeal soft tissue. The mass displaces the right parapharyngeal space anteromedially. What is the location of the soft-tissue mass? [B1 Q45]
A. Masticator space.
B. Carotid space.
C. Retropharyngeal space.
D. Mucosal space.
E. Parotid space.

A

Parotid space.
Loss of symmetry and displacement of the parapharyngeal space are useful for lesion identification and localization in the parapharyngeal soft tissues. A thorough knowledge of the anatomical relationship between the spaces is essential. The parapharyngeal space is shaped like an inverted pyramid with the apex pointing inferiorly toward the greater cornu of the hyoid bone and the skull base demarcates the base superiorly. A lesion arising from the parotid space displaces the fat in the parapharyngeal space anteromedially. A lesion in the masticator space displaces the parapharyngeal fat postero-medially. Carotid space lesions displace it anteriorly, mucosal space lesions displace it postero-laterally, and retropharyngeal space lesions displace it anterolaterally. Posterior displacement of the carotid space or parapharyngeal fat completely surrounding a lesion localizes it to the parapharyngeal space.

277
Q

A 5-year-old girl with a clinical suspicion of retropharyngeal abscess is referred for MRI of the neck. Which of the following features on MRI is useful in differentiating retropharyngeal abscess from retropharyngeal suppurative lymph node? [B1 Q55]
A. Enhancing wall.
B. Rounded or ovoid configuration.
C. Mass effect.
D. Filling of retropharyngeal space from side to side.
E. Primary infection source such as otitis media or tonsillitis.

A

Filling of retropharyngeal space from side to side.
Understanding the retropharyngeal space anatomy is crucial in differentiating retropharyngeal space abscess and retropharyngeal suppurative lymph node. The retropharyngeal space is bounded by visceral fascia covering the pharynx and oesophagus anteriorly, the prevertebral fascia covering the prevertebral muscles posteriorly, and the carotid sheaths laterally. A retropharyngeal suppurative lymph node is unilateral, whereas a retropharyngeal abscess fills the entire retropharyngeal space from side to side. The differentiation is important because many cases of suppurative lymph nodes do not have purulent material at surgery. The treatment for suppurative lymph nodes is a trial of antibiotics if the patient is stable. Surgical drainage is considered if there is progression or if the suppurative lymph node is large at presentation. The volume of central low density is a better predictor of purulence than the mere presence of rim enhancement and low-density centre.

278
Q

A 20-year-old female is under investigation for periodic halitosis. A CT scan reveals a well-defined, hypodense mass located between the longus colli muscles. There is no enhancement post-contrast injection. MRI demonstrates a midline cystic structure in the posterior roof of the nasopharynx. It shows high signal intensity on both T1 and T2 sequences. The most likely diagnosis is: [B2 Q13]
a. Benign polyp
b. Rathke’s pouch cyst
c. Ranulas
d. Tornwaldt’s cyst
e. Thyroglossal duct cyst

A

Tornwaldt’s cyst
Tornwaldt’s cyst is a benign mass typically located in the midline, between the longis colli muscles, in the posterior nasopharynx. They arise as a result of a focal adhesion between the ectoderm and regressing notochord. This causes the creation of a pouch but when the communication with the pouch is lost, a cyst develops. Tornwaldt’s cysts are usually asymptomatic and are picked up as incidental findings. Periodically, the pressure within the cyst increases causing the release of its contents into the nasopharynx. This leads to presentations including halitosis, foul taste in the mouth and persistent nasopharyngeal drainage. Peak age at presentation is 15–30 years. Imaging features can vary depending on the protein content within the cyst but typical features are of a well-delineated, thin-walled, midline cystic lesion measuring 2–10 mm in diameter. They are hypodense on CT, rarely calcify and do not enhance. They can be high or low on T1 (depending on protein content) but are high on T2-weighted imaging. Rathke’s pouch cysts are located anterior and cephalad to Tornwaldt’s cysts.

279
Q

Ultrasound examination of the face and neck is performed to investigate a buccal, soft-tissue mass that became noticeable during pregnancy. The lesion is heterogeneous and hypoechoic and has sinusoidal spaces demonstrating slow flow and circular calcifications. Which of the following is the most likely diagnosis? [B4 Q62]
a. benign lymph node
b. malignant lymph node
c. pleomorphic parotid adenoma
d. arteriovenous malformation
e. venous vascular malformation

A

Venous Vascular Malformation
Phleboliths if present are unique to vascular malformations. Arterial malformations are high flow, while venous, capillary or combined malformations are low flow. MRI is required to assess the full extent, particularly intraosseous and intracranial, of head and neck vascular malformations. Benign lymph nodes are smooth, elliptical and hypoechoic with hilar architecture and vascularity. Malignant lymph nodes are typically round, are hypoechoic, have no hilum and show peripheral vascularity. Malignant lymph nodes with necrosis are seen with squamous cell and papillary cell carcinoma of the thyroid. Internal punctate calcification is seen in metastases from papillary or medullary carcinoma of the thyroid.

280
Q

Which of the following structures lies in the parapharyngeal space? [B4 Q89]
a. internal carotid artery
b. hypoglossal nerve
c. maxillary artery
d. lingual tonsil
e. vagus nerve

A

Maxillary artery
The parapharyngeal space is triangular shaped and extends from the skull base to the hyoid. It contains fat, branches of the mandibular division of the trigeminal nerve, maxillary artery, ascending pharyngeal artery and pharyngeal venous plexus. The internal carotid artery, vagus nerve and hypoglossal nerve lie in the carotid space. The lingual tonsils lie in the pharyngeal mucosal space.

281
Q

A 55-year-old man undergoes 99mTc scintigraphy on which incidental note is made of multiple foci of increased tracer uptake in the parotid regions. Ultrasound of the parotid glands demonstrates bilateral multiple hypoechoic lesions with anechoic areas. On MRI, the lesions are of intermediate signal intensity on T1WI and intermediate signal intensity with focal areas of hyperintensity on T2WI. There is no enhancement following contrast administration. What is the likely diagnosis? [B1 Q46]
A. Pleomorphic adenomas.
B. Warthin tumours.
C. Lipomas.
D. Haemangiomas.
E. Mucoepidermoid carcinomas

A

Warthin tumours.
Warthin tumour or adenolymphoma is the second most common benign tumour of the parotid gland. It is usually solitary. Multiple or bilateral parotid masses and increased uptake on 99m Tc are strongly suggestive of Warthin tumour. Warthin tumours do not enhance following gadolinium administration.
Pleomorphic adenomas are usually solitary and unilateral. They appear as hypoechoic, lobulated, well-defined lesions with posterior acoustic enhancement on ultrasound scan (USS). On MRI, they are of intermediate signal on T1WI and hyperintense on T2WI. They demonstrate homogenous enhancement following gadolinium administration.
Mucoepidermoid carcinomas vary in appearances. Low-grade tumours are similar to pleomorphic adenomas. High-grade tumours are heterogenous with low to intermediate signal intensity on T1WI and T2WI. They demonstrate infiltrating margins and heterogenous enhancement following gadolinium administration.

282
Q

MRI of a mass in the parotid tail shows a well-circumscribed heterogeneous lesion with mixed cystic and enhancing solid components. Earlier US showed vessels in a hilar distribution with branches in the septa of the structure. There is no lymphadenopathy. What is the likely cause? [B3 Q6]
A. Warthin’s tumour
B. Ductal cyst
C. Lymphoepithelial cyst
D. Pseudoaneurysm from a branch of the external carotid artery
E. AVM

A

A.
Warthin’s tumours are benign salivary gland tumours, accounting for 6-10% of all parotid tumours. Warthin’s tumours are bilateral in 15% of cases.

283
Q

A 30-year-old man presents with a lump in the left cheek. Ultrasound examination shows an 8 mm hypoechoic and lobulated lesion with a hyperechoic centre. The most likely cause of the lesion is. [B5 Q2]
(a) Parotid duct stone
(b) Lymph node
(c) Warthin’s tumour
(d) Pleomorphic adenoma
(e) Abscess

A

Lymph node
Typical appearances of intraglandular lymph nodes are of a hypoechoic periphery with a fatty hyperechoic centre.

284
Q

A 70-year-old man with chronic rheumatoid arthritis presents with recurrent episodes of dry eyes, mouth, and bilateral parotid swellings. CT shows bilateral diffuse parotid swellings with punctate calcifications and heterogenous contrast enhancement. MRI shows diffuse cystic lesions within both parotids on STIR. The most likely diagnosis is? [B5 Q12]
(a) Sjögren syndrome
(b) Non-Hodgkin’s lymphoma of the parotid glands
(c) Warthin’s tumours
(d) Metastatic disease
(e) Bilateral pleomorphic adenoma

A

Sjögren syndrome
An autoimmune condition causing salivary and lacrimal gland destruction. The secondary type is commonly related to rheumatoid arthritis, presenting with recurrent dry eyes, mouth and skin and parotid swellings. CT and MRI appearances are typical as described.
Non-Hodgkin’s lymphoma has bilateral solid masses in the parotids and usually has chronic systemic manifestations. Warthin’s tumours are characteristically inhomogeneous and, if cystic, show mural nodules. Metastatic tumours are solid enhancing lesions and a primary would usually be apparent on further investigation. Pleomorphic adenoma is usually a unilateral, well demarcated, solid, intra-parotid lesion with contrast enhancement.

285
Q

A 25-year-old woman is brought to the A&E department following an RTA. CT of brain and facial bones are requested for assessment of head and facial injuries. On the CT studies, incidental note is made of dense calcification of the falx cerebri, midface hypoplasia, and prognathism. Multiple cystic lesions are also noted in the mandible, which are associated with cortical expansion. Some of the lesions break through the cortex. Following contrast administration there is minimal peripheral enhancement. What is the underlying mandibular abnormality?
A. Radicular cyst.
B. Keratocystic odontogenic tumour.
C. Dentigerous cyst.
D. Stafne cyst.
E. Ameloblastoma

A

Keratocystic odontogenic tumour.
These are benign but locally aggressive developmental tumours affecting adults in the second to fourth decades. They are most commonly located in the mandibular ramus and body, and are associated with an impacted tooth. They may be unilocular or multilocular and often contain daughter cysts extending into the surrounding bone. They can be associated with cortical expansion and erosion. They demonstrate minimal peripheral enhancement with contrast.
The presence of multiple keratocystic odontogenic tumours raises the possibility of Gorlin basal cell nevus syndrome, which is an autosomal dominant disorder. Associated features include multiple basal cell carcinomas of the skin, mental retardation, midface hypoplasia, frontal bossing and prognathism, calcification of the falx and dura, and bifid ribs.

286
Q

A 60-year-old with treated SCC of the orpharynx undergoes ultrasound neck to evaluate a new swelling. Which feature most likely indicates malignant nodes? [B3 Q35]
A. Matting
B. Calcification
C. Ovoid shape
D. Hilar vascularity
E. Peripheral vasculitis

A

E
Matting is associated with tubercular lymph nodes. Increased peripheral vessels is a feature of malignant nodes.

287
Q

A 38-year-old gentleman presents with a dull ache in his jaw. There is minor swelling over the left mandible. Plain radiographs demonstrate an expansile, multilocular, radiolucent lesion with internal septations involving the left body of the mandible. It is associated with an impacted tooth. CT shows infiltration of the adjacent soft tissues. There are no discernable foci of calcification. What is the most likely diagnosis?
a. Odontogenic myxoma
b. Dentigerous cyst
c. Odontogenic keratocyst
d. Ameloblastoma
e. Periapical cyst

A

Ameloblastoma
Ameloblastoma (adamantinoma of the jaw) is a benign, locally aggressive lesion that occurs mainly in patients between 30 and 50 years of age. They are found in the mandible (75%) and the maxilla (25%) and are often associated with an impacted/unerupted tooth. When in the mandible, they typically occur in the region of the molars/angle of the mandible.
Typically, ameloblastomas are radiolucent lesions that contain septa or locules of variable size, which produce a honeycombed appearance. The margin is usually well defined but when large it can produce jaw expansion with perforation of the cortex. They may infiltrate adjacent soft tissues and show local recurrence following excision.
Ameloblastomas may be unicystic but these tend to occur around the age of 20 years and there is also a very rare malignant variety which can cause lung metastases.
Odontogenic keratocysts, dentigerous cysts, odontogenic myxomas and periapical cysts are invariably unilocular.

288
Q

Which of the following most favours ameloblastoma compared with odontogenic keratocyst? [B3 Q38]
A. Growth in buccolingual direction
B. Growth in AP direction
C. Non-expansile
D. CT 40-50 HU
E. High SI on T2

A

A
Ameloblastomas grow in the buccolingual direction and are expansile.

289
Q

A 31-year-old woman presents with a painless mass involving the right side of the mandible. Radiographs show a well-defined, lucent lesion. Which of the following additional findings favour a diagnosis of ameloblastoma over a radicular cyst? [B4 Q77]
a. absence of matrix mineralization
b. location at the symphysis menti
c. location at the root of a tooth
d. soap-bubble appearance
e. rim of cortical bone

A

d
Soap-Bubble Appearance
Both ameloblastoma and radicular cysts most commonly present in the third to fifth decades of life as a painless lump, often as an incidental finding. Neither typically shows matrix mineralization. Ameloblastoma represents 10% of all odontogenic (developing during or after the formation of teeth) tumours, with the majority located in the posterior body or ramus. They are frequently associated with the crown of an impacted or unerupted tooth. They can vary in radiographic appearance, an expansile, multiloculated lesion being typical. They are often resected due to locally aggressive infiltration. Radicular cysts are unilocular with a rim of cortical bone and usually develop at the root apex of a carious tooth as the end stage of the inflammatory process.

290
Q

A 40-year-old male presents with pain in the jaw associated with dental caries. An orthopantomogram demonstrates a lytic lesion of 1 cm in diameter, associated with the apex of a carious tooth. The lesion has a thin rim of cortical bone and there is some resorption of the root. What is the likely diagnosis? [B4 Q92]
a. ameloblastoma
b. odontogenic keratocyst
c. dentigerous cyst
d. radicular cyst
e. odontoma

A

d
Radicular cyst
A radicular cyst is the commonest type of cyst in the jaw and is associated with the apex of a tooth. It may displace teeth and cause mild root resorption. Ameloblastoma (adamantinoma of the jaw), odontogenic keratocyst and dentigerous cyst are all associated with the crown of a tooth, usually unerupted, and can be difficult to distinguish from each other on imaging alone. They are all odontogenic with no mineralization and may be large and expand the mandible. Odontomas are the commonest odontogenic mass (67%) and show mineralization. They are seen in 10–20-year-olds presenting with single or multiple, tooth-like masses.

291
Q

You are reporting a CT scan of neck in a patient with a head and neck cancer. You see an enlarged necrotic jugulo-digastric lymph node on the right side and wish to describe the appropriate level of this lymph node in your report. What is the correct level? [B1 Q42]
A. I.
B. II.
C. III.
D. IV.
E. V.
F. VI.
G. VII

A

B
II
Lymph nodes in the neck have been divided into seven levels, generally for the purpose of squamous cell carcinoma staging. This is, however, not all inclusive, as the parotid nodes and retropharyngeal space nodes are not included in this system.
Level I: Below mylohyoid to hyoid bone anteriorly Level Ia: Submental Level Ib: Submandibular Level II: Jugulodigastric (base of skull to hyoid)
Level III: Deep cervical (hyoid to cricoid)
Level IV: Virchow (cricoid to clavicle) Level V: Posterior triangle groups
Level V: Accessory spinal (posterior triangle), Level Va: superior half, Level Vb: inferior half
Level VI: Prelaryngeal/pretracheal/Delphian node
Level VII: Superior mediastinal (between common carotid arteries (CCAs), below top of manubrium)
Lymph node levels of the neck. Radiopaedia.org.

292
Q

A 30-year-old male presents with a fluctuant swelling in the right side of the neck. On ultrasound examination, an anechoic lesion with posterior acoustic enhancement is noted along the anteromedial margin of the sternocleiodomastoid muscle, posterior to the submandibular gland and superficial to the carotid artery and internal jugular vein. There is no increased surrounding vascularity on power Doppler. What is the likely diagnosis? [B1 Q54]
A. Dermoid cyst.
B. Lymphangioma.
C. First branchial cleft cyst.
D. Abscess.
E. Second branchial cyst.

A

E
Second branchial cleft cyst.
The majority of branchial cleft anomalies arise from the second branchial cleft. The described findings are typical of a second branchial cleft cyst. Similar ultrasound appearances may be seen in a first branchial cleft cyst or a dermoid cyst. The anatomical location is the key to the diagnosis. Dermoid cysts are typically midline in location in the neck and first branchial cleft cysts are located in the region of parotid gland, external auditory canal, and angle of mandible.
Lymphangiomas are typically located in the posterior triangle. On ultrasound, they appear multiloculated with intervening thin septa. Abscesses appear as ill-defined, irregular collections with thick walls and internal debris. Surrounding soft-tissue oedema, hyperaemia, and enlarged adjacent lymph nodes are also noted.

293
Q

A 28-year-old woman presents with a mass in her neck. She gives a history of multiple parotid abscesses which have been refractory to drainage and antibiotics. The mass is located at the anteromedial border of her right sternocleidomastoid muscle. Ultrasound demonstrates a compressible mass with internal debris which is devoid of internal flow on Doppler imaging. MRI shows a cystic mass consisting of a curved rim of tissue pointing medially between the internal and external carotid arteries. There is slight capsular enhancement. What is the most likely diagnosis? [B2 Q18]
a. Cervical abscess
b. Necrotic neural tumour
c. Submandibular gland cyst
d. Necrotic inflammatory lymphadenopathy
e. Second branchial cleft cyst

A

e
Second branchial cleft cyst
Failure of involution of branchial clefts can lead to branchial cleft cysts, fistulae and/or sinuses. Second branchial cleft cysts account for 95% of all branchial cleft anomalies. Male and female incidence is equal and the typical age of presentation is 10–40 years.
Second branchial cleft cysts are classified into four types depending on their location. The most common is type II, which occurs along the anterior surface of the sternocleidomastoid muscle, lateral to the carotid space and posterior to the submandibular gland adhering to the great vessels.
On CT/MR the ‘beak sign’ is pathognomonic. This is a curved rim of tissue pointing medially between the internal and external carotid arteries.

294
Q

A 72-year-old man from a nursing home presents to the accident and emergency department. Nurses have noticed increasing confusion following a fall six days earlier. His inflammatory markers are normal. A non-contrast CT scan of the head demonstrates a crescent-shaped collection in the left fronto-parietal region. The collection is isodense to CSF and there is no midline shift, nor hydrocephalus. On T1-weighted MR imaging the lesion is isointense to CSF. The most likely diagnosis is: [B2 Q19]
a. Subdural hygroma
b. Brain atrophy
c. Subdural empyema
d. Chronic subdural haematoma
e. Enlarged subarachnoid space

A

a
Subdural hygroma
This is a traumatic subdural effusion which shows up as a localised CSF-fluid collection within the subdural space. They present in the elderly or in young children usually 6–30 days following trauma. The majority are asymptomatic but patients may present with increasing confusion or headaches. They are devoid of blood products on imaging, unlike chronic subdural haematomas. Subdural haematomas are also more likely to cause effacement of the ventricular system and loss of the normal sulci-gyral pattern.
Normal inflammatory markers and lack of pyrexia lessen the probability of an empyema.

295
Q

A seven-year-old boy presents with pain in his neck. His mother thinks she can feel a lump. Ultrasound shows a thick-walled cyst with internal echoes. It has a paramedian location within the strap muscles. MRI shows a heterogeneous cystic mass measuring 3cm in diameter. It demonstrates high signal on T1 and contains areas of low signal on T2. There is marked enhancement of the wall after administration of gadolinium. What is the most likely diagnosis? [B2 Q41]
a. Infected thyroglossal duct cyst
b. Fibroma
c. Branchial cleft cyst
d. Teratoma
e. Lymphangioma

A

a
Infected thyroglossal duct cyst
Thyroglossal duct cysts arise from the remnants of the embryonic thyroglossal duct and account for up to 70% of congenital neck masses in children. Typically, children present with a non-tender mass that elevates on swallowing. If infected there may be pain, local tenderness and recent increased growth. Most are midline, although they become more paramedian below the level of the hyoid. Approximately 20% are suprahyoid, 15% occur at the level of the hyoid and 65% are infrahyoid. Generally they are thin-walled cysts and show typical cystic imaging characteristics. If infected or if the cyst has haemorrhaged, high signal may be seen on T1 and low signal may be present on T2-weighted MR images. Haemorrhage and infection may also cause thickening and marked enhancement of the wall.

296
Q

A 46-year-old patient presents with an enlarged level IV lymph node in the neck. Histology from FNA demonstrates metastatic squamous cell carcinoma. Which of the following is the most likely site of the primary malignancy? [B4 Q10]
a. oral cavity
b. nasopharynx
c. tongue
d. salivary gland
e. larynx

A

e
Larynx
The lymph node chains in the neck are complex and consist of a superficial circular nodal group and a deep cervical chain. Supraglottic laryngeal squamous cell carcinoma tends to spread to the high internal jugular vein chain of nodes, which are in level II, with tumours of the epiglottis, aryepiglottic folds and pyriform sinuses most likely to present with adenopathy. This can be ipsilateral (most common), contralateral or bilateral. Subglottic tumours tend to spread to bilateral middle- or lower-level nodes of the internal jugular vein chain in level III or IV. The level IV lymph nodes also drain the hypopharynx, thyroid and upper oesophagus. The oral cavity, including the tongue and nasopharynx, drains to the level II nodes. The salivary glands drain to level I or II nodes. Tumours confined to the vocal cords do not normally metastasize to lymph nodes.

297
Q

A 43-year-old man presents with cough and numerous masses in the neck bilaterally. CT confirms multiple lymph nodes that enhance peripherally and contain areas of calcification. A cavitating lesion is noted in the right lung apex. What is the most likely diagnosis? [B4 Q26]
a. tuberculosis
b. metastatic laryngeal carcinoma
c. metastatic nasopharyngeal carcinoma
d. metastatic papillary thyroid carcinoma
e. metastatic squamous cell carcinoma of the lung

A

a
Tuberculosis
Tuberculous lymphadenitis is the most common form of head and neck tuberculosis, representing 15% of all extrapulmonary tuberculous infections. It is frequently bilateral and, the more inferior the involved nodes, the higher the prevalence of associated pulmonary disease. Peripherally enhancing lymph nodes are seen with tuberculosis, metastatic disease (usually squamous cell tumours), lymphoma or infection. The presence of calcification suggests tuberculosis, but it may also be seen with papillary or medullary thyroid carcinoma. The other diagnoses listed may cause cervical lymphadenopathy, but the lung lesion would be unusual in all except a cavitating squamous cell carcinoma of the lung.

298
Q

A 62-year-old male patient presents with a mass in the right side of his neck. CT shows an enlarged, right-sided lymph node anterior to sternocleidomastoid, below the level of the hyoid bone and above the cricoid ring. At which level does the node lie? [B4 Q44]
a. level I
b. level II
c. level III
d. level IV
e. level V

A

c
Level III
Levels II, III and IV lie anterior to the sternocleidomastoid. Level II is from base of skull to hyoid, level III from hyoid to cricoid, and level IV from cricoid to clavicle. Level I is the submandibular space. Level V is posterior to sternocleidomastoid.

299
Q

Regarding radionuclide imaging of thyroid cancers, which radiological finding best fits the diagnosis? [B2 Q30]
a. Usually concentrates radioiodine – follicular carcinoma
b. Usually concentrates pertechnetate – papillary carcinoma
c. Usually concentrates pertechnetate and radioiodine – papillary carcinoma
d. No radioiodine or pertechnetate uptake but frequently concentrates 201 Thallium – medullary carcinoma
e. No radioiodine or pertechnetate uptake but frequently concentrates 201 Thallium – anaplastic carcinoma

A

No radioiodine or pertechnetate uptake but frequently concentrates thallium201 – medullary carcinoma
Types of thyroid carcinoma in order of worsening prognosis are papillary, follicular, medullary and anaplastic.
Papillary tumours usually concentrate radioiodine, follicular tumours concentrate pertechnetate but fail to accumulate radioiodine, and anaplastic tumours show no radioiodine uptake.

300
Q

A 48-year-old woman presents with symptoms of hyperparathyroidism. Radionuclide and ultrasound imaging suggest the cause is a solitary parathyroid adenoma. The surgeon requests further localisation with MRI prior to surgery. Which imaging sequence and plane would you choose as the most sensitive for detection of the adenoma? [B2 Q35]
a. T1-weighted in the axial plane
b. T2-weighted in the coronal plane
c. FLAIR in the coronal plane
d. T2 fat-suppressed in the axial plane
e. Gradient-echo in the axial plane

A

T2 fat-suppressed in the axial plane

301
Q

A 50-year-old woman presents with a palpable lump in her neck. Imaging demonstrates a malignant looking mass in the thyroid gland. Which of the following findings would direct you towards a confident diagnosis? [B2 Q37]
a. Complex mass with areas of necrosis – papillary carcinoma
b. Calcified lymph nodes – medullary carcinoma
c. Osteosclerotic bone metastases – follicular carcinoma
d. Intra-tumoural calcifications – anaplastic carcinoma
e. regional lymphadenopathy – anaplastic carcinoma

A

Calcified lymph nodes – medullary carcinoma
Lymph node calcification accompanied by a thyroid tumour is highly suggestive of medullary carcinoma. This tumour arises from parafollicular C-cells and can cause elevated calcitonin levels. The familial form of medullary carcinoma is associated with MEN II (parathyroid hyperplasia, phaeochromocytoma).

302
Q

You are asked to provide an opinion on skull and facial radiographs of an infant. The history provided is recent fall, known dwarfism. The radiographs show brachycephaly, widened sutures, relatively large sella, wormian bones, delayed dentition, decreased pneumatisation of the paranasal sinuses and hypertelorism. No fracture is demonstrated. What is the most likely diagnosis? [B2 Q53]
a. Cleidocranial dysostosis
b. Hypophosphatasia
c. Hypothyroidism
d. Achondroplasia
e. Pyknodysostosis

A

Hypothyroidism
Many of the features listed may be seen in any of the differential conditions provided. However, large sella and hypertelorism favour a diagnosis of hypothyroidism.

303
Q

A 40-year-old woman under investigation for hypoparathyroidism undergoes ultrasound of the neck. Which is the best answer regarding parathyroid adenoma? [B3 Q34]
A. Usually indistinguishable from margin of thyroid gland
B. High T1
C. Low T2
D. Typically hypoechoic nodules
E. Usually hypodense on CT

A

D
Parathyroid adenomas are typically well circumscribed, round or oval hypoechoic nodules. They are usually low signal on T1 and high on T2. However, they may appear as high signal on both in the presence of haemorrhagic change. Parathyroid adenomas are usually hyperdense on CT.

304
Q

Which of the following is a case of increased radiotracer uptake in the thyroid gland? [B3 Q48]
A. Thyroid replacement therapy
B. Lithium therapy
C. Propylthiouracil
D. Chronic thyroiditis
E. Radioiodine ablation

A

B
Other causes include early thyroiditis, hyperthyroidism, and rebound after withdrawal of anti-thyroid medications.

305
Q

On a thyroid radioisotope study, which is a cause of a hot thyroid nodule? [B3 Q49]
A. Involutional nodule
B. Focal thyroiditis
C. Adenomatous hyperplasia
D. Granuloma
E. Abscess

A

C
Other causes include autonomous adenoma and, very rarely, thyroid cancer. Any hot nodule on Tc-99m should be imaged with I-123 to differentiate between autonomous and cancerous lesions.

306
Q

A patient presents with colicky, right-sided, abdominal pain and is found to have multiple renal calculi. Blood tests reveal hypercalcaemia and hyperparathyroidism. Ultrasound scan shows a hypoechoic nodule posterior to the left lobe of the thyroid suggestive of parathyroid adenoma. Which features on parathyroid scintigraphy, using pertechnetate and sestamibi (with delayed washout images), would suggest a functioning parathyroid adenoma at this site? [B4 Q51]
a. increased uptake on pertechnetate and sestamibi studies with delayed washout
b. no uptake on pertechnetate; increased uptake on sestamibi with normal washout
c. no uptake on pertechnetate; increased uptake on sestamibi with delayed washout
d. no uptake on pertechnetate or sestamibi studies including delayed image
e. increased uptake on pertechnetate and no uptake on sestamibi study including delayed image

A

No uptake on pertechnetate, increased uptake on sestamibi with delayed washout
Parathyroid adenomas do not take up pertechnetate, which is accumulated by thyroid tissue, whereas sestamibi is taken up by both thyroid and parathyroid tissue. Subtraction of these two images can then be used to show any difference that can be attributed to a parathyroid adenoma. The delayed image typically shows retention of sestamibi in parathyroid adenomas (delayed washout) compared with normal parathyroid and thyroid. Small parathyroid adenomas are often missed by scintigraphy.

307
Q

A 43-year-old female presents on an ultrasound list for a thyroid FNA. She has an enlarging thyroid gland, right lobe more so than the left, and is biochemically euthyroid. Ultrasound scan shows multiple solid nodules with some cystic areas and foci of calcification throughout both lobes and the isthmus, with no obvious dominant nodule. Regarding FNA, how should the radiologist proceed? [B4 Q90]
a. target solid lesion
b. target calcification
c. use multiple passes in both lobes
d. use single pass at multiple sites
e. do not perform FNA

A

Do not perform FNA
The appearances are characteristic of a multinodular goitre, and FNA can be misleading in these cases. Imaging diagnosis alone is usually sufficient. If there is a dominant nodule that is clearly larger than the others or a solitary enlarging nodule found on follow-up, then FNA should be performed to exclude malignancy.

308
Q

A patient with multiple previous ischaemic strokes has an MRI including T2W and DWI
sequences. In the left frontal lobe, there is a region of increased T2 signal. The same area is
dark on the DWI and bright on the ADC map. Which of the following is the most likely age of
this region of ischaemia? [B4 Q43]

a. 10 minutes
b. 30 minutes
c. 3 days
d. 2 weeks
e. 4 months

A

4 months

The imaging findings are in keeping with a chronic infarct, in which the unrestricted extra
water within gliosis gives a high T2 signal, dark DWI and bright ADC. Cytotoxic oedema occurs
because of early ischaemic damage to the cell membrane Na+ /K + ATPase pump. This can be
seen as early as 30 minutes after symptom onset as a bright DWI region. T2W images will be
normal this early and the corresponding ADC map dark. DWI signal increases during the first
week due to restricted diffusion but may remain bright for a prolonged period due to T2
‘shine-through’. ADC will return to normal at 1–4 weeks (pseudonormalization) when
restricted diffusion is matched by increasing amounts of vasogenic oedema that is not
restricted. Long-term gliosis has extra water that is not restricted. DWI can be usefully
thought of as a T2 signal diminished by net water movement. Since the DWI can be bright